NBCOT

Ace your homework & exams now with Quizwiz!

An OT designs a dynamic splint for an individual recovering from tendon repair. To ensure an appropriate angle of pull, the therapist positions an outrigger at: a. 110 degrees to the joint b. 60 degrees to the joint c. 90 degrees to the joint d. 45 degrees to the joint

C. 90* to the joint

an individual with hemiplegia has inadequate ankle dorsiflexion on the affected side. Which of the following equipment is best for the OT to recommend the person use to compensate for this deficit and facilitate safe and effective ambulation? a. an ankle-foot orthosis (AFO) b. a wide based quad cane (WBQC) c. a narrow based quad cane (NBQC) d. knee-ankle-foot orthosis (KAFO)

a. an AFO The knee is not involved so a KAFO is not indicated. The other options would be indicated for a person with poor balance.

an individual with amyotrophic lateral sclerosis requires the use of an environmental control unit to access electrical devices and a personal emergency response system. The individual lives alone and self directs personal care attendants to perform personal activities of daily living. During instruction to the individual on the capabilities and use of the ECU, which is the most important for the OT to discuss with the client? a. the ECUs back-up power source and charging instructions. b. additional assistive technology available. c. augmentative alternative communication options. d. funding for assistive technology

a. the ECUs back-up power source and charging instructions

An OT measures the active ROM of an individual's index finger. The measurements are MCP 0-45, PIP 10-60, and DIP 10-40. which is the most accurate for the therapist to document as the finger's total active motion (TAM)? a. 62.5 b. 125.0 c. 72.5 d. 145.0

b. 125

While completing the screening, the OT observes that the patient uses only the left side to participate in activities. The therapist suspects that the patient has unilateral inattention and difficulties with body scheme. Which should the therapist have the patient do during the OT eval to determine if these deficits are present? a. point to various body parts named by the therapist b. complete an UE dressing eval c. Complete the draw-a-person d. complete a body

b. complete a UE dressing eval

a person with a TBI is assessed to score a 6 on the glasgow coma scale. Which should the OT use to initiate intervention with this person? a. demonstrated directions b. sensory stimulation c. verbal cues d. hand-over-hand assistance

b. sensory stimulation a score of 6 on the GCS is just one level above completely non-responsive coma. As a result, this person can open eyes in response to pain and make incomprehensible sounds. Other choices are too high-level for this individual.

A OTR observes two children at preschool playing with a pile of blocks. They are sitting next to each other. One child is stacking blocks to make a tower. The other child is sorting all the blue blocks into a container. Which BEST describes this type of play? a. symbolic play b. associative play c. parallel play d. onlooker play

c. parallel play

an OT provides consultation services to a psychogeriatric unit for individuals with mid-stage dementia. In designing activity program, which groups are the best for the OT to include? a. reality orientation b. sensory stimulation c. reminiscence d. coping skills

c. reminiscence reminiscence groups are designed to review past life experiences to promote use of intact long-term memory.

An adolescent with spina bifida at the C-8 level wants to access the new computerized play systems that was received as a birthday gift. Which is the best adaptation for the OT to recommend the adolescent use to access this system? a. a chain switch b. a tenodesis splint c. a dorsal wrist splint with a universal cuff d. a joystick control

d. a joystick control a chain switch would be indicated for a C3/4 level lesion. A tenodesis splint would be indicated for C6 lesion, and a dorsal splint with a universal cuff is indicated for a C5 lesion.

the private practitioners pay for their newly hired OTs' registration for an advanced course on pediatric assessment. Which is the most important outcome of attending this course for the OT? a. fulfillment of CEUs for independent credentialing agencies d. keeping up-to-date on current trends in OT c. Networking with other pediatric professionals d. improvement of their professional skills and competencies

d. improvement of their professional skills and competencies

a person experiencing an acute manic episode is completing the admission process to an inpatient psych unit. The intake coordinator has to unexpectedly complete an emergency admission. The admissions coordinator asks the OT to spend some one on one time with this individual until the coordinator can return to complete the intake process. Which is the best way for the OT to use this time with the client? a. discuss the precipitants to the hospitalization with the person b. ask the person to make positive statements about him/herself c. have the person do a craft activity requiring attention to detail d. take a walk around the unit to orient the person

d. take a walk around the unit to orient the person walking with the person can allow for some energy release which is important for a person with mania. It is a non-threatening activity that can facilitate interaction. Upon admission, the person may be uncomfortable discussing precipitants to the hospitalization and may have difficulty making positive statements about his/herself. Activities that require concentration and attention can also be difficulty with acute mania.

a large general hospital in a n urban area initiates several quality improvement committees that include OTs. Which of the following does participation in these committees provide to the OT? a. the ability to develop personal performance skills b. the capability to enhance leadership skills c. the power to promote OT services d. the opportunity to improve overall service delivery

d. the opportunity to improve overall service delivery Quality improvement analysis is supposed to improve service quality and meet the needs of the population

The OTR is consulting with a teacher about strategies to use during handwriting instruction for a school-age child who has decreased proprioceptive awareness in his hand and wrist. What is the MOST EFFECTIVE strategy for the OTR to suggest for the teacher to explore? a. writing with a pen instead of a pencil b. providing an angled desktop surface c. writing with a triangle grip on the pencil d. using a pediatric-weighted pencil holder

d. using a pediatric-weighted pencil holder The weighted holder will make the pencil heavier to increase awareness to the position of the pencil and increase the stability of the wrist when the child writes. Writing with a pen instead is a strategy that could be used with children that press to hard or are sensitive to the sound of pencil writing on paper. An angled surface may be effective when a child is first learning handwriting because it promotes wrist extension and an efficient grasp. A triangle grip addresses the grasp pattern, not the proprioceptive issue.

During a home visit, a client who had incurred a CVA reports difficulty finding objects during BADL and IADL. The client reports that the directions family members provide (e.g., look on the refrigerator door, look in the medicine cabinet) are not helpful. Which cognitive-perceptual ability should the OT further evaluate? a. stereognosis b. organization c. spatial relations d. visual closure

d. visual closure This behavior may be evidence of difficulties with visual closure since the person may \not be able to find an item if its in its complete form (I.e., if it is covered partially by other objects in the fridge or cabinet). Spatial relations are the ability to relate objects in relationship to each other (up/down, front/back)

Experimenting with different sexual positions would be an effective recommendation for individuals with what deficits? Scheduling sexual expression activities after rest periods is effective for who?

neuromuscular or musculoskeletal deficits a person who experiences fatigue that limits activity (ex: MS)

An OT constructs a splint for a person who incurred full thickness facial and anterior neck burns. The therapist splints the neck in the position of: a. extension b. 15* flexion c. 15* lateral flexion d. 15* hyperextension

a. extension the neck should be splinted in extension because the contracture tendency for the neck is flexion.

During an OT intervention session, a client with a left CVA demonstrates extinction to the right and a tendency to ignore items on the right side. When documenting this behavior, what should the therapist report? a. agnosia b. unilateral inattention c. poor right/left discrimination d. poor visual scanning

b. unilateral inattention This is when the individual neglects the side of the body contralateral to the CVA site.

an OT completes an ergonomic examination of a computer programmer and the programmer's workstation. Which is the best recommendation for the therapist to make to ensure the programmer uses ideal wrist and elbow positioning? a. elevate the keyboard to increase wrist flexion b. use a keyboard rest to maintain a neutral wrist position c. lower the keyboard to increase wrist extension d. add armrests to keep the elbows away from the body

b. use a keyboard rest to maintain a neutral wrist position

an entry-level OTA recently hired for an outpatient rehab clinic requires supervision during the supervisor's scheduled vacations. To maximize departmental efficacy, who should provide supervision to the OTA? a. the rehabilitation clinic's administrator b. the OTA with 1 year of experience c. an OT d. an OTA with advanced credentialing

c. an OT

to ensure the provision of best practice, the new entry-level OT will be provided with supervision of their caseloads. At what level should this supervision be provided? a. routine b. general c. close d. minimal

c. close It is recommended that entry-level OTs receive close supervision (daily). Intermediate level OTs can receive routine-general supervision (every 2 wks-at least monthly) respectively.

Person in this case: has degenerative joint disease and incurred an injury on the right hand. Client has pain, stiffness, and extreme temp changes in the hand. The person also has edema and hypertrophic changes consistent with CRPS. the OT completes the pain evaluation. What additional eval methods should the therapist use to assess this client? a. vigormeter b. dynamometer c. volumeter d. sphygmomanometer

c. volumeter volumeters measure edema.

An individual cannot independently get from a supine position to a sitting position. The person has good scapular, shoulder, and elbow muscle strength. Which of the following should the OT recommend as most effective for the client to use to improve bed mobility? a. a leg lifter b. a bed rail assist c. a log roll technique d. a rope ladder

d. a rope ladder The other options do not adequately assist with independent supine to sitting. A bed rail is used for help with rising from sitting to standing.

the OT evaluates a client's pain by asking the client which movement or activities elicit pain. Which of the following is the therapist assessing? a. quality of pain b. location of pain c. intensity of pain d. triggers of pain

d. triggers of pain

Person in this case: has degenerative joint disease and incurred an injury on the right hand. Client has pain, stiffness, and extreme temp changes in the hand. The person also has edema and hypertrophic changes consistent with CRPS. The OT plans intervention to address the client's goals to engage in meaningful occupations. Which PAM should the OT use in preparation for functional activity a. TENS b. contrast baths c. whirlpool d. none, PAMs are contraindicated for this diagnosis.

B. Contrast baths CRPS type 1 is a vasomotor dysfunction. Contrast baths facilitate the opening and closing of the vascular and lymphatic vessels and is the preferred modality for CRPS.

An elder with peripheral neuropathies resulting from the chronic effects of diabetes expresses concern over his/her sexual relationship with his/her partner. The most appropriate response for the OT to make is to suggest the elder and his/her partner: a. experiment with different positions during sexual expression activities b. focus on intact senses and areas of intact sensation c. schedule sexual expression activities around rest periods d. position the partner to the unaffected side

B. Focus on intact senses and areas of intact sensation

An older adult with a diagnosis of OA in both knees is referred to inpatient OT. During the screening, the patient expresses a desire to return home to live alone independently. Which should the OT do first in response to the patient's stated goal? a. recommend adaptations to the patient's home environment to increase safety b. evaluate the patient's BADL and IADL using a standard measure c. teach the patient energy conservation techniques to use during IADL tasks d. Train the patient in a home resistive exercise program to build strength and ROM

B. evaluate the patient's BADL and IADL using a standard measure The patient has only been screened, they need to be evaluated next before any recommendations or interventions can begin.

An elder diagnosed with dementia, Alzheimer's type was recently admitted to a SNF. The OTA working with the resident reports to the OT that during the morning care session, the OTA observed bruises on the residents back and upper arms. Which action is best for the OT to initially take? a. talk to the resident to obtain more info b. follow facility procedures for investigating resident safety c. contact the resident's family to obtain more info d. contact the state office for adult protective services

B. follow facility procedures for investigating resident safety whenever there are concerns for a client's safety, facility procedures for investigating must be followed. Talking to the resident may not enable the OT to obtain the accurate info due to the diagnosis of dementia.

A single parent is hospitalized for an exacerbation of schizophrenia. Actively psychotic upon admission, the client has been stabilized on medicine. The client is currently not demonstrating hallucinations or delusions. Residual deficits include several negative symptoms and decreased cognitive skills. At the team meeting, the psychiatrist decides to discharge the client within 48 hours. The client lives with his/her elementary aged children. Which is the best recommendation for the OT to make during this team meeting? a. an extension of hospitalization to further evaluate cognitive skills b. a family meeting to discuss the need for children to assume home management tasks. c. a home visit to assess the client's safety skills within the home environment d. a referral to social services to explore foster care for the children

C. home visit to assess the client's safety skills within the home environment

a toddler with severe congenital anomalies and irreparable cleft palate has a DNR order. While being fitted for a molded seat for a wheelchair, the child stops breathing and turns blue. The entry-level OT determines that the child has a brachial pulse. Which of the following is the first action the therapist should take in response to the situation? a. inform the physician about the situation and the child's DNR order b. call the supervising OT to discuss the best response c. Implement the facility's emergency procedure d. perform obstructed airway maneuver and monitor HR for 5 minutes.

The child is not breathing and the therapist should initiate the facility's plan for medical emergencies and cardiac codes. It is not the decision of the therapist to withhold treatment or to provide intervention. The medical team that responds to the code must make the best decision about the DNR.

An OT plans a task-oriented group for adolescent girls recently diagnoses with anorexia. Which is the best activity for the therapist to include in the initial session of the group? a. making cards to send to veterans in a local hospital b. baking cookies for the residents in a homeless shelter c. performing low impact aerobic exercises d. composing lyrics and melody for a group song

a task-oriented group utilizes a psychodynamic approach to increase participants' understanding of their needs, values, ideas, feelings, and behaviors. Activities are selected and designed to facilitate self-expression and the exploration of feelings, thoughts, and behaviors. Composing a song allows each participant to express themselves.

During OT screening session, the therapist observes that a child bangs objects on a tabletop but is unable to give up a toy upon request. The OT documents these behaviors. Which developmental level would be most accurate for the therapist to report the child's behavior indicates? a. 3-4 months b. 7-8 months c. 9-10 months d. 11-12 months

a. 3-4 months At this age children can bang toys on a tabletop but do not have voluntary release. At 7-8 months, children begin to be able to give up objects with an assisted release, and voluntary release is achieved at 12 months.

an OT conducts an on-site accessibility assessment with a building contractor who is remodeling an apartment building. The therapist recommends modifications for doorways which do not meet minimum accessibility width standards. The contractor states the buildings owner wants to exceed minimum standards without incurring unreasonable costs. Which is the most preferred doorway measurement for the OT to recommend the contractor use? a. 36 inches b. 32 inches c. 34 inches d. 38 inches

a. 36 inches This is the preferred doorway measurement for wheelchair accessibility that exceeds the minimum accessibility standard of 32 inches.

An OTR is working in a school system and is about to evaluate a student in the fourth grade who has a learning disability and attention-deficit/hyperactivity disorder (ADHD). Which standardized assessments would be MOST APPROPRIATE for the OTR to use in the evaluation process? a. Bruininks-Osterestsky Test of Motor Proficiency and Test of Handwriting skills b. Peabody Developmental Motor Scales and the Vineland Adaptive Behavior Scales c. Hawaii Early Learning Profile and clinical observations d. Wee-Functional Independence Measure (FIM) and the Denver II Developmental Screening

a. Bruininks-Osterestsky Test of Motor Proficiency and Test of Handwriting skills These assessments measure skills and school-related occupational performance such as writing and fine motor coordination. The HELP is only used for children up to 6 yrs, and the Peabody only goes up to 5 years. The Wee-FIM is also for much younger children.

an OT working in a SCI unit evals a client for bowel and bladder training. The client and therapist set a goal of independence in all aspects of bowel and bladder care, including skin inspection. Which of the following is the highest of most severe level of complete SCI injury the client can have to achieve this goal? a. C7-C8 b. C4-C5 c. C5 d. C6

a. C7-C8 Individuals with SCIs at the other levels do not have the fine motor control to perform the skills independently.

An OT works with a survivor of a house fire. The client has burns on both hands that limit thumb mobility. The client identifies a personal goal of being able to pick up and hold cans to enable independent shopping and meal preparation activities. Which movement of the thumb should the goal statement include as the desired functional outcome? a. CMC palmar abduction b. CMC extension c. MCP flexion d. IP flexion

a. CMC palmar abduction this movement is the major movement required for the thumb to pick up cans. It is when the thumb moves away from the palm in a plane perpendicular. CMC extension is the hitchhiking position. The other two choices will not expand the web space to pick up a can.

An OT evals a person who complains of persistent wrist pain after painting a house 3 weeks ago. The patient demonstrates signs and symptoms consistent with de Quervain's tenosynovitis. Which assessment measure should the therapist use to confirm the diagnosis? a. Finklestein's test b. Phalen's test c. Froment's signs d. Craig's test

a. Finklestein's test This test is specific for reproducing the pain associated with this condition of the abductor pollicis longus and extensor pollicis brevis. Froment's sign is for ulnar nerve dysfunction. Phalen's test is for median nerve (Carpal tunnel)

The OTR documents the results of the evaluation and notes a positive Phalen test. Which is the MOST appropriate for the OTR to note that the client is experiencing? a. Median nerve pain b. ulnar nerve pain c. peripheral nerves d. radial nerve pain

a. Median nerve pain This is like the prayer position. A positive test results from tingling in the median nerve distribution including the thumb, index, and middle fingers.

a teenager with spinal muscle atrophy shows decreased trunk balance and strength. UE strength and ROM are unchanged from the last eval. Which is the best recommendation for the OT to make? a. a re-evaluation of the client to be completed b. the client be referred to an orthotist for a soft spinal support c. the client be measured for a power wheelchair d. a trunk strengthening program to be initiated with the client

a. a re-eval of the client to be completed the most important action to take after noticing a change in the functional status of a person with a progressive condition is to re-evaluate. Based on the evaluation, further interventions can be planed.

An OTR is working in a juvenile detention center with inner-city teenagers who have experienced emotional and physical trauma. What trauma-informed sensory approach is MOST EFFECTIVE for the therapist to use to promote safety and support for recognizing and regulating sensory experiences? a. a crisis plan that helps teens identify what upsets them, how it feels in their body, and what activities calm them down b. a parachute game to illustrate movements to facilitate calm and alerting responses in their body c. bioenergetics exercises focusing on breathing and movement exercises to heighten emotional awareness and expression d. expressive therapy group where teens explore and uncover memories through creative writing, poetry, or art.

a. a crisis plan that helps teens identify what upsets them, how it feels in their body, and what activities calm them down A crisis plan will help the teens modulate how they are feeling in order to apply an appropriate SI intervention to help them calm down. Parachute games focus more on movement and ROM than understanding calming responses. Bioenergetic exercises are aimed at feeling expression and expressive therapy is a way to identify and express emotion but these may bring up negative emotions and may not promote safety.

a child with mild spastic diplegia wants to participate in neighborhood activities with peers. The family's main goal for the child is to ride a bike. Which bike is the best for the OT to recommend? a. a hand-propelled bike with hand brakes b. a foot-propelled bike with hand brakes c. a foot-propelled bike with foot brakes d. an adapted tricycle with foot brakes

a. a foot-propelled bike with hand breaks mild spastic diplegic CP is characterized by mild LE involvement and minimal to no UE involvement. The use of lefts to propel the bike increases LE strength. Hand brakes are better for safety since because this child would likely have better function of the UE>

A woman with a complete SCI at the C5 level has given birth to her first child. The client seeks suggestions on the methods to facilitate independent and safe parenting. Which of the following is most beneficial for the OT to recommend the mother use to help her independently feed her child? a. a pillow to support the mother's arms during breast feeding b. bottles with pre-measured formula c. bottles that have molded easy to grip shapes d. a sling to support the infant's head during breast feeding

a. a pillow to support the mother's arms during breast feeding the individual with a C5 SCI has sufficient UE function to be able to support the infant's head without the use of a sling. The individual would need a splint or other piece of AE to hold the baby's bottle.

discharge plans. An adult who incurred a severe traumatic injury (TBI) is entering his second week of care at a long-term TBI rehabilitation center. His family visits regularly and frequently ask multiple questions of the treatment team. A team and family conference is planned to address family concerns. The most important information for the team to share with the family is: a. realistic and clear info about the individual's current status and care plan b. each team member's expert opinion about the expected prognosis and discharge recommendations c. reimbursement info about each professional service to assist in determining treatment choices d. community resources that are available to support the family and provide respite care

a. a realistic and clear info about the individual's current status and care plan the family needs to understand the individual's current status and what is being done in the treatments to facilitate recovery. Since the individual has only been in the unit for 2 weeks, it is not possible for the team to know prognosis or

An older adult is referred to OT for intervention to develop independence in the performance of ADLs. During a dressing session, the OT notices a persistent area of redness over the sacrum that is thill evident after the patient has been upright for 30 minutes. The client reports that the area is itchy. Based on this observation, which of the following is most accurate for the therapist to document as needing attention in a revised intervention plan? a. a stage I pressure ulcer b. a stage II pressure ulcer c. a stage III pressure ulcer d. a stage IV pressure ulcer

a. a stage I pressure ulcer This stage is characterized by a defined area of persistent redness. Stage II is defined by a shiny or dry shallow, open ulcer. This could look like a blister. For stage III ulcers, bone, muscle, or tendons are not visible or palpable, but in stage IV they are.

An OTR is working with a 12-year-old child first diagnosed with emotional disturbance who has a history of being off task, poor social skills, and becoming overly upset about things when something does not go his way. He attends middle school and his teacher reports that he has difficulty following directions, completing schoolwork, and has frequent outbursts resulting in visits to the principal's office. What intervention or strategy is MOST effective for improving school behavior and performance? a. a token economy system to reinforce cooperation and task completion b. a therapy ball to sit on in the classroom so he can move while working c. movement breaks throughout the school day for self regulation d. referral to a social worker or school counselor for further intervention

a. a token economy system to reinforce cooperation and task completion A token enconomy method is most effective to help this child engage in nonpreferred activities. Tokens are earned through performance of tasks, engagement in activities, or demonstration of appropriate behavior. A therapy ball may be an effective strategy for self-regulation but it does not effectively address school behavior and performance. Movement breaks only really focus on self-regulation instead of actual school performance.

Following nerve injury repair surgery, an individual is evaluated for sensory return. Which measurement tool is best for the OT to use to asses the return of vibration? a. a tuning fork b. nylon filament c. a disk-criminator d. the ninhydrin test

a. a tuning fork Nylon filaments are used to asses cutaneous pressure thresholds. A disk-criminator assesses 2 point discrimination, and a ninhydrin test is a test of the ability to sweat.

An OT establishes a program for a new acute psychiatric unit at a community hospital. The therapist designs the physical layout of the OT department to include storage for arts and craft materials. Which of the following should the therapist recommend to store arts and crafts materials? a. a ventilated locked metal cabinet accessible only to staff b. open shelving accessible to patients c. shelving next to a sink for easy cleanup d. a locked closet outside of the intervention to ensure safety

a. a ventilated locked metal cabinet accessible only to staff these types of materials may be flammable such as paint, stains, and thinners. They need to be kept in a ventilated area. Since these supplies are dangerous and toxic, access must be controlled by staff.

an OT supervises a Level 2 fieldwork student regarding the evaluation procedures of a work hardening program. The therapist explains that some individuals attending the program magnify these symptoms to retain benefits; therefore, the validity of some evaluation measures may be compromised. Which assessment tool does the therapist identify as providing the most valid results? a. a volumeter b. a dynamometer c. a standardized pegboard test d. a total active motion evaluation

a. a volumeter this is the only true objective measure because it is based on the displacement law of physics. It is the only too listed that cannot be somehow manipulated.

an OT works with a child who has developmental delay and untillegible speech to develop the child's functional communication skills using a communication board. The child has attained 100% accuracy in pointing to "yes" and "no" on the communication board in response to questions. The OT determines that the child should be provided with the opportunity to expand communication skills. Which action should the therapist take to help attain the goal? a. add 2 more choices such as "play" and "snack" to the communication board b. reverse the positions of "yes" and "no" on the communication board to assure competence c. add the options of "play" "thirsty" "hungry" and "TV" to the communication board d. evaluate the child's ability to use a joystick to access a computure

a. add 2 more choices such as "play" and "snack" to the communication board The best choice for a child with developmental delay is maintain the consistency of the original selections and add one or two more options at a time. The best choices are to pick concrete items that the child prefers or enjoys.

A client receiving OT services in an inpatient rehabilitation unit demonstrates difficulty with feeding and is often spilling food, with food frequently falling off utensils. Which is the BEST action for the OTR to take in response to this client's activity of daily living (ADL) performance? a. address trunk control, specifically anterior weight shift b. address lateral weight shifts during basic ADL performance c. address lateral trunk extension during all eating and feeding tasks d. address symptoms of dysphagia and positioning during ADLs

a. address trunk control, specifically anterior weight shift In order to promote eating the client must be able to shift weight forward. Lateral weight shifting will not improve feeding. The client does not currently demonstrate symptoms of dysphagia.

The parents of a 18 month old bring their child to a free community developmental screening. The child can attend to shapes and use them appropriately. However, the parents are worried because the child cannot match shapes or manipulate different shaped objects into a shape sorter. Which is the best response for the OT to make in response to the parents' expressed concern? a. advise the parents that the child is showing a typical, age-appropriate skill b. complete an OT eval of the child's cognitive skills c. refer the child to the early intervention program for developmental delay d. provide the parents with activity recommendations to develop shape recognition

a. advise the parents that the child is showing a typical, age-appropriate skill The child is showing an age-appropriate skill. The ability to recognize shapes and manipulate different shaped objects into a shape sorter does not typically develop until the age 21-24 months. The ability to attend to the shape of things and use them appropriately is typical of children aged 18-21 months.

An individual has relocated to a new area and begins treatment at an outpatient OT clinic for follow-up after rotator cuff surgery. It is 6 weeks' postoperation. Which is the most effective intervention for the OT to implement at this time? a. an isometric strengthening program b. PROM c. AROM d. an isotonic strengthening program

a. an isometric strengthening program strengthening should begin with isometrics at 6 weeks and then progress to isotonics. PROM progressing to active/assisted rom is the intervention for 0-6 weeks postoperation

A client in the descending phase of the Guillian Barre syndrome has a bilateral shoulder strength of 2/5. The client fatigues easily. Which equipment should the OT recommend to enhance the person's performance of ADLs? a. an overhead suspension sling b. long-handled utensils and tools c. angled/curved-handled utensils and tools d. an environmental control unit

a. an overhead suspension sling This is suited for individuals presenting with proximal weakness with muscles grades in the 1-3/5 range. Long handled and curved utensils are useful for individuals who have significant motor deficits, proximally and distally, and who cannot independently perform tasks such as controlling the switches on electronic equipment.

in an outpatient rehab clinic, the OT is developing a fall prevention group for at risk elderly clients with PD. The clients live alone and have had at least 3 falls within the past 6 months. When presenting strategies to prevent falls, which is the most common risk factor for falls in the elderly that the therapist should review with the client? a. ascending and descending stairs b. dressing while seated in a chair c. walking with a walker with wheels d. transferring out of the shower with grab bars.

a. ascending and descending stairs

an OT reviews the positioning protocol for a premature infant with severe spastic CP with the infant's parents. The protocol is in written format. During the review, the therapist notices that the parents do not seem able to follow along with the protocol's text. Which action is the best for the therapist to take initially in response to this observation? a. ask the parents if they have any concerns about positioning their infant b. ask the parents if they can read and understand English c. include pictures of the proper positioning in the protocol d. demonstrate proper positioning techniques

a. ask the parents if they have any concerns about positioning their infant this is an open-ended question that enables the parents to express any concerns that they may have about positioning their infant. These concerns may be comprehension related and/or task-related. Caring for a child with severe physical disabilities can be overwhelming and the parents' perceived difficulties in following the written protocol may be due to emotional stress, not limitations in literacy.

a 6 y/o begins prosthetic training with a right below-elbow myolectric prosthesis. To learn to operate the terminal device, the OT implements intervention using age-appropriate play activities. Which activity is best for the therapist to first include during intervention? a. assembling building blocks b. squeezing a squeeze toy c. playing board games d. stacking 1 inch blocks

a. assembling building blocks in prosthetic training, the person first learns to open and close the terminal devise. Assembling the building blocks is the best opportunity to practice that. A squeeze toy is appropriate for a child up to 2 years of age. Stacking blocks is appropriate for a preschool child.

A carpenter recovering from injuries incurred during a fall from a ladder has decreased strength in the triceps, bilaterally. the most recent MMT indicated that the triceps muscle strength is 3. The OT provides the individual with a tabletop wood project to complete. To develop triceps muscle strength, how should the therapist position the tabletop when the person sands the project? a. at a 45* incline angled so that the individual's hands are above the elbows when the elbows are flexed b. at the individual's waist height so that the individual's hands and elbows are on the same plane when the elbows flex c. at a 45* incline angled so that the individual's hands are below the elbows when the elbows are flexed

a. at a 45* incline angled so that the individual's hands are above the elbows when the elbows are flexed This position requires the triceps to perform movement against gravity, which is possible at a muscle strength grade of 3.

an OT receives a referral to provide home-based services to an elder adult who lives alone in a 4th floor walk-up apartment. Upon entering the apartment, the therapist notes the sweltering heat. The apartment has no fans or air conditioners. The client's skin is hot, dry and red, and breathing is labored. The therapist offers the client a glass of water and places ice compresses on the arterial pressure points to help with cooling. Which is the most important action for the therapist to take next? a. cancel the intervention session and call for an ambulance to provide emergency medical services b. proceed with the planned intervention session and include documentation about the client's environmental conditions in the intervention report c. contact the home health agency's case manager to report the client's environmental conditions and then proceed with the planned intervention session d. cancel the intervention session and advise the client to speak to a doctor on how to best address the impact of hot weather on personal health

a. cancel the intervention session and call for an ambulance to provide emergency medical services the client is exhibiting signs of heat stroke. The elderly are particularly at risk for heat-induced illnesses. These extended periods of intense heat can be life threatening and must be treated as a medical emergency.

A single parent of 2 children, a 4 y/o and a 6 y/o, is referred to OT services in a rehab center. The client had a brain tumor removed 1 month ago. The referral states that the client has residual cognitive-perceptual deficits. Screening indicates that the client's sensori-motor abilities are WFL. During an intervention session using a transfer training approach, the OT gives the client a list of items commonly found in a closet and asks the client to separate the items into grooming and dressing items. Which skill is the OT working on? a. categorization b. sequencing c. problem solving d. memory

a. categorization sequencing would involve the planning, organization, and implementation of the steps of a task in the appropriate order. Memory is the registration, integration, recall, and retrieval of info.

An individual with a C3 SCI is participating in a community mobility group at a shopping mall. The client expresses the desire to return to the rehab center due to a pounding headache. The OT notices that the client is sweating profusely. Which is the best initial action for the therapist to take? a. check the client's urinary catheter and colleting bag b. call the rehabilitation center's transport department to relay the client's request c. escort the client outside of the mall to cool off in the fresh air d. immediately activate the recline feature of the patient's tilt-in-space wheelchair

a. check the client's urinary catheter and collecting bag Profuse sweating and headaches are signs of autonomic dysreflexia. This is an extreme rise in BP caused by noxious stimulus, which must be treated immediately by removing the stimulus.A blocked cath and overfilled urine bag are common complications that result in this medical emergency for people with SCI. The person should be placed in an upright position to help manage the rise in BP.

An OTR has a child whose family needs support with feeding. The child has a medical diagnosis of cerebral palsy and requires external supports during feeding times. The child is placed in a supported feeding chair to facilitate pelvic and trunk stabilization. The OTR is following the recommended guidelines for a safe swallowing pattern. What would be the BEST position for a safe swallow after the trunk is stabilized? a. chin tuck b. neck extension c. turn chin laterally to the right d. turn chin laterally to the left

a. chin tuck This keeps the head upright, in midline, with neck flexion, so the chin is directed slightly downward and inward. Turning the chin would not support a safe swallow.

An OTR is working with a 1-year-old with spastic quadriplegic cerebral palsy on the development of sensorimotor play skills. Which family-centered intervention is MOST EFFECTIVE to address this goal? a. coaching families to use switch-operated toys with child b. guide the child through movements in an obstacle course c. play computer or tablet activities with the child in the home d. teach caregivers massage to promote engagement in play

a. coaching families to use switch-operated toys with child guiding the child through an obstacle course is not the most effective because of the movement limitations of children with this condition. Computer or tablet activities promote engagement in play but are limited in the sensory realm.

an OT conducts an initial home visit to a family with a premature infant who, at 4 mo and 5 lbs, has just been sent home. The child has multiple disabilities. Which is the best primary goal for the therapist to work on with the family during this session. a. communicate effectively to develop a therapeutic relationship with the family b. teach the family proper body mechanics for lifting the child c. teach the family assertiveness training to develop advocacy skills determine whether adaptive aids or positioning equipment is needed

a. communicate effectively to develop a therapeutic relationship with the family.

an OT evals the feeding abilities of a patient who had a CVA 2 weeks ago. The therapist observes that the patient's dentures seem to slip when attempting to chew food. Which is the best action for the therapist to take in response to this situation? a. complete a referral to a dentist b. continue the eval using only soft foods c. develop an intervention plan to teach the individual compensation method d. request the nursing staff reapply denture adhesive prior to all feeding activities

a. complete a referral to a dentist dentures slip or move due to improper fit. Poorly fitting dentures must be evaled and corrected by a dentist. It is inappropriate to continue the eval or intervention prior to ensuring that the individual's dentures are properly fitted. Reapplying the adhesive does not address the underlying issue.

After a few months of OT services in the schools, Michael can finally flip his pencil to erase his misspelled words without dropping his pencil while writing, allowing him to finish his work in a timely manner. What hand manipulation skill pattern does this BEST represent? a. complex rotation b. simple rotation c. shifting d. palm-to-finger translation

a. complex rotation. This type of rotation involves rotating an object 180-360 degrees. Simple rotation involves turning an object held in fingertips for 90 degrees or less. Shift is a linear movement of an object on the finger surface.

a school-based OT is providing contracted per diem coverage for an OT on disability leave. Upon reviewing the caseload, the therapist notes that 10 students have had evals completed during the past month. 5 of the 10 student's individualized IEPs have also been completed by the team and approved by their families. Which is the best initial action for the therapist as a contract practitioner? a. consult with the team and family to complete the IEPs for the remaining children b. conduct independent evals of each child c. implement the IEPs that have been established and approved d. report to the supervisor of the contract agency that the school has failed to comply with IEP guidelines

a. consult with the team and family to complete the IEPs for the remaining children IDEA mandates that IEPs must be written within 30 days of eval. This must be done as a team effort with the professionals providing their recommendations to the team and the child's family. There is nothing indicating need for further eval or reporting of the school. While it is important to implement IEPs, the first priority is to ensure that all students have an IEP within 30 days.

An OT receives a referral from a physician that outlines a specific course of treatment. Following the evaluation of the person, the therapist believes that a different course of treatment would be more beneficial. Which course of action is best for the therapist to take? a. contact the physician and discuss the alternative treatment b. provide prescribed treatment on a 3 to 4 wk trial c. combine the prescribed treatment with the therapist's preferred treatment d. provide the treatment the therapist believes would be more beneficial for the person

a. contact the physician to discuss the alternative treatment

A 4-year-old child with decreased trunk and head control attends a 2-day-a-week mom's day out day care group. The mom mentioned that during the time the child is there, the staff does not move the child from her wheelchair onto seating options. The preschool stated to mom that they are not comfortable with the seating options they have in the classroom and would like recommendations on possible options for the child. The child is most engaged during circle and calendar time, so the staff would like an option that would support sitting on the floor or at the same level with peers and provide support, as she tends to fall sideways. What would be the BEST option for the OTR to recommend in this situation? a. corner chair b. infant sitter c. cube chair d. Howda hug chair

a. corner chair Corner chairs provide stable seating options for a child with decreased neck, head, and trunk control. Infant sitters are designed for children of a smaller frame. A cube chair would be an option if the child did not need neck support. The howda hug chair provides proprioceptive feedback but not enough support.

An OTR is working with a family of a 3-year-old child with autism spectrum disorder who has ongoing sleeping problems. The child has difficulty settling down to fall asleep and stay asleep and occasionally is seen night walking. What intervention is MOST EFFECTIVE for the family and child to promote sleep? a. create a consistent bedtime routine b. check on the child when the child cries c. paint the bedroom walls bright yellow d. use light-weighted sheets on the child's bed

a. create a consistent bedtime routine Routines are a important for a child with ASD. The parent can create a consistent bedtime routine with activities that provide an ideal environment for sleeping. Yellow is not a soothing color, and light-weighted sheets are more alerting than calming in tactile sensation. Heavy or weighted blankets are more effective and provide deep pressure that can help calm children for sleep.

an entry level OT is hired to work on an acute psych unit. The OT supervisor orients the therapist to the hospital policies and procedures. Which are the most important policies and procedures for the supervisor to include in the initial orientation session? a. crisis intervention b. reimbursement c. employee benefits d. group program scheduling

a. crisis intervention crises can occur at any time in an acute facility. All employees must immediately learn these policies.

the manager of a large OT department is planning to expand the current inpatient cardiopulmonary rehab program to include outpatient services. Which information is most relevant for the manager to include in the proposal to the hospital's administrative board? a. data on the inpatient rehab program's outcome b. testimonials from patients regarding their satisfaction with the inpatient program c. statistics on physician referrals to the inpatient unit and the average length of stay d. literature on cardiopulmonary rehab across the continuum of care

a. data on the inpatient rehab program's outcomes The provision of data on the inpatients rehab program's outcomes will indicate that patients are discharge when they are able to carry out activities at a 3.5 MET level. Since many IADLs, work, and leisure activities are at a MET level greater than 3.5, there will be a documented need for a continuation of cardiopulmonary rehab services on an outpatient basis. A needs assessment that provides info indicative of an unmet program is the first component of a program proposal.

an OT evaluates a client using the Allen Cognitive Level screen. The client successfully completes 3 running stitches. Which step should the therapist take next? a. demonstrate the whip stitch and ask the client to imitate this and complete 3 stitches b. demonstrate the single cordovan stitch and ask the client to imitate this and complete 3 stitches c. demonstrate the double cordovan stitch and ask the client to imitate this and complete 3 stitches d. document that the person demonstrates a cognitive level consistent with level 3 of the cognitive disabilities model.

a. demonstrate the whip stitch and ask the client to imitate this and complete 3 stitches According to the Allen Cog Level screen, after completion of 3 running stitches, the person should move onto the next step which is the completion of 3 whip stitches. After that, the client completes the single cordovan. The double cordovan is not used in this screening.

the parents of a child referred to OT ask the OT to treat their child who does not have insurance and bill for the services in the name of their older who does have insurance. Which is the therapist's best response to this situation? a. deny the request b. report the parents to the insurance company c. refer the parents to the department supervisor d. ask the family to get a referral from a physician

a. deny the request This is an unethical request.

upon screening, an 8 month old child demonstrates a positive downward parachute reflex. Which action should the OT take next in response to this observed behavior? a. document that the child exhibits normal reflex development b. document that the child exhibits a developmental delay c. evaluate the protective extension downward reflex d. evaluate the standing tilting reflex

a. document that the child exhibits normal reflex development a downward parachute reflex is normal from 4 months and persists throughout one's life unless neurological damage occurs. It is also called the protective extension downward reflex.

a middle school-aged child with right UE amelia attends OT to learn how to dress independently. Which of the following is most beneficial for the OT to focus on during intervention? a. donning and doffing a variety of shirt types of personal preference b. donning and doffing only shirts that can be donned overhead c. donning and doffing shirts that button in the front d. donning and doffing shirts with Velcro tabs sewn on to replace buttons

a. donning and doffing a variety of shirt types of personal preference a child with amelia, or absence of 1 arm, can easily learn to use a diversity of dressing techniques. Velcro tabs are appropriate for someone with decreased fine motor skills and/or strength, but are not indicated in this situation.

A 7 y/o with spina bifida at the C7 level receives home-based OT services. Which ability is most relevant for the OT to focus on during treatment? a. dressing the lower body b. dressing the upper body c. brushing teeth d. playing tabletop games

a. dressing the lower body an individual with a SC lesion at the C7 level has difficulty dressing the LEs. At 7, a child is typically independent with dressing, therefore, an intervention to increase the ability to dress the lower body is age appropriate. The individual would likely be independent with all other listed activities.

An OTR working in an outpatient rehabilitation clinic receives a referral for a client with a high-level radial nerve laceration from a car accident. Upon evaluation, the OTR determines that the client would benefit from a splint. Which is the MOST effective splint for the OTR to create? a. dynamic extension splint b. long dorsal blocking splint c. strong velcro buddy splint d. multi-dynamic flexion splint

a. dynamic extension splint The radial nerve innervates the extensor-supinator muscles of the forearm. Injury here would result in inability to extend the hand. A long dorsal blocking splint limits full extension of the hand and is used to immobilize the joints of the hand to promote healing. A buddy splint is used to protect a finger after a fracture by splinting it to another finger. Dynamic flexion splints are used with median nerve injuries to promote flexion of the hand.

aan OT working in a private clinic receives a referral for a client who incurred a nerve laceration while working as a cable installer and repair person. Upon eval, the therapist determines that the client exhibits maximum motor and sensory losses consistent with radial nerve laceration below the supinator the OT constructs a splint to facilitate healing and promote function. WHich is the most effective splint for the therapist to fabricate for this individual? a. a dynamic extension splint b. a figure 8 splint c. a dynamic flexion splint d. a splint to support functional positions

a. dynamic extension splint a radial nerve injury below the supinator is classified as a low-level radial nerve injury. A dynamic splint will provide wrist, MP, and thumb extension which is important in order to prevent over-stretching of the extensor tendons during healing. This splint also positions the hand for functional use.

An individual receives treatment for major depression on an inpatient psych unit. The patient has received an electroconvulsive treatment at 8 am. At 2 pm, the patient walks into the OT department stating a desire to participate in the leisure skills group. What is the OTs best response? a. encourage client to select 1 of 3 structured leisure activities to complete b. call nursing staff to escort the client back to his room so he can rest c. commend the client's motivation but remind the client that rest is recommended for 24 hrs after ECT d. have the client complete a leisure history questionnaire to enable individualized treatments

a. encourage client to select 1 of 3 structured leisure activities to complete 6 hrs after ECT, the individual is capable of engaging in a structured task. Giving the individual a choice increases likelihood that he or she will be interested in the task selected.

An adolescent with a 3-year history of anorexia nervosa is being discharged home in 2 days. The OTR will conduct a family meeting prior to discharge in order to provide essential information to assist the client in transitioning to home and in recovery. Which is the MOST relevant information for the OTR to provide? a. encourage family to select and prepare food initially b. encourage independent meal planning, election, and preparation c. blame the client for lack of healthy eating habits and choices d. suggest healthy eating options and identify maladaptive behavior

a. encourage family to select and prepare food initially Initial treatment necessitates the responsibility for selecting and preparing the food be shifted away from the client with gradual transition of responsibility back to the client over time. Initially, the client has difficulty re-engaging with food, so meal planning and selection will require assistance. Suggesting healthy eating options is helpful but the underlying mechanism of the disorder must be understood to be able to identify the maladaptive behviors.

An OT leads a social skills group for children aged 10-12 with conduct disorders. One of the children complains that the group activity is stupid and boring. Which is the most effective response for the OT to provide in response to this complaint? a. encourage the child to complete the activity with the group b. allow the child to leave the group since uninterested c. allow the child to suggest a different group activity d. tell the child the complaint will be discussed at the next family meeting

a. encourage the child to complete the activity with the group Encouraging the child to complete the activity with the group provides the child with the opportunity to develop age-appropriate skills.

An OTR is working with a family who is receiving early intervention services for their 2-year-old who has difficulty with self-regulation. The occupational goal area the team is working on is sleep. What is the MOST APPROPRIATE intervention to use to improve the child's overall sleep quality. a. establishing a nighttime routine that reinforces quiet behavior b. diffusing lavender oil in the bedroom to enhance a calm state c. watching cartoons before bedtime routine d. leaving a light on in the room to ensure safety

a. establishing a nighttime routine that reinforces quiet behavior Diffusing lavender oil may be one small piece of a routine, but it is not enough to solve the issue. Screen time is not appropriate before bed time on self regulation because it can actually disrupt a childs sleep patterns. Leaving the light on can make it more difficult to sleep.

an OT completes an early intervention screening of an 8 mo old. The results indicate that the child is able to sit independently by propping forward on both arms. Which is the best action for the OT to complete next? a. evaluate the child's sensorimotor skills using a standardized eval b. inform the patents that the child exhibits typical behavior c. develop goals to improve sitting balance d. provide play activities to develop sitting balance

a. evaluate the child's sensorimotor skills using a standardized eval Sitting with arms propped forward is typical of a 5-6 mo old. At 8 mo, a child typically sits without support. Plus the question says that a screening was completed. Evaluation is the proper next step after a screening. A therapist cannot set goals or create interventions until a full eval has been completed.

a young adult with a 10 year history of serious and persistent mental illness is being discharged home in 2 days. The client collaborates with the care coordination team to plan discharge with the client's primary family members. The team consists of a psychiatrist, registered nurse, social worker, and OT. The team conducts a pre-discharge family meeting to provide family members with info to assist them in supporting the client's recovery. Which is the most relevant information for the OT to provide to the client's primary family members at this meeting? a. family role activity suggestions for potential adaptions b. the therapeutic effects and potential side effects of medications c. advocacy strategies and consumer/family resources d. family dynamic information and family support groups.

a. family role activity suggestions for potential adaptations The OT is the only one in the care team who is qualified to provide info about role activities and potential activity adaptations. The other choices are all relevant, but the other members of the care team can provide this info.

An elementary school-aged child is referred to OT by a pediatrician. The pediatrician has ruled out ADHD and notes that the child demonstrates poor school performance and unexplained clumsiness. The OT screens the child and determines that the child is a candidate for a sensory processing eval. Which skills are most likely to have been observed to be intact during the screening and needing no further evaluation? a. fine motor coordination b. sequencing motor tasks c. initiation of activities b. ocular pursuits

a. fine motor coordination Poor fine motor coordination may occur along with a sensory processing problem, but it is not one of the criteria for a sensory processing disorder.

An OT provides intervention to develop independent feeding skills in an 18 month old child with significant developmental delays. The child has mastered the ability to hold a spoon and bang it on the tray of the high chair. The child can also hold and such on a cracker. Which activity is the best for the therapist to provide next? a. finger-feeding soft foods b. scooping food and bringing it to the mouth c. taking cereal from a spoon held by the therapist d. bringing a filled spoon to the mouth

a. finger feeding soft foods the next developmental milestone after holding and banging a spoon is finger-feeding soft foods. Due to the child's developmental delay, an OT would work on the acquisition of feeding skills according to normal developmental milestones. Taking cereal from a spoon occurs at 5-7 months, self-feeding by sucking a cracker 6-9 months, holding and banging a spoon scooping food and bringing it to the mouth 15-18 months.

a 2 y/o child receives home care early intervention services. The OT intervention plan includes a goal to develop the child's pincer grasp. Which is the most appropriate activity for the OT to work on with the child during an intervention session? a. finger-feeding of O-shaped cereal b. picking up marbles c. drawing with jumbo crayons d. stacking 1 in. cubes

a. finger-feeding of O-shaped cereal Picking up marbles will also facilitate use of a pincer grasp, but it is a potential choking hazard. A jumbo crayon uses a gross grasp, and stacking cubes uses a radial digital grasp.

An OT conducts an in-service at an outpatient wheelchair clinic for individuals with CNS dysfunction. According to the principles of wheelchair prescription, which of the following statements is accurate for the therapist to make during this presentation? a. firm seats are needed to provide stability b. soft seats are needed to prevent decubiti c. back heights should be extended to facilitate weight shifting d. seat angles should be 45* to prevent falling forward

a. firm seats are needed to provide stability firm seats provide stability and a solid base for seating systems that can be used to prevent decubiti, contractures, and deformities, and to increase sitting tolerance, proper positioning and functional abilities. Soft seats are contraindicated because they do not provide sufficient pressure relief.

A child with moderate Klumpke's paralysis on the right side wants to cut meat. The child holds the fork with the right hand and the knife in the left hand, and practices with therapy putty. To begin cutting which is the best way to configure the therapy putty? a. flattened and placed directly on the table b. rolled and placed on a plate on the table c. flattened and placed on a plate that rests on a non-skid mat d. flattened and placed on a plate on the table

a. flattened and placed directly on the table Klumpke's palsy is a brachial plexus injury affecting the C8 and T1. Placing the therapy putty on the table is the first step in learning to cut meat. Here the putty is stabilized and the child practices to refine technique. The second gradation in the activity is to put the putty on a plate on a non-skid surface

A client with a right below-elbow amputation begins prosthetic training with a body-powered myoelectric prosthesis. The OT collaborates with the client to begin training in the use of the terminal device to grasp and release objects. Which elbow position is best for the therapist to place the client's elbow in during grasp and release activities? a. flexed at 90* with neutral degrees of pronation/supination b. flexed at 90* and pronated 60* c. flexed at 120* and with neutral degrees of pronation/supination d. flexed at 90* and supinated 60*

a. flexed at 90* with neutral degrees of pronation/supination this is the easiest position in which to begin grasp and release activities. It is also the most functional and natural.

a 19 y/o with a diagnoses of dysthymic disorder and narcissistic personality disorder attends a vocational rehab program. When the client arrives for the work adjustment group, the OT notes that the client demonstrates and unsteady gait and slurred speech. The client's breath smells of alcohol. which is the best action for the OT to take in response? a. follow program procedures to arrange for transportation to bring the client home b. introduce the topic of alcohols effect on work performance as the focus of the scheduled group session c. have the client meet with the social worker to discuss treatment options for potential alcohol abuse. d. contact the client's parents to pick up the client and take him or her home.

a. follow program procedures to arrange for transportation to bring the client home

A single parent of 2 children, a 4 y/o and a 6 y/o, is referred to OT services in a rehab center. The client had a brain tumor removed 1 month ago. The referral states that the client has residual cognitive-perceptual deficits. Screening indicates that the client's sensori-motor abilities are WFL. The OT uses a task-oriented approach to remediate the client's cognitive dysfunction. Which is the most appropriate for the OT to include in the intervention program? a. functional activities in their real contexts b. client education on strategies to remediate deficits c. tabletop activities to practice remediation strategies d. computer games to develop performance component skills

a. functional activities in their real contexts the other options reflect a transfer of training approach.

an OT provides caregiver training to the spouse of an individual with cerebellar cortical degeneration. The focus of the session is on community mobility using a wheelchair. The individual is dependent upon the spouse's assistance for mobility. Which of the following is most effective for the therapist to recommend the spouse do when descending a steep grade? a. go down backwards with all wheelchair wheels maintaining contact with the ground surface b. tilt the wheelchair backward to its gravitational balance point and then go forward. c. tilt the wheelchair backward to its gravitational balance point and then go backward d. push forward as on flat surfaces but lean body back for extra drag

a. go down backwards with all wheelchair wheels maintaining contact with the ground surface This method enables the spouse to use body weight to slow the chair's momentum. If the spouse tires, he/she can readily stop and use his/her body weight to hold the chair in place while putting the breaks on. Maintaining the chair in a tilt can be more taxing and unnecessary.

An OTA provides community mobility training for a resident in a group home for individuals with developmental disabilities. A resident successfully completes the intervention activity the OTA had designed with the OT supervisor. The OTA needs to plan the next day's intervention session but the supervising OT is on a 2 week vacation. Which is the best action for the OTA to take in response to this situation? a. grade the activity that the client successfully completed to its next level of difficulty b. use the same activity that the resident successfully completed during the next session c. delay the next treatment session until the supervisor returns from vacation and is able to provide guidance d. ask the director of the residential program to assign another supervisor for the duration of the supervisor's vacation

a. grade the activity that the client successfully completed to its next level of difficulty OTAs are trained in activity gradation and implementation of OT intervention. There is no need to delay the treatment session or have a new supervisor for the time being.

A person is 5 days s/p CABG. The patient expresses anxiety about performing any type of activity and reports chest pain during ambulation. The cardiologist has approved activities at a 2-3 MET level. Which activity is best for the OT to use when initiating intervention with this person? a. grooming while standing at the sink b. grooming while sitting c. sponge bathing while sitting d. performing light housework

a. grooming while standing at the sink activities in sitting are 1-2 METs and housework is 3-4 METS.

An individual who is scheduled for a right total hip arthroplasty is referred to OT. A posterolateral surgical approach will be used. The client has expressive aphasia resulting from a CVA incurred 4 years ago. Medicare is the client's only health care benefit. The client states that private payment for any health care services or equipment is not possible at this time. how can the OT most reliably determine that hip precautions will be effectively implemented post-discharge? a. have the patient demonstrate the techniques that have been taught b. ask the family to observe the patient once home c. ask the patient to point to pictures of the hip precautions used during activities d. have the patient demonstrate positions that should be avoided.

a. have the patient demo the techniques that have been taught expressive aphasia interferes with the person's ability to verbally express his/herself. Asking the family to observe may be helpful for carryover, however, this source can be unreliable.

several patients in a cardiovascular unit are referred to OT for rehab in areas of occupation. Which diagnosis would be an inclusive criterion for participation in the home management activity group conducted in the department's stimulated apartment? a. hypotension b. unstable angina c. venous thrombosis d. uncontrolled atrial arrhythmia

a. hypotension persons with hypotension can be included in a rehab group. Engagement in these activities would be contraindicated for the patients with the other conditions.

a patient with a diagnosis of borderline personality disorder attends a stress management group on an inpatient psych unit. During the group, the patient's roommate states that there is a pocket knife in the patient's backpack. The patient says the roommate is exaggerating and that the item is only a keychain. Which should the OT do in response to these statements? a. immediately inform the charge nurse b. immediately check the patient's backpack c. check the patient's backpack after the group session d. meet with the patient's individual therapist after the group session

a. immediately inform the charge nurse the possibility that there is any item on an inpatient psych unit that could be used by a person to harm him or herself presents danger to all; therefore the charge nurse must be immediately notified. Searching the backpack may or may not produce the item and its not the OT's role to conduct a room or person search.

A OTR is planning treatment for adolescents diagnosed with eating disorders. Which goal is MOST IMPORTANT to address INITIALLY? a. increasing self-awareness through expressive activities b. increasing self awareness of nutritional issues c. improving school performance skills d. making recommendations or referrals for family therapist

a. increasing self-awareness through expressive activities This intervention focuses on the feelings behind the eating disorder. Once these feelings are revealed, alternative ways of coping can be addressed. Increasing awareness of nutritional needs is important but we need to understand the underlying issues. Making recommendations for family therapy will likely happen later in treatment.

An OT working for a home care agency is vacationing and sees a colleague at an all-day concert. Upon returning from the vacation, the therapist notices that his colleague had billed for a full-day of home visits on the day of the concerts. Which action is best for that therapist to take first in response to this situation a. inform the home care supervisor b. contact the state regulatory board c. contact the NBCOT d. ask the colleague to clarify the situation

a. inform the home care supervisor Speaking to a colleague to clarify a situation proper to reporting it to a supervisor can often be an appropriate initial step. However, in this case, the person has committed potential fraud which is very serious and must be brought immediately to the attention of a supervisor. The supervisor can then investigate and decide how to proceed.

After 6 months of rehab for a T2 SCI, a patient is being discharged. The OT conducts a home visit to evaluate accessibility. The individual lives with 2 roommates in an apartment in a private home. The doorway measurements currently range from 30-32 inches throughout the apartment. The patient's landlord is amenable to making changes in the apartment but has no financial resources. Which recommendation is best for the therapist to make for independent accessibility in the apartment? a. install offset hinges on all doors b. remove doorframes of doorways less than 32 inches and install wider frames c. remove all doors except the apartment entrance door d. remove all door frames ad install 36 inch wide door frames.

a. install offset hinges on all doors these can increase a doorway's width by 2 inches which would result in doorways meeting or exceeding minimum accessibility standards. The other options are costly or unnecessary. Removal of all doors eliminates privacy which may not be desirable when living with roommates.

an OT uses behavior modification techniques to help shape the behavioral responses of students with behavioral disorders. Which action is most consistent with this approach? a. provide frequent positive reinforcement for all desired behaviors b. reprimand the students every time an undesirable behavior occurs c. allow each student enough time for self-correction d. encourage the teaching staff to tell the student which behaviors are correct or incorrect

a. provide frequent positive reinforcement for all desired behaviors with this technique, negative behaviors should be ignored. Self-correction is not a form of behavior modification

an individual with a SCI at C7 reports noticeable redness on the ischial tuberosity during self exam with a mirror. Which action is most effective for the OT to recommend in response to these observations? a. integrate weight shifting into daily activities b. use a tilt-in-space wheelchair c. use an angled foam cushion d. self-direct caregivers to assist with weight shifting at least every 20 minutes

a. integrate weight shifting into daily activities during rehab, a person with an SCI must be instructed on the need for pressure relief. At this level of injury the individual can perform depression transfers so the ability to perform weight shifting is in tact. Other options are too low-level for this client.

an individual recovering from hepatitis type C has decreased UE and LE strength and hypertension. 6 months ago the client had an angioplasty and is very fearful of having a heart attack. Which should the OT instruct the client to perform to increase muscle strength? a. isotonic exercises b. isometric exercises c. contract-relax exercises d. muscle contractions and hold

a. isotonic exercises these are the only exercises listed that are not contraindicated for a person with hypertension or heart disease. The other choices describe isometric exercises or activities that are contraindicated in this case.

An OT working in a SNF conducts an in-service on validation therapy for the recently hired staff of a new psychogeriatric unit. Which fundamental principle of validation therapy is important for the therapist to include in this presentation? a. listen to the words an individual uses to ascertain the person's underlying message b. provide highly structured activities to refocus the individual on reality c. provide unstructured activities to facilitate the expression of feelings d. listen to the words an individual uses and provide reality orientation for individual statements

a. listen to the words an individual uses to ascertain the person's underlying message validation therapy is an approach to working with individuals with dementia founded on the principle that the unspoken messages an individual conveys with his/her speech are more important than the actual content of the speech. Individuals with dementia often make statements that are not based in reality. Example: A patient introduces her daughter as her mother. The fact is that the patients daughter is not her mother, but the underlying message is that the relationship is valued and worthy of comment. The focus of this type of therapy is to facilitate communication with persons with dementia in a caring, respectful, and empathetic manner.

A patient diagnosed with depression and anorexia nervosa attends an evening support group for 90 minutes each week. He states that this group is the only activity he engages in outside of work. The OTR collaborates with the individual to develop a plan to increase his involvement in avocational activities. He expresses interest in exercise and volunteerism. In high school, the client was captain of the swim team, played tennis, and worked in an after-school program for young children. What is the MOST BENEFICIAL interest for the client to explore? a. local elementary school for volunteer opportunities b. local fitness center for yoga classes c. area soup kitchen for volunteer opportunities d. town swimming pool for open swimming sessions

a. local elementary school for volunteer opportunities This addresses the person's interest in volunteerism and since he has had experience with after-school programs for youth, a school setting is appropriate. While yoga is exercise, it is not the type the client engaged in previously. People with anervosia often do not want to be around food so a soup kitchen is not appropriate. People with this disorder also tend to obsess about exercise so this choice could be more detrimental.

During a chart review, an OTR notes the client has experienced extensor tendon injury in zone I of the hand. Which BEST describes what the client is experiencing? a. mallet deformity b. clawhand deformity c. complex regional pain syndrome d. finger contractures

a. mallet deformity This is a result of a disruption of the terminal extensor tendon which manifests as a DIP lag. Clawhand is the result of high level ulnar nerve involvement resulting in hyperextension of the MCP joints.

An OT designs a dining rehabilitation program in a long-term care facility. The OT instructs paraprofessional staff in feeding techniques. Which point is the most important for the therapist to include in this staff training? a. meals should occur in a homelike environment with staff conversing with the elders being fed b. individuals with swallowing difficulties should be fed in a group so that staff can remind them to swallow at the beginning of each meal c. placing 3 fingertips on the throat and pressing firmly will stimulate a swallow response d. the head should be tilted slightly backwards during feeding to facilitate an assisted swallow

a. meals should occur in a homelike environment with staff conversing with the elders being fed proper feeding techniques include a facilitative environment. Placing 3 fingertips on the throat does not stimulate a swallow response. Tilting the head back during feeding may facilitate aspiration and is contraindicated.

an individual with advanced Huntington's chorea is newly admitted to a SNF. The resident weights 280 pounds and cannot independently transfer. What is the best recommendation for the OT to make to the resident's direct care staff to ensure a safe transfer? a. a mechanical lift transfer b. a two-person lift transfer c. a stand pivot transfer d. an assisted sliding board transfer

a. mechanical lift transfer This is the safest for both resident and staff. The other transfers require motor and cognitive abilities that are beyond the capacity of an individual with advanced Huntington's chorea

Child with juvenile arthritis wears bilateral night resting splints with wrists in 0* of extension, MPs and IPs flexed, ulnar deviation of 10*, and thumbs in opposition. The child complains of pain in the wrists upon awakening. No redness ins noted upon removing the splints. ROM measurements show ulnar deviation of 5*. Which action should the OT take in response to this complaint and these observations? a. modify the splints at the wrist b. pad the ulnar aspect of the inside of the splints c. discontinue the splints and monitor the status of pain for two weeks. d. construct a volar cock-up splint for use during the day.

a. modify the splints at the wrist.

an OT evals an individual with a partial tear of the supraspinatus muscle. The therapist documents the results of muscle testing. Which is most accurate for the therapist to state when reporting these results? a. strength is fair and the person reports pain b. strength is fair and the person reports no pain c. strength is good and the person reports pain d. strength is good and the person reports no pain

a. strength is fair and the person reports pain a partial tear results in weakness. It is painful because it is only partially torn. A complete tendon rupture results in muscle weakness that is painless. Tendonitis would result in pain but the muscle would remain strong .

An OTR is planning a group for adolescent children with fetal alcohol spectrum disorder. They are experiencing social withdrawal, teasing and bullying at school, and difficulty keeping peer relationships. What interventions are MOST BENEFICIAL to use with this group of children with fetal alcohol spectrum disorder to learn? a. negotiate with friends and reduce interfering behaviors b. create a social story on good manners and giving compliments c. participate and cooperate in a group project d. verbalize understanding of intentions of others

a. negotiate with friends and reduce interfering behaviors Adolescents with this diagnosis benefit from interventions that improve executive function, theory, and problem-solving. Creating social stories is an intervention geared more towards people with ASD. Cooperating on a project addresses social rules. Verbalizing intentions addresses having awareness of others not executive functions.

You are working with a resident of an assisted living facility. She enjoys the afternoon card and board games provided by the assisted living facility at the recreational hall. However, the client begins to complain of bilateral hand pain from moderate to severe rheumatoid arthritis. As a result, she has been unable to participate in the leisure activity she really enjoys. The rheumatologist referred her to occupational therapy to address the progression of her arthritis. What is the MOST appropriate intervention that an entry-level OTR can provide for the client? a. observe the card and board games the client enjoys and carefully grade or modify the activity b. use PAMs to help alleviate the pain and maintain ROM c. Provide the client with a prefabricated resting hand splint to provide positioning comfort d. Provide the client with resistive exercises to strengthen the intrinsic muscles of the hand and improve grip

a. observe the card and board games the client enjoys and carefully grade or modify the activity RA is a progressive joint disease that is painful. While PAMS help relieve pain, some are contraindicated for RA. furthermore, they are not entry-level practices. Splinting is used for clients with RA, but must consider adherence, fatigue, sensory impairment, skin integrity. Strengthening exercises are not recommended during acute stage of inflammation.

An OT works in an adult home. A resident has just returned from the hospital and is receiving 400mg of Thorazine to control symptoms. He has ben attending OT for a gardening group, physical exercise group, and a current events group prior to his hospitalization. He would like to resume participation in these groups. The OT advises him that prior to his group participation, a potential side effect he should be aware of, and take precautions for, is: a. photosensitivity b. akathisia c. akinesia d. tardive dyskinesia

a. photosensitivity anti-psychotic meds can result in all of the side effects listed. However, photosensitivity would be of the greatest concern for this individual given that the stated interest is resuming participation in a gardening group.

The parents of an infant born at 32 weeks gestation are about to take the baby home after 4 weeks in the NICU. Which is the most important for the OT to instruct the parents avoid? a. placing the infant in prone for sleeping b. placing the infant in supine for sleeping c. using an infant swing with a head support for calming d. presenting toys in the mid-line with the infant in the prone position for playing

a. placing the infant in prone for sleeping the infant should be encouraged to sleep in supine to prevent SIDS.

An individual is recovering from deep partial thickness burns on the UE, chest, and lower neck. The OT provides equipment to prevent positions that can result in contractures. Which are the most important positions for the therapist to prevent? a. positions of comfort b. anti-deformity positions c. positions resulting in edema d. positions of pain

a. positions of comfort this position occurs when a person assumes protective postures of adduction and flexion of the UEs, flexion of the hips and knees, and plantar flexion of the ankles. Preventing edema is important in burn rehab, but the question is about preventing contractures.

An OT is instructing a patient who had a left CVA on how to lock the brakes on a wheelchair. The patient is right-handed and the right UE has partial paralysis. Based on motor learning theory, which is the best intervention approach for the therapist to use when instructing the person to lock the right brake? a. practice with the left hand first and then with the right b. show the person how to use an extension brake c. practice the locking motions with both hands simultaneous d. have the patient use the left hand to assist the right hand

a. practice with the left hand first and then with the right Approaches based on the motor learning theory acknowledge the importance of practice and repetition. An extension brake is a compensatory strategy, so is using the left hand to assist the right hand.

An individual is recovering from lumbar surgery. The patient must remain flat in bed during the initial recovery stages. The patient expresses an interest in reading from a personal collection of classic comics. Which adaptation is best for the OT to recommend? a. prism glasses b. a page magnifier c. audiotapes of books of interests d. large-print books of interest

a. prism glasses prism glasses bend the light by 90*. This enables a person lying on his/her back to read anything resting on his/her lap. The use of audiotaped books does not meet the client's expressed interest of reading comics, and the other options do not address the issue.

an OT completes a risk management report for the reaccreditation of a rehab facility. In this report, which info is most important for the therapist to include? a. procedures for informed consent b. revenue resulting from reimbursement claims c. medical necessity of resources used d. the appropriateness of services provided

a. procedures for informed consent risk management is a process that identifies,evaluates, and takes corrective action against risk. A vital component of preventing risk and doing no harm is the acquisition of informed consent. A description of the appropriateness of services provided is a component of peer review and retrospective review.

An individual with schizophrenia continues to experience hallucinations. Which action should the OT take when the individual begins to actively hallucinate during an OT project group? a. redirect the individual's attention back to the project b. provide tactile reassurance to the individual c. verbally reassure the individual that the hallucination is not real d. use humor to divert the individual's attention away from the hallucination

a. redirect the individual's attention back to the project The best action to take when a person is experiencing a hallucination is to direct their attention back to the project using a calm, matter-of-fact tone. This can help the person focus on reality. Explaining that the hallucinations are not real is ineffective because to the individual, the experiences are real. Uninvited touch can be threatening during hallucinations.

An OTR is working with a 17-year-old adolescent girl with high-functioning autism spectrum disorder. The adolescent is planning on enrolling into a 4-year college after graduation. She does well academically except when her routines are disrupted. She loses track of time during classes, startles often by the school bell which leaves her disorganized, and struggles to transition between classes. What is the MOST appropriate intervention to use to promote her independence in transitions between classes? a. program an app providing an alert structure on her cell phone to enable her to move between classes in an orderly timely manner b. use a visual schedule to keep on track and go from task to task during the college day c. enlist an educational assistant or class buddy to aid her in the training process d. carry a detailed calendar outlining the day with a checklist of classes to check off after attending each class

a. program an app providing an alert structure on her cell phone to enable her to move between classes in an orderly timely manner This idea uses a cell phone which adolescents generally have with them at all times. It promotes more independence than calendars or visual checklists. A college schedule changes more than a high school schedule so being able to change the app alarms provides more flexibility.

An individual attends a community day treatment program to assist in recovery from major depression. The client has fair eye contact and responds verbally to interactions initiated by others. The person's cognition is intact. Which group level is best for the OT to recommend this client attend? a. project b. parallel c. cooperative d. mature

a. project a project group utilizes short term activities that require participation of 2 or more people. Tasks are shared and the focus is on interaction rather than task completion. This level is appropriate for someone who is socially responsive to others with intact cognition. A parallel group doesn't require interaction for task completion which is too low-level for this individual. Cooperative and mature groups require members to be self-expressive and meet socio-economic roles. These groups are too high level for this individual at this point.

an adult incurred an injury to the anterior spinal artery at the T12 level. The OT completes a sensory eval with this client. Which sensation is most likely for the therapist to document that the client has retained? a. proprioceptive b. pain c. crude touch d. temperature

a. proprioception proprioception is maintained with anterior cord syndrome, which is caused by damage to the anterior spinal artery or cord.

An individual has had a brain tumor removed from the cerebellum. The OT conducts a screening to determine the need for further eval. During the screening, the OT observed deficits indicative of the need for further eval. Which would be most relevant for the OT to eval based on this screening. a. proprioception and coordination b. tactile and sensory integration c. vision and visual-perception d. audition and communication

a. proprioception and coordination the cerebellum receives input from the proprioceptive pathways and modulates the smooth coordination of voluntary movements. Tactile and sensory input would be most affected by damage to the parietal lobe. Audition and communication would be most affected by damage to the temporal lobe.

An OTR is working with a school-aged child with autism spectrum disorder in his second-grade classroom. The teacher reports that the child is continually off task, not finishing his assignments in a timely manner, and roams the classroom frequently not knowing what task to do next. What intervention is MOST BENEFICIAL to use to improve behavior and task completion in this child with autism spectrum disorder? a. provide choices and visual schedules to aid in knowing classroom expectations b. provide movement breaks in the classroom during the off-task behavior times c. alter the lighting, noise, and smell in the classroom d. discuss ways to use objects based on their properties

a. provide choices and visual schedules to aid in knowing classroom expectations Schedules provide structure which fosters independence with understanding a classroom routine. Providing movement breaks is most beneficial to improve motor skills and may or may not improve attention span. Altering lighting, noise, and smells, is a way to reduce fear and anxiety but is not best for attention span.

During an OT group, a person receiving electroconvulsive therapy complains about short-term memory loss. Which should the therapist do in response to the client's stated concerns? a. provide cues during activities to compensate for memory loss b. immediately contact the psychiatrist to inform him of this symptom development c. reassure the person that short-term memory loss is a typical response to ECT d. tell the person to inform the psychiatrist of this symptom development

a. provide cues during activities to compensate for memory loss Short-term memory loss is typical after ECT, therefore, there is no need to inform the psychiatrist. However, reassuring the person that the loss is typical does not address the problem at hand. Providing cues can help the individual deal with this memory loss and allow effective engagement in meaningful activities.

a child with tactile defensiveness is receiving intervention from an OT using a sensory integrative approach. Which method is most effective for the therapist to use when introducing tactile stimuli to the child? a. provide deep touch and firm pressure where the child can see the stimuli b. apply the stimuli in the direction opposite of hair growth with the vision occluded c. apply light touch across the face and abdomen with vision occluded d. provide light brushing across the palmar surfaces of the extremities with the child watching.

a. provide deep touch and firm pressure where the child can see the stimuli this helps decrease tactile defensiveness, but the child needs to see the stimuli to do so. The other options are all averse to a person with tactile defensiveness. Stimuli should be applied in the direction of hair growth for this is less averse.

An OTR is working with an infant in the NICU who was born at 25 weeks' gestation. The infant is the first child of young parents without high school diplomas who live about 2 hours from the hospital. Both parents appear unsettled and afraid to touch and hold their child for fear they will hurt the infant. What INITAL action should the OTR take when providing developmental care to the infant and family? a. provide family education on kangaroo care and explain the benefit of this care for relaxation and caregiver closeness b. consider using supportive measures such as containment to promote self-regulation and attachment c. explore ways of creating a home-like environment at the infant's bed space to provide family privacy and comfort d. provide written home program activities and training to the parents before discharge to ensure carry-over into the home

a. provide family education on kangaroo care and explain the benefit of this care for relaxation and caregiver closeness family education is essential for new parents. This will boost the caregiver closeness and ease their fears. Creating a home environment may comfort the parents but it doesn't address their fear.

An OTR is working with a child with schizophrenia and depression in the middle school setting. This child is receiving tier 3 intensive intervention for students with mental health challenges. Which intervention strategy BEST describes a tier 3 intervention that an occupational therapist would implement in the classroom? a. provide individual or group therapy and identify ways to modify or enhance school routines and reduce stress b. consult with the child's teacher about modifying learning demands and academic routines to support social-emotional skill development c. provide training school-wide to teachers and staff on sensory processing and OTs unique role in the promotion of mental health d. Informally observe all children for behaviors that might suggest mental health concerns and bring findings to the educational team

a. provide individual or group therapy and identify ways to modify or enhance school routines and reduce stress Consulting with the child's teacher is a tier 2 strategy and the other strategies are tier 1.

An OTR is working with a 5-year-old child with sensory processing disorder and autism spectrum disorder who has difficulty with social interactions, including play dates. He becomes upset and talks loudly, invades his friend's space, and may even hit the other child. What is the BEST approach to promote social participation INITIALLY for this child? a. provide sensory integration in each session to enhance the child's ability to filter sensations and recognize the optimal level of arousal b. encourage the child to make his or her own choices in social situations to motivate and produce feelings of confidence c. apply newly developed social skills in a small, less intimidating peer context then decrease therapist support d. use a goal chart as a visual reminder of positive behaviors to encourage and perform with more confidence

a. provide sensory integration in each session to enhance the child's ability to filter sensations and recognize the optimal level of arousal Children with ASD improve social communication and play after they receive sensory integration intervention. Once a child is regulated, there is more likelihood for a positive social behavior to be achieved. Encouraging the child to make his/her own behaviors increases self-determination rather than social participation. Applying new skills in small situations is a good approach to take AFTER sensory integration is applied and the child is regulated.

A resident of a SNF is severely dehydrated after a viral illness. The resident is agitated and confused. The doctor has prescribed IV fluids, but the nursing staff is concerned that the individual will pull out the infusion line. They request that the OT provide a restraint for this resident. Which is the best action for the therapist to take in response to this request? a. provide soft, fleeced mittens for the person's hands b. report the request to the administrator as a violation of restraint-free standards c. decline the referral and explain that restraints are no longer allowed to be used in SNFs d. provide a lap board as this is the least restrictive restraint

a. provide soft, fleeced mittens for the person's hands wearing mittens will help prevent the individual from pulling out he IV. Providing ones that are soft and fleeced can provide tactile input that is not noxious. Wile federal guidelines emphasize restraint reduction and the provision of a restraint-free environment, they also recognize the potential need to provide restraints in certain circumstances. A restraint is permissible and acceptable if it is medically necessary and temporary for lifesaving treatment. If the person begins to rub the IV line with his/her hands even with the mittens on, the therapist could recommend the use of bilateral elbow splints that fix the elbow at 20-30* of flexion. A lap tray would not limit the person's UE mobility and would not be effective.

A high school student with a diagnosis of borderline personality disorder and a history of self-abusive behaviors attends a transitional school-to-work program co-lead by an OT and an OTA. During vocational skills group, the student expresses feelings of hopelessness about the future and questions the point of participating in the group. The student also asks to leave the group due to being to tired to concentrate as a result of sleepless nights. The OT asks the OTA to assume leadership of the group while the therapist addresses the student's concerns. Which action is best for the therapist to take in response to the student's statements? a. pull the student aside from the group and ask if the student is feeling self-destructive b. allow the student to leave the group after reminding the student to relate concerns to the guidance counselor c. support the validity of the student's feelings and encourage the student to remain in the group d. remind the student that in a work setting the norm is to work even if fatigued.

a. pull the student aside from the group and ask if the student is feeling self-destructive statements of hopelessness and lack of future vision must be taken seriously, as they can indicate suicide risk. This is especially important because of his self-destructive behavior. If the student is allowed to leave the group, there is a risk of self-destructive behavior or suicide. Validating the student's feelings does not deal safely with a potential crisis.

an adult is referred to OT for ADL training. The patient incurred a fracture of the right proximal humerus and is using a shoulder immobilizer for the first two weeks to aid healing and help control pain. The patient is right hand dominant. Which activity will be the most difficult for the patient? a. putting on a pullover top b. completing online banking c. taking off a winter coat d. brushing teeth

a. putting on a pullover top treatment for this issue includes a sling with no shoulder mobility for the first two weeks. Because of front openings, jackets would not be as difficulty. Banking and brushing teeth can be done with one handed techniques.

A pre-school aged child with recurring headaches and decreased gross and fine motor skills is hospitalized on an acute care unit for a diagnostic work-up. Just prior to the OT eval, the parents are told that their child has cancer. The parents are upset when the bring their child to the evaluation session. Which are the best actions for the therapist to take in response to this situation? a. recommend the parents speak to their spiritual advisor or the social worker and proceed with the OT session b. cancel the OT session and recommend that the parents speak to their spiritual advisor or the social worker c. spend the OT session providing support to the parents and addressing the parent's acceptance of the diagnosis. d. recommend the parents speak to their spiritual advisor or the social worker and reschedule the OT session for later in the week.

a. recommend the parents speak to their spiritual advisor or the social worker and proceed with the OT session. the best actions are recommending the parents speak to a source of help and proceeding with the session. The child needs this eval as a part of the diagnostic work-up. The therapist can help the family adjust and accept the diagnosis via therapeutic activities during the session. However, counseling is most appropriately provided by the social worker or religious provider. Canceling or rescheduling is not necessary and in a hospital setting it is highly unlikely that immediate re-appointments would be available.

An individual with borderline personality disorder is admitted to the hospital following a suicide attempt. After attending an OT orientation group, the patient tells the OT, "You are the only therapist who has ever really been helpful." The patient asks to meet with the therapist privately on a regular basis instead of the assigned primary individual therapist. Which is the best action for the therapist to take in response to the patient's request? a. refer the patient to the assigned primary individual therapist b. agree to meet with the patient since a positive therapeutic connection has been expressed c. tell the patient that an OT provides only occupation-based group treatment d. explain that this type of manipulative behavior is not acceptable

a. refer the patient to the assigned primary individual

a teenager with Ducheene muscular dystrophy can no longer snap blue jeans or zip zippers. Which recommendation is best for the OT to make to this client? a. replace snaps and zippers with Velcro b. replace snaps and zippers with large buttons c. use a zipper pull to zip jeans and leave the snaps unsnapped d. purchase and wear elastic waist paints

a. replace snaps and zippers with Velcro MD is a progressive condition. The therapist must be able to assist the person in adjusting its progressive nature and provide options that maintain independence as long as possible. Velcro can be more easily managed than snaps or zippers. Elastic waist pants facilitate the process of donning pants but they do not address the identified difficulty of fastening jeans meaning jeans would no longer be accessible for the teen.

2 OTAs working for a school district are assigned to 2 different schools. 1 school has a supervising OT, the other school does not. The OTA without a supervising therapist expresses concern to the OTA that is supervised about the lack of supervision. Which is the best response for the supervised OTA to take in response to this situation? a. report the situation to the OT supervisor b. share information acquired during supervisory sessions with the unsupervised OTA c. advise the unsupervised OTA to contact the state regulatory board d. report the situation to the school's district's administration

a. report the situation to the OT supervisor the first step is for the OTA to discuss the situation with the supervisor since all of the concerned parties work in the same school district. The OT supervisor would then be the appropriate person to further explore the situation and find a supportive solution with his/her supervisor. While it can be beneficial to share info gained during supervisory sessions, this process is no substitute for supervision.

An individual is transferred from an acute care hospital to a sub-acute rehab unit in a long term care facility. The patient incurred a left CVA in the middle cerebral artery 1 week ago. The individual is referred to OT. The referral states that the patient has right hemiplegia and a subluxed right shoulder. The OT also advises the direct care staff on proper positioning of the patient's right arm while the patient Is seated in a wheelchair. Which is the most appropriate recommendation for the therapist to make for positioning of the patient's right arm? a. rest the person's arm on a wheelchair lap board b. rest the arm in the person's lap with hands folded c. wear a resting hand splint to avoid joint contractures d. have the person wear a shoulder sling throughout the day

a. rest the person's arm on a wheelchair lap board Proper positioning must avoid shoulder traction, scapular downward rotation, and weight on the shoulder. Lap boards help with this and prevent arms and hands from getting caught in the wheels of the wheelchair. Wearing a sling is contraindicated because it can result in contractures, edema, and pain syndromes.

The OTR is working with a pediatric patient with right-side weakness and dressing tasks. The non-affective upper extremity is able to assist with dressing, but he is still working on increasing strength and coordination. The child has been working on independently zipping and unzipping his jacket. Which adaptation would BEST facilitate success in this task? a. ring pull b. elastic shoelaces c. Velcro straps d. sock aid

a. ring pull Individuals with decreased fine motor control, strength, and in-hand manipulation benefit from this. Providing a larger item to grasp allows the client to grasp the zipper with more ease. Velcro, sock aids, and elastic shoelaces all work on independence donning/doffing shoes.

An OT implements intervention in a preschool program for children who are over-responsive to touch. In the prior intervention session, the children had responded favorably when the therapist had rolled a large ball over their bodies as they lay supine on a mat. Which intervention methodshould the OT use next? a. roll the ball with increased pressure over the children's bodies b. bounce the ball across the children's bodies c. have the children jump into a pool with small balls d. roll the large ball, as in the prior session, with the children prone

a. roll the ball with increased pressure over the children's bodies Increasing the pressure on the ball is the next gradation of the activity that has been reported to be successful. Children who are over-responsive to touch (tactile defensive) respond well to firm touch. Jumping into a pool of small balls is too large of a progression, and changing the position from supine to prone does not grade the activity related to the over-responsiveness to touch

A school system hires an OT to implement a SI program. The OT plans a staff in-service to explain the indications, contra-indications, and precautions for the use of a sensory integrative approach. The most important precaution for the therapist to review is: a. seizures b. self abusive behavior c. somatodyspraxia d. hyperresponsiveness to sensory stimuli

a. seizures individuals with seizures often have difficulty tolerating SI. especially brushing and vestibular input. These types of SI approaches can trigger seizures. Somatodyspraxia is a disorder in motor planning due to poor tactile perception and proprioception and it is an indication for SI intervention.

A OTR is working with a 10-year-old boy who will only eat mashed foods, does not like noises (e.g., he gets upset when the phone rings), and has difficulty sitting during meals. His parents report that he does not like new clothes and cannot wear jeans. What type of sensory processing problem should the OTR address INITIALLY when planning intervention activities? a. sensory modulation b. sensory discrimination c. vestibular-bilateral d. dyspraxia

a. sensory modulation sensory discrimination is not being able to distinguish between different textures.

An OTR is asked to evaluate a 7-year-old girl who is having difficulty in her first-grade classroom. Her teacher reports that she fidgets a lot in class and has difficulty paying attention. In addition, she tends to be aggressive with the other children, bumps into the furniture, and chews on her pencil. Which assessment tool should the OTR use INITIALLY? a. sensory profile b. school function assessment c. developmental test of visual perception d. Peabody developmental motor scales

a. sensory profile The sensory profile evaluates sensory processing, modulation, and the behavioral and emotional responses to sensory processing. The school function assessment looks at the quality and quantity of children's engagement in daily occupations. The Peabody is only appropriate for children up to 5 years of age.

an adolescent incurred a C4 SCI. During the initial session, the patient refuses to speak to the OT. The therapist supportively acknowledges the client's response. Which action should the therapist take next? a. set up a chin-operated, bedside environmental control unit to enable access to the environment b. provide passive ROM to prevent contractures c. Explain what OT can offer the adolescent to adjust to decreased abilities d. ask the adolescent to tell nursing staff when personally ready for OT

a. set up a chin-operated, bedside environmental control unit to enable access to the environment The individually immediately needs a method to access the environment. Being able to call staff, operate a TV, answer the phone, turn off/on the lights, and other functions are important basic tasks for the adolescent to be able control independently. It is not necessary to explain what OT can offer. Providing PROM ignores the client's feelings.

During an intervention session in a school, the OT observes a young child turn the pages of a book. The therapist identifies this behavior as an example of an in-hand manipulation task. Which task should the OT document the child is capable of performing? a. shift b. simple rotation c. translation d. translation without stabilization

a. shift turning the pages of a book involves a linear movement of each page on the finger surface. Simple rotation is not correct, as this involves a turning/rolling of objects held at the finger pads with the fingers acting as a unit and the thumb in opposition (unscrewing a bottle cap). Translation is palm to fingers or finger to palm movements.

am OT is treating an 8 month old child with mild DD. The child exhibits normal cognitive development. The child has developed adequate static sitting balance but has poor dynamic sitting balance. The therapist implements intervention by positioning the child and having the child find a toy that is covered with a cloth. Which positioning and toy placement are the most beneficial for the therapist to use with this child? a. sit the child between the therapist's extended legs and alternate placing the covered toy to the child's right and left side b. sit the child in a child seat and alternate placing the covered toy to the child's left and right sides c. lay the child in prone position and place the covered toy in front of the child d. lay the child on the right side and place the covered toy to the left of the child

a. sit the child between the therapist's extended legs and alternate placing the covered toy to the child's right and left side Having the child sit between the therapist's extended legs allows the therapist to easily provide postural support as needed. Placing the toy to the right and left will facilitate the child's sideward protective extension response. Sideward protective extension in sitting is a functional, protective reaction that typically occurs at 7 months and persists in normal development. It is a key component to the development of dynamic sitting balance as it protects the child from a fall.

a 2 y/o child is placed in foster care due to child abuse and neglect. The child as frequently beaten and locked in a dark closet. The child is fearful and suffering from sensory deprivation. which sensory input is best for the OT to recommend the foster parents provide for the child? a. slow rocking b. fast rocking c. bright lights d. upbeat music

a. slow rocking intervention for sensory deprivation should begin with slow linear movements.

A client with ataxia is receiving OT services to improve coordination and promote use of upper extremities during ADL tasks. Which of the following should the OTR recommend to promote engagement in ADLs? a. stabilize joints proximal to distal b. promote movements farther from body c. request assistance for difficult ADL tasks d. joint stabilization at the distal portion of the task

a. stabilize joints proximal to distal Stabilizing joints proximally reduces ataxia. completing movements closer to the body also reduces ataxia. Although requesting assistance is helpful, clients are encouraged to increase independence.

An OT has scheduled a discharge planning session with an individual recovering from hip replacement surgery. In preparation for this session, the therapist reviews the nursing reports and learns that the person has TB which is currently in the dormant phase. Which type of precautions should the therapist use when entering the patient's room? a. standard b. airborne c. droplet d. contact

a. standard standard precautions are observed in all clinical situations. The additional use of airborne precautions is required when working with persons known or suspected to be infected with a serious illness transmitted by airborne nuclei. When in the active phase, TB, measles, and chickenpox are transmitted in this manner. They are not needed when TB is dormant. Droplet precautions are used for rubella, mumps, pertussis, and influenza.

a restaurant employee incurred a fracture to the left humerus. After cast removal, the patient received OT and now demonstrates fair strength (3/5) of the left triceps and full ROM of the left elbow. To increase elbow function, in order to perform work-related tasks, which activity is most effective for the OT to next include during intervention? a. storing glasses on shelves at chest height b. wiping off a table while standing c. carrying a tray of dishes from the table to the sink d. wiping off a counter at chest height

a. storing glasses on shelves at chest height this is a lightly resistive activity of the triceps against gravity, one of the best ways to increase strength in a muscle to attain full elbow ROM. Wiping a table at either surface is an isotonic, gravity assisted activities, which are a downgrade in strengthening. Carrying a tray is a high resistance activity that is too difficult at this point.

An OTR is working with a group of 4 to 6-year-old children in an out-patient pediatric clinic to create a wall mural by tearing small pieces of tissue paper and pasting them to a large piece of paper. Which preparation activity is BEST to promote the hand skills needed to create the mural? a. stringing small beads b. pushing heavy objects c. playing finger games d. rolling clay into a ball

a. stringing small beads This requires a precision grasp, hand strength, and bimanual hand skills to participate in the task effectively. These are the same skills needed to make a tissue-paper mural. Pushing heavy objects increases strength and endurance. Finger games addresses tactile discrimination in the hand. Rolling clay is for regulation of pressure in the hand.

An individual recovering from a hip replacement surgery prepares for discharge home. The client has a secondary diagnosis of GERD. Which is the best bed position for the OT to recommend to this client? a. supine with elevation of the shoulders and head b. sidelying with the neck in neutral c. sidelying with elevation of the shoulders and head d. supine with the elevation of the hips

a. supine with elevation of the shoulders and head

an individual with peripheral neuropathies due to diabetes is scheduled for a bilateral LE amputation. During an OT session to develop UE strength to assist with post-amputation transfers, the patient happily chats about plans to go shopping for new clothing to wear to a grandchild's wedding. After the session, which defense mechanism should the OT document that the person seems to be exhibiting? a. suppression b. regression c. displacement d. projection

a. suppression suppression is a defense mechanism that allows an individual to divert uncomfortable feelings into socially acceptable feelings in order to avoid thinking about a disturbing issue. Displacement is the redirection of an emotion or reaction from one object to a similar but less threatening one (ex: child who is angry at parents but yells at siblings). Projection is the attribution of unacknowledged characteristics or thoughts to others (ex: someone who feels guilty interprets the statements of others as blaming him/her)

An OT who is a driver rehab specialist provides on-the-road eval for an individual recovering from a right CVA. The individual is right dominant and has regained sensorimotor functions in the affected extremity. Which is the most important for the OT to assess during on-the-road eval? a. tactical aspects of driving b. operational aspects of driving c. ergonomic aspects of driving d. social aspects of driving

a. tactical aspects of driving. These aspects of driving involve ability to respond to changes in road conditions, traffic, and driving risks. Therefore, intact cognitive skills are necessary. Operational driving aspects involve the ability to steer, break, and turn. Since the person in this scenario has intact dominant UE and LEs, there is no need to assess this.

an OT works with a patient who incurred a right CVA. The patient has homonymous hemianopsia. Which is the most effective compensatory strategy for the therapist to use initially with this patient? a. teach the patient to turn the head to the affected side b. provide printed notes on the left side telling the patient to look to the left c. place the patient's plate and eating utensils on the left side of the bed tray d. rearrange the patient's room so while the patient is in bed, the left side is facing the doorway

a. teach the patient to turn the head to the affected side Left homonymous hemianopsia results in an inability to receive information from the left side. Initially, the patient needs to be made aware of his or her deficit and instructed to compensate by turning the head to the affected side. Providing notes and putting utensils in the left side will not be helpful because they would be in the patients affected visual field.

An OT provides beside BADL training to a patient recovering from multiple injuries incurred during an MVA. The patient's children arrive for a visit and ask the therapist to let them look at their parent's chart while they wait outside the room for the session to conclude. Which response is the best for the therapist to make in response to this request? a. tell the family members that they must have the permission of their parent before they can look at the chart b. commend the family members for their interest in their parent's status and give them the chart to read c. tell the family members they cannot see the chart because they could misinterpret the info d. tell the family members to go ask the unit's charge nurse for permission to look at the chart

a. tell the family members that they must have the permission of their parent before they can look at the chart according to HIPPA, the therapist must obtain the person's permission prior to sharing any info about the person's status with family members or significant others.

A client participates in OT for intervention following a rotator cuff injury. The therapist provides progressive resistive exercises. When grading these exercises, which of the following is best for the therapist to increase? a. the amount of resistance provided with a stronger level of therapy band b. the ROM involved in completing the exercises c. the proximal load on the muscles the client uses during the exercises d. the repetitions of external rotation exercises with less distal weight

a. the amount of resistance provided with a stronger level of therapy band increasing the ROM of an exercise helps increase ROM of the muscle but not strength. Increased repetitions helps with endurance not necessarily strength.

A small toddler has been stuffing his mouth with solids when eating. The parents are worried that he will choke because he is over stuffing. He is reluctant to accept help when feeding and tends to excessively control the environment. The OTR has completed an evaluation and observation of the child during mealtimes. The OTR has determined the toddler may be experiencing issues related to feeding. What is the MOST likely issue the toddler may be experiencing? a. the child may be experiencing oral hyposensitivity b. the child may have decreased oral control and coordination c. the child may be experiencing oral hypersensitivity d. the child may have decreased tongue lateralization and oral movements

a. the child may be experiencing oral hyposensitivity children with oral hyposensitivity are better able to feel food in the mouth if it is stuffed into the mouth.

An OT provides home-based services to a child with developmental delays. The child picks up and puts away toys when reminded by the parents and mimics the parents when they dry dishes and fold clothes. The family has identified a goal of including the child in home management activities. Which activity should the therapist introduce next intervention? a. wiping tabletops b. sorting laundry c. making a bed d. taking out trash

a. wiping table tops picking up and putting away toys when reminded are typical of 2 y/o children. According to the typical developmental sequence of home management tasks, wiping spills is a task completed by 3 y/o children. Sorting laundry is for 4 y/o children and taking out trash is for 5 y/o children.

A 6-year-old girl is referred to school-based occupational therapy after her mother expressed concern that she is having difficulty using her hands to color, getting on the swing at the playground, and her teacher reported she is having difficulty with handwriting and reading at school. What is the MOST APPROPRIATE INITIAL short-term goal for improving handwriting skills at school for this child? a. the child will write first name on the upper right hand corner of the paper with 90% legibility b. the child will write a paragraph, without verbal prompts, in 10 minutes with 80% legibility c. the child will write one simple sentence in less than 5 minutes with 90% legibility d. the child will complete written assignments in a timely manner in the clasrrom with 80% legibility.

a. the child will write first name on the upper right hand corner of the paper with 90% legibility The most appropriate initial goal is to start with the easiest task to achieve within the writing sequence. Writing paragraphs is too complex, and writing a simple sentence is more complex than writing a first name. Completing written assignments is the most complex task.

An adult with ALS frequently coughs and chokes when eating finely chopped foods and drinking thin liquids. The SLP and OT collaborate and recommend a videofluoroscopy procedure. Which behavioral information would be most relevant for the OT to include in the referral? a. the client demonstrates minimal limitations in cognitive level b. the client is able to consume chopped foods and apple juice with no difficulty c. the client demonstrates only oral stage problems during eating d. the client cannot tolerate therapy focused on improving feeding and swallowing skills

a. the client demonstrates minimal limitations in cognitive level the client with minimal cognitive limitations can be a candidate for the procedure. If the client could not tolerate therapy, an alternative intervention, such as a gastronomy tube, would be indicated

an OT plans intervention for an individual with cognitive perceptual deficits. In deciding whether to use a dynamic interactional approach or a deficit-specific approach which is most important for the OT to consider? a. the client's auditory processing skills b. the availability of financial support c. the client's social interaction skills d. the client's problem-solving skills

a. the client's auditory processing skills the dynamic interactional approach utilizes awareness questioning to help the individual detect errors, estimate task difficulty and predict outcomes. Therefore, the therapist must consider the client's level of auditory processing skills to determine if adaptations or modifications are needed when implementing this approach.

An OTA employed in a work-hardening program in a rehab facility demonstrates limited skills when evaluating clients' body mechanics. The OTA demonstrates excellent skills in other areas of eval and intervention. In the OTA's initial performance appraisal, which is the best for the supervising OT to suggest to the OTA? a. the development of a plan of action to acquire service competence in the evaluation of body mechanics b. assignment to another area of OT service delivery until service competence in the evaluation of body mechanics is established c. the completion of evaluations of client's body mechanics under routine supervision of an OT experienced in evaluations of body mechanics d. The completion of a continuing education course in body mechanics to increase knowledge about this area of practice.

a. the development of a plan of action to acquire service competence in the evaluation of body mechanics The OTA without established service competence cannot evaluate under routine supervision since this level of supervision involves limited contact between the OT and OTA. A continuing ed course can help increase knowledge but does not ensure competence in actual performance of an evaluation.

an OT designs a research project to study the efficacy of training nursing assistants in feeding techniques for individuals in a SNF who have dementia. Before implementing the study, whom must the OT first obtain consent? a. the facility's institutional review board b. the residents who will be the study's sample c. the designated representatives of residents who will be the study's sample d. the nursing assistants who will participate in the training

a. the facility's IRB researchers must obtain permission from the IRB to conduct any human research.

an individual who is acutely psychotic has been brought to the hospital by a legal guardian. The individual neither responds to questions nor attends to visual stimuli in the room. Who should the therapist collaborate with to determine the individual's short-term goal? a. the guardian b. the individual c. the psychiatrist d. the case manager

a. the guardian an individual decides goals in conjunction with the therapist except when the person is unable to take care of self, is a danger to self or others, or is unable to participate in the process.

The adult is hospitalized and diagnosed with mild COPD. During the discharge planning session, the person identifies a desire to exercise regularly. Which of the following should the therapist recommend the client to pursue? a. the hospitals wellness program's yoga group b. low-impact aerobics at a local gym c. weight-lifting under the direction of a personal trainer d. jogging in a local park with friends

a. the hospitals wellness program's yoga group yoga and stretching program put the least amount of pressure on the pulmonary and cardiovascular system. Also, the program is monitored by hospital personnel.

the supervising OTs meet to plan the workload of the recently hired OTAs. Which is an appropriate task for them to assign the OTAs? a. the in-home eval of the adult clients' IADLs b. the determination of long-term goals to include in the adult clients' OT home care intervention plan. c. the design of home-based sensory integration protocols for infants and toddlers with sensory processing. d. the administration and interpretation of the Hawaii Early Learning Profile (HELP) to infants and toddlers

a. the in-home eval of the adult client's IADLS the OTA can complete IADL evals under the supervision of an OT. The design of sensory integration protocols cannot be done by an OTA. An OTA can contribute to those protocols, but cannot independently complete them.

an OT receives a referral to eval an individual's executive functioning followed a mild CVA. Which are the most relevant foci for this intervention? a. the person's initiation and planning b. the person's attention and memory c. the person's job interests and efficacy d. the person's spatial relations and praxis

a. the person's initiation and planning executive functions are the higher-level cognitive abilities that are needed to perform unstructured multistep activities and tasks. Attention and memory are considered primary cognitive capacities that are prerequisite to higher-level cognitive ability.

a young adult with a diagnosis of schizophrenia, paranoid type is scheduled to be discharged from an inpatient setting to a halfway house and psychosocial clubhouse. The OT is assisting the team with their discharge plan. Which is the most important information for the therapist to provide to the team about this person? a. the persons IADL skills b. the possible effects of medication on the persons performance c. the persons volitional skills d. the persons social interaction skills

a. the persons IADL skills knowledge of IADLs is essential so that the halfway house staff know how much structure and support the person will need to have a successful transition. Nursing could help discuss the effect of medications, all team members can help with social skills, and vocational services are addressed at the clubhouse.

after a work-related injury to the left index finger, an assembly line worker is fit with a buddy strap incorporating the index and middle fingers. In describing the primary purpose of the strap to the client, which of the following explanations is most accurate for the therapist to state? a. the strap provides PROM to the index finger b. the strap reduces edema in the index finger c. the strap immobilizes the index finger d. the strap provides AROM to the index finger

a. the strap provides PROM to the index finger the strap can also help improve a deformity that has been caused by immobilization due to injury, weakness, or casting. A buddy strap can sometimes result in increased edema due to decreased circulation. The strap immobilizes the finger IP joints and thereby limits active ROM.

a rehabilitation facility is completing a utilization review. The OT on the utilization review committee contributes to the process by submitting a detailed report about the reasons for, costs of, and outcomes of a specific aspect of the OT service delivery. Which info is the most relevant for the OT to include in this utilization review report? a. the therapeutic use of the OT department's ADL apartment b. the splinting and AE prescribed to clients prior to discharge c. the use of personal care aides to provide morning self-care assistance to clients d. the craft and art materials used during leisure exploration groups

a. the therapeutic use of the OT department's ADL apartment A utilization review involves the analysis of the use of resources within a facility. It examines the medical necessity and cost efficiency of these resources. An ADL apartment is considered a facility resource. Splints, AE, arts and crafts materials are supplies, not facility resources. Personal care aids are personnel resources, not facility resources.

A client exhibits residual difficulties in a variety of perceptual skills. Which procedure should the therapist include in the eval of the client's proprioception? a. the therapist positions 1 extremity with the patient's vision occluded and asks the patient to duplicate the position with the contralateral extremity b. the therapist moves 1 extremity with the patient's vision occluded and asks the patient to duplicate the movement with the contralateral extremity c. the therapist demonstrates the desired position and asks the person to replicate the position in mirror fashion d. the therapist shows the client a picture of a position and asks the client to replicate the position with each extremity

a. the therapist positions 1 extremity with the patient's vision occluded and asks the patient to duplicate the position with the contralateral extremity the evaluation of movement is kinesthesia, not proprioception. Proprioception is not evaluated by replicating a demonstrated or pictured position

an OT designs a qualitative research study to examine the efficacy of an after-school play-based program for the development of social interaction skills. Which method of data collection is best for the therapist to use? a. the therapist's observations of the participating children in the classroom and during recess b. the completion of a social skills Likert-scale questionnaire by the participants' teachers c. the completion of a social skills Likert-scale questionnaire by the participant's parents d. the administration of a social skills eval pre and post-intervention

a. the therapist's observations of the participating children classroom and during recess qualitative research is a form of descriptive research that studies people individual individually or collectively in their natural social and cultural contexts. Likert-scales collect quantitative data

an OT conducts a sensory eval for an individual with cubital tunnel syndrome. Which of the following is the most likely location for this person's sensory symptoms? a. the ulnar aspect of the forearm and hand b. along the radial nerve distribution of the hand c. the medial aspect of the forearm and hand d. along the ulnar nerve distribution of the hand

a. the ulnar aspect of the forearm and hand cubital tunnel syndrome is an ulnar nerve compression at the elbow. Its symptoms include numbness and tingling along the ulnar aspect of the forearm and hand, pain at the elbow with elbow flexion, weakness of power grip and a positive Tinels sign at the elbow.

Several residents of a SNF report that they are bored with their individual daily ROM exercise programs. The OT collaborates with the PT to design a group format to facilitate participation in ROM exercises. Which would be the most beneficial for the therapist to recommend incorporating into the proposed group? a. the use of several exercise videos with diverse exercise styles and music b. the pairing of residents to perform gentle ROM on each other c. the use of exercised performed in rhythm to a marching band video d. the provision of coffee and cake after the group

a. the use of several exercise videos with diverse exercise styles and music adding variety can stimulate interest and increase socialization. Clients should not do hands-on movements with each other for safety reasons. A marching band video may be too vigorous for some residents. Providing food and drink does not address the need to increase interest in performing daily ROM exercises.

An individual with bilateral knee replacements and cataracts is newly admitted to a SNF. The individual retains some residual vision. During the initial eval, which is the most effective placement for the OT to use when presenting materials to this person? a. to the side of the person with no direct lighting b. directly in front of the person at eye level c. directly in front of the person at tabletop level d. to the side of the person, with a strong light shining

a. to the side of the person with no direct lighting an individual with cataracts loses central vision first, therefore presenting eval materials directly in front of the person will be ineffective. Individuals with cataracts have increased difficulty with glare, so indirect lighting is indicated.

An OTR is working with a 2-year-old child with feeding difficulties. The child is experiencing delays in self-feeding due to hand weakness and oral tactile hypersensitivity. What interventions are MOST BENEFICIAL to address these feeding concerns and improve self-feeding? a. universal cuff with a small smooth spoon b. a dycem mat and textured foods the child likes c. a mobile arm support and cold or sour foods d. a straw for drinking and assistance from the therapist

a. universal cuff with a small smooth spoon The universal cuff secures a utensil to the hand in a stable position which addresses hand weakness. The small, smooth spoon addresses the tactile hypersensitivity. A dycem mat helps stabilize but doesn't address weakness. A mobile arm support is more appropriate for children with severe shoulder weakness and cold or sour foods are contraindicated for children with tactile defensiveness in feeding issues. A straw may help but the child in this scenario does not appear to have cognitive or behavioral problems that require therapist assistance.

a 6 y/o child with difficulties in figure-ground perception is being discharged from OT services. In the discharge plan, which should the therapist recommend for this child? a. use a different solid-colored folder to hold materials for each class subject b. use a toothbrush and comb that are the same color as the counters c. wear clothing with bold colors and loud patterns d. place all school books and handouts on the desk while completing homework

a. use a different solid-colored folder to hold materials for each class subject This will help the child easily find class work. Contrasting color facilitate performance by accommodating for the figure-ground difficulties.

A client with rheumatoid arthritis is practicing meal preparation in an outpatient OT clinic. The client complains of pain in the hands with meal preparation. Which action is MOST appropriate for the OT to address? a. use of built-up handles for stirring b. use of heavy bowls to prevent sliding c. use of thin-handled knives for chopping d. use of large jars with more quantities

a. use of built-up handles for stirring RA is a chronic, systemic inflammatory condition characterized by pain, swelling, and stiffness of joints. Built-up handles decreases the grip strength required to manage spoons. Heavy difficult are more difficult to transfer. Thin knives are more difficult to manipulate and large jars are more difficult to carry.

An adult has been referred to OT. The individual demonstrates decreased ROM in the dominant hand secondary to a nerve injury. Active thumb ROM for the IP and MP is WNL. Active ROM of the IPs of all 4 fingers is 0*-60*. The individual wants to be able to hold a knife, spoon, and fork. Which utensils are best for the OT to recommend? a. utensils with cylindrical foam handles, 1.5 inches in diameter b. standard utensils with no adaptions c. utensils with custom-built handles made of low temperature thermoplastic splinting material d. utensils held in a universal cuff

a. utensils with cylindrical foam handles, 1.5 inches in diameter Without built up utensils, the individual can only use pad-to-pad grasp, which is unstable. A custom handle is most commonly used for someone with spasticity or more difficulty than mentioned in this scenario. A universal cuff is used by someone who has no functional grasp.

an OT reviews the use of the OT department's resources to determine medical necessity and cost efficiency. Which service management task is the therapist performing? a. utilization review b. retrospective peer review c. total quality management d. risk management

a. utilization review

An OT interviews an OTA for a position at a high school for gay and lesbian youth. The position involves the provision of transitional vocational programming and life skills training. The OTA uses a wheelchair for mobility and has dysarthric speech. Which of the following is the most relevant for the OT to ask the OTA about during the interview? a. verbal group leadership skills b. sexual orientation c. personal beliefs about homosexuality d. accommodations needed due to the evident disabilities

a. verbal group leadership skills Other questions are in violation of civil rights legislation and the ADA.

Following medical treatment for a brain tumor, a client is referred to OT home care services for a functional eval. During the initial interview, the client reports difficulty locating desired items. For example, at lunchtime the client could not find a can of soup in the pantry, Based upon this self-report, which functional ability should the OT evaluate? a. visual scanning b. visual acuity c. spatial relations d. topographic orientation

a. visual scanning visual acuity is clarity of far and near. Topographic orientation is the ability to find one's way in space.

An OT provides services to a homeless shelter which includes residents who are HIV positive. The therapist conducts several activity groups. Which should the therapist do while working with this population? a. wash hands before and after each group session b. always wear latex gloves during groups c. wear latex gloves when handling food d. implement transmission-based precautions

a. wash hands before and after each group This is a standard precaution that should be used by health care providers at all times. Due to potential latex allergies, health care environments must be latex-free.

A young adult with a 10 year history of bulimia participates in an outpatient vocational exploration group and completes a vocational interest inventory. This eval identifies 4 possible areas for a career. Which career choice is the best for the OT to advise the client to explore further? a. web designer b. dietician c. athletic trainer d. fashion designer

a. web designer

An OT working in an outpatient cardiac rehab center develops an intervention plan for an individual who has entered phase 2 of cardiac recovery. Which activities should the therapist recommend be included in this intervention plan? a. weeding a garden and doing low impact aerobics b. putting away groceries and keyboarding c. washing dishes and playing tabletop board games d. carrying groceries upstairs and playing basketball

a. weeding a garden and doing low impact aerobics at phase 2 of cardiac rehab, activities should begin at a 4-5 MET level and progress. Putting away groceries, keyboarding, washing dishes, and playing tabletop activities are at a 1-2.5 MET level. Carrying groceries and playing basketball are a 6-10 MET level.

The parents of a 2 y/o child with bilateral congenital UE amputations express concern to the homecare OT about their child's complete disinterest in toilet training. At which point should the OT advise the parents to begin toilet training? a. when the child indicates discomfort with being wet or soiled b. immediately because toileting is a developmentally appropriate task c. when the child is 3 y/o as this is the typical developmental age for toilet training d. by placing a child on the potty for 5 min per hr each day.

a. when the child indicates discomfort with being wet or soiled

a person recovering from hip replacement surgery wants to begin meal prep. The client refuses to use the walker that was ordered by the physician. The physician is unavailable for consultation. Which is the best initial action for the OT to take in response to this situation? a. work on meal prep activities with the client sitting at a table. b. work on meal prep activities with the client standing without the walker c. delay working on meal prep activities until the physician can be contacted d. tell the client the walker must be used until the physician changes the order

a. work on meal prep activities with the client sitting at a table There are many meal prep activities that can be done while seated so there is no need to delay meal prep activities. The therapist should not conduct the session without the prescribed ambulatory aid.

A patient incurred a traumatic UE amputation. During pre-prosthetic treatment, the OT molds the contours of the residual limb to shape it in preparation for a prosthesis. Which method is most effective for an OT to use? a. wrapping b. percussion c. shrinking d. massage

a. wrapping Wrapping by applying an elastic bandage to the residual limb in a figure 8 pattern will reduce the volume of the residual limb and shape it for a prothesis. Shrinking is the result of wrapping. Percussion and massage are used to desensitize a residual limb.

Which is the most effective approach for an OT to use when giving directions for a task to an individual with schizophrenia who is experiencing auditory hallucinations? a. written directions for making vanilla pudding b. verbal step-by-step directions for making a leather link belt c. general verbal directions for the completion of a group collage d. demonstration of steps during a swing class.

a. written directions for making vanilla pudding written directions with a structured, expected outcome are best to reinforce reality and to provide concrete feedback for the person experiencing hallucinations.

An OT conducts a satisfaction survey to evaluate the quality of OT services in an outpatient program. The questionnaire developed by the therapist asks respondents to rate their responses to quality statements according to a 4-point Likert scale of agreement. The statement that best reflects an adequate measure of satisfaction is: a. "Setting my own goals was important to me." b. "The amount of time devoted to stress management was adequate." c. "My experience in OT helped me." d. "The OT staff was respectful and fair to me."

b. "the amount of time devoted to stress management was adequate." this is the most helpful for those listed because it provides concrete info to evaluate service quality and improve service provision.

an OT implements intervention for individuals on an inpatient cardiopulmonary rehab unit. The therapist assesses a patients HR during intervention sessions by palpating a peripheral pulse. Which of the following most accurately describes the timing the therapist should use to complete this assessment? a. 30 seconds prior to, during, and after cessation of the activity. b. 1-2 minutes prior to, during, and at cessation of the activity and 5 minutes post-activity c. 1-2 minutes prior to, during and at cessation of the activity d. 30 seconds prior to, during and at cessation of the activity and 5 minutes post-activity

b. 1-2 minutes prior to, during, and at cessation of the activity and 5 minutes post-activity Individuals with normal HR only require 30 seconds of palpation, but individuals receiving treatment at a cardiopulmonary rehab unit will likely have irregular heart rhythms which requires 1-2 minute palpation

An OTR is interpreting a developmental test that was administered to a 5-year-old child. The child scored in the 74th percentile for the child's age and gender group. What can the OTR conclude based on this score? a. the child displays below-average developmental skills compared to similar children b. 26% of children in the sample group scored higher than this child c. the child has major delays compared to the normative sample d. the scores are reliable for measuring changes and guiding treatment planning

b. 26% of children in the sample group scored higher than this child We do not have enough information to conclude that the child has below-average developmental skills. 74th percentile does not necessarily equate to major delays. A criterion-reference test is the type of test used for measuring changes in development and guiding treatment planning.

an OT provides an accessibility consultation to a business that has hired a new employee who uses a wheelchair for mobility. The only entrance to the business has 4steps, each 7 inches high. which ramp length is the best for the therapist to recommend? a. 14 feet b. 28 feet c. 35 feet d. 46 feet

b. 28 feet rise to run is a 1:1 ratio

an OT professional education program provides an after-school play program for normally developing children to help students understand typically development. The students observe a child who is beginning to use blunt scissors to snip paper. The child opens and closes the scissors and moves them in a controlled forward motion, but the child cannot cut circles or figure shapes. At which age are these behaviors typical? a. 2 y/o b. 3 y/o c. 4 y/o d. 5 y/o

b. 3 y/o the described behaviors are too advanced for a 3 y/o, but children ages 3.5-4.5 are typically cutting circles. Children 4-6 are typically cutting figure shapes.

an OT provides consultation services to members of a town chamner of commerce who have expressed interest in improving their businesses' accessibility. Which is the minimum door width that the OT should recommend tot he chamber members as accessible and not requiring modification? a. 28 in b. 32 in c. 30 in d. 34 in

b. 32 in

an OT conducts a home eval for an individual who uses a wheelchair for functional mobility. The only entrance to the home has 5 steps, a total of 35 inches high. Which ramp length should the therapist recommend to the family? a. 17.5 feet long b. 35 feet long c. 48 feet long d. 70.5 feet long

b. 35 feet long rise to run= 1:1 ratio

A college is converting a historical building into wheelchair accessible dormitory space. To allow for a 360-degree turning radius, the occupational therapist recommends that the space between the student's desk and bed be a minimum of: a. 4 ft by 4 ft b. 5 ft by 5 ft c. 6 ft by 6 ft d. 7 ft by 7 ft

b. 5 ft by 5 ft This is the minimum. More space would be beneficial, but it isn't always allowable and the question specifically asked for the minimum.

An OT provides home-based early intervention evals. A referral for an 18 month old child notes that the child is able to finger feed effectively but is not able to use a spoon or suck from a straw. The OT puts together supplies to bring to the child's home and plans activities to use during the developmental eval. Which developmental age is most important for the therapist to consider when selecting objects and activities to bring to the eval session? a. 6-9 months b. 9-12 months c. 12-18 months d. 18-20 months

b. 9-12 months the referral notes abilities that are typical of the age of 9-12 months. Straw use typically develops about 18 months.

an OT administrator is designing a patient satisfaction questionnaire to be administered upon discharge from the OT program. The administrator designs the questionnaire so that the individuals will indicate their best level of agreement with a series of statements by circling a number with 1= very dissatisfied, 2= dissatisfied, 3= neutral, 4= satisfied, and 5= very satisfied. Which method of data collection is the administrator using? a. gutman scale rand ordering b. Likert scale c. retrospective data d. a semantic differential

b. a Likert scale

An OTR is working with a 2-year-old child with arthrogryposis, a congenital orthopedic disorder, who has significant limitations in hand function. What is the BEST intervention to promote play skills in this child? a. hand over hand manipulation of a variety of toys b. a button switch to operate a toy car to move c. the mother playing peekaboo with her child d. finger painting on the floor in the kitchen

b. a button switch to operate a toy car to move These children often have contractures that effect play. AE, such as a button switch, can promote occupational performance. Hand over hand is not the best choice to get independent engagement, a mother playing peekaboo does not allow the child to interact with her environment, and finger painting may be too frustrating for a child with this condition.

A high school senior with Friedreich's ataxia is working on developing keyboarding skills in a school-to-work transition program. While assessing the student's capabilities for computer keyboarding, the OT observes signs of dysmetria. Which is the most appropriate adaptation for the OT to recommend to increase the effectiveness of the student's keyboarding skills? a. an eye-gaze input system b. a key guard overlay c. a voice-activated input system d. a reduced size keyboard

b. a kay guard overlay Dysmetria is the overshooting or undershooting of a target. A key guard overlay provides raised separations between each key. This enables the individual to place his or her finger into the desired key space. A person with this type of ataxia has poor coordination of all muscles including ocular muscles making eye-gaze ineffective. Dysarthria is also a characteristic of Friedreich's ataxia which limits the efficacy of a voice-activated system.

An individual is 5' 11'' and is of average weight for this height. Following a recent TBI, the person has flaccid hemiparesis and demonstrates poor righting and equilibrium responses in standing. Which wheelchair is best for the OT to recommend for this client? a. a power wheelchair b. a light weight, standard size wheelchair c. a reclining, hemi-height wheelchair d. a one-arm drive, standard sized wheelchair

b. a light weight, standard sized wheelchair It is light and therefore easy to use. The client is too tall for a hemi-height wheelchair. A power chair is used for a person with significant functional deficits (quadriplegia)

A 21-mo. old child with severe spastic quadriplegia is evaluated by an OT. The therapist determines that the child is cognitively intact, exhibiting age-appropriate cognitive skills despite major sensorimotor deficits. The therapist recommends a play activity to enhance these cognitive abilities and provide the child with a fun and pleasurable experience. Which is the best object for the therapist to recommend? a. a multi-colored mobile of objects of interest placed over the child's stroller b. a mechanical toy with a chin controlled on/off switch c. a shape sorter with foam squares, triangles, and circles d. a battery controlled hammock swing

b. a mechanical toy with a chin controlled on/off switch at 21 mo, a child is cognitively able to operate and control mechanical toys . A mobile is too low for the cognitive abilities. Ability to identify and sort shapes does occur at 21 mo. but the use of a shape sorter requires motor abilities beyond this child's capacities.

An OT advises the parents of an 18 mo old with DD on techniques to facilitate feeding. The child has a reflexive bite. which utensil is most beneficial for the OT to recommend the parent to use when feeding the child? a. a deep-bowled soup spoon b. a narrow shallow coated spoon. b. traditional teaspoon d. a plastic spork

b. a narrow shallow coated spoon a narrow shallow coated spoon will help the food slide off while a deeper spoon makes it more difficult

an OT conducts a sensory evaluation of an individual recovering from a left CVA. The individual has right hemiplegia and expressive aphasia. During the evaluation of stereognosis, which should the therapist have the client use to identify reponses to the testing stimuli? a. pictures of the objects b. a set of identical objects c. cards with "one" and "two" printed on them d. cards with "yes" and "no" printed on them

b. a set of identical objects cards with "one" and "two" on them would be relevant for two point discrimination tests, and "yes" or "no" cards would be relevant for light touch.

The parent of a newborn infant has bilateral shoulder weakness and is referred to OT for training in energy conservation techniques for the performance of parenting and home management tasks. Which adaptation is most effective for the OT to recommend? a. a top-loading washer and dryer for clothing care b. a steamer, steamer basket, or crockpot for meal preparation c. a front pack carrier for holding the infant d. cloth diapers and the use of a weekly diaper care service

b. a steamer, steamer basket, or crockpot for meal prep this suggestion eliminates the need to move and lift heavy pots or pans, which require intact bilateral UE strength. A top-loading washer/dryer requires more effort than a front-loading washer/dryer. A front pack infant carrier has straps that cross the shoulders which would be contraindicated in this case.

an OT designs a quantitative research study to examine the treatment efficacy of a pain management program. Which method of data collection is best for the OT to use? a. a survey with open-ended questions b. a survey with Likert scale questions c. a discussion with focus groups d. individual interviews with program participants

b. a survey with Likert scale questions This scale allows for numerical data to be collected. All other options produce qualitive data

an OT implements intervention with 5 patients using group format. The therapist charges each patient's insurance provider for individual treatments. Which of the following does this action represent? a. an example of impairment b. a violation of procedural justice c. an established, accepted practice d. a correct action, if group interventions are individualized.

b. a violation of procedural justice procedural justice is the principle in the AOTA code of ethics. This includes being truthful in charging for services and meeting legal requirements for documentation.

an individual with RA is currently in a stage of remission. During this inactive chronic phase of the disease, the OT works with the client to maintain ROM and muscle strength. Which of the following is most effective for the therapist to recommend the client include I a daily home exercise program? a. passive ROM b. active ROM c. isotonics d. progressive resistance

b. active ROM activities that use active ROM are indicated for the treatment of RA both in its acute and chronic stages. Passive ROM and progressive resistance is generally contraindicated with these clients. Use of isotonics with this population is rather controversial. The OT must determine that the individual's joints are stable and would benefit from isotonic exercises without jeopardizing other joints.

An OTR is working with a child with arthrogryposis in a school setting. What type of intervention approach is MOST BENEFICIAL for a child with this type of congenital condition? a. CIMT b. activity adaption approach c. neurodevelopmental therapy approach d. motor learning approach

b. activity adaptation approach Children with this condition have significant limitations in hand function beginning at birth with limited increase in function as they age. Because they experience a loss of ROM they often learn adapted methods. CIMT is used for children with hemiparesis CP. NDT is used to inhibit spasticity and primitive reflexes. The motor learning approach is most beneficial for children who have the aptitude to follow instructions and are motivated to tolerate repetitive practice. Children with this condition fatigue easily and may not best fit this approach.

An OT works with a patient with diabetes and a below-knee amputation to learn how to effectively perform home management tasks while wearing a BK prosthesis. When taking laundry from a front-loading washer and placing it into a top-loading dryer, the patient reports feeling weak, dizzy, and somewhat nauseous. The OT notices that the patient is sweating profusely and is unsteady when standing. Which is the best immediate course of action for the therapist to take in response to the patient's complaints? a. return the person to the unit of care due to an insulin reaction b. administer OJ for developing hypoglycemia c. call the nurse to administer an insulin injection for developing hyperglycemia d. have the patient sit down until the orthostatic hypotension resolves.

b. administer OJ for developing hypoglycemia This results from too much insulin. It requires prompt intervention. Having the patient sit and ingest an oral sugar is the best immediate action for the therapist to take. Once the patient is stable, the physician should be notified. Profuse sweating and nausea don't typically accompany orthostatic hypotension.

a parent admitted to a psychosocial day treatment program has a diagnosis of major depressive disorder. Results of the OT screening process indicate that the person is having difficulty performing home management tasks and caring for 2 young children. The OT has determined the need to gather further data regarding past and current performance. The therapist decides that the recently hired, entry-level OTA will contribute to the eval process. Which evaluation is best for the therapist to assign the OTA to complete? a. a cognitive eval b. an ADL eval c. A mental status exam d. an occupational history interview

b. an ADL eval OTAs are trained and qualified to perform ADL evals during their entry-level education. The other answer choices require the establishment of the OTA's service competence. In this scenerio, the OTA was new, so these competencies would not have been established

an OT receives a referral to construct a splint for an individual with Erb's palsy. Which orthosis would be most effective for this condition? a. a flail arm splint b. an elbow lock splint c. a figure 8 splint d. a deltoid sling

b. an elbow lock splint Erb's palsy results from injury to the 5th and 6th brachial plexus roots causing the arm to hang limp with the shoulder rotated inward due to atrophy and paralysis in the biceps, deltoid, brachialis, and brachioradialis muscles. The elbow lock splint stabilizes the elbow to enable the individual to position the hand closer to or away from his or her body for functional use.

During an OT session, a veteran recovering from bilateral traumatic LE amputations expresses several concerns about having a changed body and questions its sex appeal. Which response is best for the therapist to take in response to the client's concerns? a. refer the client to the primary care physician b. ask open-ended questions to explore the concerns further c. reassure the client that the concerns are a normal part of the recovery process d. explain that positioning can compensate for changes in functional mobility

b. ask open-ended questions to explore concerns further sexuality is within an OTs domain, therefore, there is no need to refer to a physician

In an acute inpatient psych facility, an OT designs a therapeutic activity group for individuals with poor orientation to reality. Which is the best activity choice for the therapist to provide in this group? a. a discussion of the effects of hospitalization on occupational roles b. the assembly of wooden toys for a children's unit c. guided imagery for stress management d. structured verbalizations of personal assets and limitations

b. assembly of wooden toys for a children's unit on an acute inpatient psych floor, activities should be structured, easily completed in one session, and provide a concrete result to reinforce reality. Wooden toy kits meet these criteria and donating them to children's units facilitates the curative factor of altruism. discussions and verbal activities are abstract and would be difficult for persons with poor orientation to reality. Guided imagery can be difficult for a disoriented person to focus on and can be frightening to an acutely ill person.

A child with spastic CP has increased tone and flexor synergies in both UEs. The child receives Botox injections into both biceps. In designing play activities to use during intervention, which is most important for the OT to include as an activity performance characteristic? a. reaching overhead and to both sides in supine b. bilateral UE weight-bearing and sitting in prone c. grasp and release of objects in supported sitting d. transitions into and out of sitting, supine, and prone

b. bilateral UE weight-bearing and sitting in prone The theory supporting the use of Botox is that the Botox lessens the spasticity for 3-6 months. In this case, the child has flexor spasticity, so relation of the biceps to allow strengthening of the triceps is indicated. Therefore, the most important focus of the intervention is weight-bearing of the UEs since this can strengthen the triceps.

The OT plans intervention for a client with a recent diagnosis of CRPS type 1. Which intervention approach is most effective to use to reduce pain and increase function? a. hot packs b. biofeedback c. paraffin d. PROM

b. biofeedback CRPS type 1 is a vasomotor dysfunction which causes extreme hypersensitivity to touch, edema, intense burning pain, and dramatic temperature and color changes to the affected limb. Biofeedback is a technique where electrodes are used to measure muscle responses and stress levels. The goal is to train the individual to release tension, which can reduce pain and prepare the individual for increased tolerance to ROM. Hot packs and paraffin can be too painful to tolerate in the initial stages and are contraindicated if the affected limb demonstrates elevated temp.

During an intervention session focused on the development of grasp and shoulder mobility, the OT asks the client to move numerous identical 1 pound cans of vegetables from the counter top to the cabinet shelf above the counter. According to contemporary motor learning approaches, which type of practice has the therapist designed this activity to provide to the client? a. random practice b. blocked practice c. planned practice d. contextual practice

b. blocked practice blocked practice involves repeated performance of the same motor skill. Since the cans are identical, lifting them requires the same motor skill.

An occupational therapist was working with a client to address his ability to feed himself during breakfast. The client suffered a middle cerebral artery stroke, which affected his right hemisphere. The client was able to drink from the cup of coffee using both hands. He set the cup down but kept the cup in his left hand as he reached for the fork on the right side of his plate. Coffee spilled on his clothes as a result. The client made a comment that he forgot he had the coffee cup in his left hand. What BEST describes the behavior that the occupational therapist observed during this activity? a. motor apraxia b. body neglect c. impaired figure-ground d. ideational apraxia

b. body neglect this is a tactile perception disorder or body schema disorder. A person with this disorder will often neglect the functional use of the affected side. A client with motor apraxia will have difficulty adjusting his grasp. A client with ideational apraxia will have difficulty identifying the correct use of the coffee cup or fork.

A client attends a work hardening program. The client arrives on time for the scheduled session but complains of significant substernal pain, extreme discomfort in the epigastric area, indigestion and nausea. Which is the best action for the OT to take in response to the client's expressed complaints? a. initiate the session and provide breaks and activity modifications as needed b. cancel the session and immediately call emergency medical services c. cancel the session and tell the client to call and reschedule when feeling better d. cancel the session and call the client's physician to discuss the complaints

b. cancel the session and immediately call emergency medical services The client is complaining of symptoms that can indicate a MI. The therapist should not delay calling EMS by cancelling the session and having the client reschedule or speak with the physician.

An adult is referred to OT and PT for treatment of chronic back pain that hiders the satisfactory performance of home maintenance tasks. The OT in the department has little experience with back pain, while the PTs in the department have extensive experience in this area. Which is the best choice for this OT to take in response to the situation? a. refer the patient to PT b. co-treat the person with a PT c. give the patient the option to work in OT or PT d. review a video on treatment of back pain

b. co-treat This is an opportunity to improve the therapist's skills by collaborating with a team member.

an OT in a long-term care facility is working with a resident to develop swallowing and eating skills. The resident has a history of dysphagia and is taking long periods of time to chew chopped food. The food consistency was prescribed by the SLP. Which action is best for the OT to take to address the resident's swallowing and eating difficulties? a. modify the prescribed food consistency to pureed to prevent aspiration b. collaborate with the SLP to determine the best food consistency for the resident c. train nursing staff to feed the resident slowly and in small amounts d. call the physician to request a videofluroscopy procedure

b. collaborate with the SLP to determine the best food consistency for the resident Because the resident's food consistency was prescribed by the SLP, it is best to take a collaborative approach. Training the nurse to feed slowly does not address the issue at hand.

An OTR completed a developmental assessment of a 5-year-old child with Down syndrome. Results indicated the child is dependent with most self-care activities. The parents do not place a high importance or priority on feeding or dressing independence, but the OTR does. What action is MOST IMPORTANT for the OTR to take as part of the intervention planning process? a. talk to the parents about establishing independence in self-care skills as a primary goal from the child b. collaborate with the parents and identify mutually acceptable treatment goals for the child c. ask for consent from the parents to begin working on feeding and dressing skills with the child d. inform parents of the importance of self-care independence for school-aged children

b. collaborate with the parents and identify mutually acceptable treatment goals for the child Collaborating with the parents to identify mutually acceptable goals is essential to the intervention process. Talking to the parents about making self-care goals the primary goal or asking for consent does not respect the family's roles or decision making process. Informing the parents of the importance does not incorporate the parents into the goal setting.

an individual recovering from myasthenia gravis has fair minus (f-) muscle strength in both UEs. The OT develops an intervention plan to include the goal of increasing muscle strength. According to the biomechanical approach, which should the therapist work on with the patient during the intervention? a. complete active ROM with gravity decreased b. complete active ROM against gravity c. incomplete active ROM against gravity d. complete active ROM against gravity and slight resistance

b. complete active ROM against gravity. An F- indicates that the body part can move through incomplete ROM against gravity. The next muscle grade is fair which indicates the ability of the body part to move through its complete ROM against gravity.

an OT provides home care services to a neonate with significant developmental delays. 2 hours before the next scheduled home visit, the child's parent informs the therapist that 1 of the other 3 children has developed chicken pox. While the other children do not show signs of chicken pox, the parent expresses concern that they are highly contagious. What is the therapist's best response to this situation? a. cancel the scheduled session and reschedule after 2 weeks have passed. b. complete the scheduled session using airborne precautions c. complete the scheduled session using standard precautions d. complete the scheduled session using droplet precautions

b. complete the scheduled session using airborne precautions there is no need to cancel the session. Standard precautions are used in all clinical situations. Chickenpox is a disease transmitted by airborne droplet nuclei.

an OT works with a person who incurred full thickness burns to both arms. Which intervention approach would be most effective for the OT to provide to control hypertrophic scar formation? a. axillary splints applied in the airplane position b. compression garments c. wound crafting d. elevation of the areas just above heart level

b. compression garments custom-made compression garments provide equal pressure over the area to prevent scarring. They must be worn 23 hrs a day for 1 yr or until the scar and wound maturation is complete. Wound grafting is used as biological dressing to provide covering and pain relief. Elevation is a technique to reduce edema.

during an intervention session focused on developing home management skills, a client made a grocery list. The client grouped needed items together to make shopping easier and listed eggs separately from all of the other items. When explaining how the list was composed, the client stated, "eggs break, they should be on top." Which of the following is the most accurate for the OT to report that the client's approach to this task represents? a. diminished insight b. concreteness c. anosognosia d. poor sequencing

b. concreteness the statement reflects concrete thinking which is concerned with the actual properties of things and the realities of situations, rather than abstract properties or situational potentialities.

During a home care intervention planning meeting, the team discusses a client with a right CVA. The PT states that the individual's ambulatory status is now WFL. PT services will be discontinued because the person is no longer home bound. The OT reports that the individual is frequently confused during home management tasks and becomes extremely anxious when community activities are discussed. Which recommendation should the therapist make? a. refer the individual to a psychiatrist for a mental status eval b. continue OT services as the person should continue to be considered homebound c. discontinue OT services as they are non-reimbursable since the person is no longer considered homebound d. contact the physician to discuss the need for OT on an outpatient basis and for psychosocial counseling

b. continue OT services as the person should continue to be considered homebound the individual can be considered homebound for cognitive and psychosocial deficits. Discontinuing services can place the individual at risk, and there is no need for physician input because the OT can continue to provide services in this scenario without physician input

An OTR is working with a 5-year-old child with a diagnosis of developmental delay. During the evaluation, the child demonstrated sensory-seeking behaviors, distractibility, and poor engagement or avoidance of tabletop activities. Evaluation results in revealed deficits in fine motor and visual motor skills as well as poor processing of vestibular, tactile, and proprioceptive input. What INITIAL intervention should the OTR use based on the evaluation results? a. tracing letters on a slanted surface b. crawling through an obstacle course c. cutting out shapes while sitting at a table d. copying and printing name on lined paper

b. crawling through an obstacle course This uses the SI approach and provides the child with the opportunity to process vestibular, tactile, and proprioceptive input. Tracing letters and cutting shapes are not the initial interventions needed because the child would be too distracted because sensory processing concerns have not been addressed. Copying and printing name will be important to work on once attention span has increased.

A child with developmental delay has mastered the ability to cut simple figure shapes with scissors. Which scissor activity is the best for the OT to next introduce the child? a. cutting simple geometric figures b. cutting complex figure shapes c. cutting multiple circles d. cutting additional simple figure shapes

b. cutting complex figure shapes In typically developing children, these abilities develop between ages 4-6. Abilities to cut circles and geometric shapes begin around ages 3-4.

An OT develops a task group for the newly admitted patients of a psych inpatient unit of a busy city hospital. The therapist considers several activities to use for the group's first session. Which activity is the best for the therapist to present to the group members? a. planning a weekend pizza party for the patients and their visitors b. decorating Styrofoam cups and planting cuttings in them c. publishing a weekly newsletter about city attractions for patients in the unit d. patining a large mural to cover one wall of the day room.

b. decorating Styrofoam cups and planting cuttings in them this is a simple, concrete, and safe task. In addition, individuals on an acute unit have a short length of stay and require activities that can be completed in 1 session.

The OTR applies metal cylinders that vary in temperature to a client with a hand injury. The client demonstrates no response to all cylinders applied to the hand. Which BEST describes this client's impairment? a. decreased pain and touch awareness b. decreased temp sensation c. decreased stereognosis awareness d. decreased proprioception sensation

b. decreased temp sensation Temperature awareness is tested by applying various cylinders with hot and cold to the involved hand.

a person is recovering from a major cardiac infarct. During the initial OT session, the patient loudly and vigorously expresses plans to immediately resume a daily rigorous exercise routine. The OT reports the individual's plan tot he cardiac rehab team and explains that the individual appears to be in which of the following disability adjustment stages? a. shock b. denial c. acting out d. acceptance

b. denial denial is characterized by unrealistic expectations of recovery and a minimization of one's difficulties. Shock is characterized by emotional numbness, depersonalization, and reduced speech and mobility. Acceptance is reflected in acknowledgement of the situation. Acting out is a term used to describe behavior that challenges societal norms.

An OTR working in early intervention is developing a plan of care for a new client. The 2-year-old demonstrates sensory challenges and social participation delays, so the pediatrician recommended occupational therapy services to the family. What is FIRST major goal in the evaluation and intervention planning of infants and toddlers? a. determine skill sets that may be delayed and need intervention based on the parental report b. determine eligibility for early intervention services through observation and assessment c. develop transition plan for day care and school that supports sensory challenges in the environment d. write therapist-based goals and objectives including focus on sensory processing and social engagement challenges

b. determine eligibility for early intervention services through observation and assessment There must be an attempt to determine eligibility because if the child is not eligible, the OT cannot provide further service. Developing transition plans are usually one of the final steps as one ages out of the program. Writing goals occurs after the eval with the family.

an experienced OT assumes a new position as the director of a department of 10 therapists, 4 OTAs, 3 aides, and a secretary. The department is in a state of chaos with therapists complaining about case loads and poor support from the administration. Which action is most important for the new director to take first in response to the situation? a. request that the director of HR complete performance appraisals on all department employees b. determine the basis of the complaints to design targeted conflict-resolution strategies c. organize the distribution of patients among all the therapists d. offer positive feedback to all staff on performance and attitudes as merited

b. determine the basis of the complaints to design targeted conflict-resolution strategies once info has been gathered, conflicts can be targeted. These actions could then include reorganizing the distribution of patients and providing feedback.

An adult who incurred a TBI 3 months ago is referred to a home care agency to receive OT services. The client's referral states that cognition is at a level VII of the Rancho Los Amigos scale and that grasp and shoulder mobility are limited. During the initial interview, the client reports being frustrated by the inability to independently engage in the previously enjoyed and personally meaningful activity of cooking. The client reports frequently losing place when reading recipes and an inability to find things in the kitchen. The client is attending a vocational rehab program 3 days a week but is frequently late due to difficulties with getting ready in the morning. The client asks the home-care therapist for suggestions to address this problem. Which is the most appropriate action for the OT to take in response to this request? a. advocate that the vocational program provide the client with a flexible start time b. develop a visual chart with the client, depicting the necessary sequence of his morning activities c. advise the client to call the vocational program to tell staff when running late d. advise the client to wake up 1 hour earlier on vocational rehab days

b. develop a visual chart with the client, depicting the necessary sequence of his morning activities Individuals at this rancho level have cognitive abilities that are automatic-appropriate. They can initiate and attend to highly familiar tasks in a distraction-free environment, but have poor recall of what has been completed. A visual chart will allow the individual to check off what he or she has already done. In addition, having extra time can increase potential for distraction and decrease focus. While it is polite to call when running late, this does not address the issue.

The parents of an 8 mo old child bring their child to a free community health developmental screening program. The OT evaluates the oral motor development of the child and determines that the child's development is within normal limits. When documenting this, which is most likely for the therapist to state this child demonstrates? a. rotary chewing b. diagonal jaw movements c. effective mastication d. cup drinking with a firm jaw

b. diagonal jaw movements Diagonal jaw movements develop as early as 7 mo and would be evident by 8 mo. Effective mastication typically develops at 9 mo. Cup drinking and rotary chewing are typical for a child at 1 yr.

a homemaker and parent is hospitalized for depression and prescribed Parnate to treat depressive symptoms. The patient's hobbies are gardening and jogging. Upon discussing the functional effects of medications with the patient, which is the most important precaution for the OT to review. a. photosensitivity b. dietary restrictions c. orthostatic hypertension d. amenorrhea

b. dietary restrictions Parnate is an MAOI. It has serious effects when a person eats foods that contain the amino acid tyramine. The other side effects are common of psychiatric meds but not typically MAOIs.

A school-based OT receives a referral for evaluation from a 3rd grade teacher. The teacher reports difficulty with a student who has illegible handwriting, poor attending behaviors, questionable visual skills, and problems with pencil management. After speaking with the teacher, reviewing the classroom work samples, and reading the student's history, which action should the therapist take next? a. provide pencil grips and specialized paper as a trial to determine interventions b. directly observe the student during a naturally occurring writing time c. administer standardized visual perceptual and visual motor assessment d. administer a standardized handwriting assessment

b. directly observe the student during a naturally occurring writing time skilled observation during a writing activity is an essential part of the eval process. This should precede standardized testing of performance skills. The observation and teacher interview should then direct the therapist to the most appropriate standardized measures if needed.

An OT is scheduled to give a 1 hour presentation to a support group of parents of infants with a diversity of developmental disabilities. Which of the following is the most important focus of the therapist's presentation? a. demonstration of infant positioning techniques b. discussion of typical areas of concern addressed by OT practitioners c. demonstration of different types of developmental assessments d. discussion of the individual family service plan (IFSP)

b. discussion of typical areas of concern addressed by OT practitioners Demonstration of positioning techniques and developmental assessments can be informative, but these techniques must be individually tailored to each child.

An OT wishes to compare the results of an eval of lateral pinch to the norms. Which is the best positioning for the therapist to place the individual in when using the pinch meter? a. forearm in neutral and the pinch meter placed on the DIP joint b. forearm in neutral and the pinch meter placed on the middle phalanx c. forearm in pronation and the pinch meter placed on the DIP joint d. forearm in pronation and the pinch meter placed on the middle phalanx

b. forearm in neutral and the pinch meter placed on the middle phalanx norms for lateral pinch have been established in this position.

An OT provides home-based services to a person recovering from a recent CVA. The individual lives alone and receives home care Medicare Part A benefits. The therapist arrives at the client's house at the scheduled session time, but there is no response to the knocking on the door. A neighbor states that she saw the client leave with a friend. The most appropriate action for the therapist to take is to: a. call the nurse case manager to report the missed appointment b. document that no one answered the door and that the appointment will be rescheduled c. documents that no one was home and that the appointment will be rescheduled d. document that the client is engaged in community mobility activities and should be evaluated for discharge

b. document that no one answered the door and that the appointment will be rescheduled documentation must state that no one answered the door because that is factually correct and allows the individual to continue to receive home care service reimbursement from Medicare. One missed appointment is not basis for discharge.

a 15 y/o with asymptomatic HIV attends an outpatient clinic. The OT protocol for patients diagnosed with HIV includes presentation of information on safe sex. the adolescent's parents refuse to allow this info to be presented to their child. Which id the best action for the therapist to take? a. ask the patient's opinion and act on the patient's refusal or consent. b. document that the parents refuse the intervention for the child. c. refer the family to the social worker for counseling. d. have the parents sign a waiver that they refused the intervention for their child.

b. document that the parents refuse the intervention for their child.

A child with moderate spastic CP works on ambulation with a walker in PT. The OT eval reveals problems in LE dressing, transitional skills, self-feeding, and grasp and release skills. To facilitate the attainment of the child's ambulation goal, which activity is the best for the OT to include in the OT intervention session? a. rolling in and out of prone and supine while on a mat, with minimal hands-on facilitation b. donning and doffing shoes and socks in bench sitting with one leg externally rotated and placed on the opposite knee c. self-feeding with elbows supported on a cut-out table at waist height, using a spoon with a built-up handle d. practicing grasp and release of small objects while wearing bilateral soft splints to support a more functional hand position

b. donning and doffing shoes and socks in bench sitting with one leg externally rotated and placed on the opposite knee this activity encourages dynamic trunk balance, LE external rotation, and dissociation. This activity is the best one listed to address the skills needed for ambulation while meeting OT goals related to performance in areas of occupation.

an adult recently diagnosed with scleroderma receives OT services to deal with the functional changes caused by this disease. Which recommendation is best for the OT to make? a. dress in lightweight clothing for thermal comfort b. dress in layers for neutral warmth c. use pull on clothing to ease donning and doffing d. use Velcro/or a button hook to ease fastening

b. dress in layers for neutral warmth scleroderma is a systemic disease. Symptoms are grouped into the CREST syndrome, including calcinosis, Raynaud's phenomenon, esophageal dysfunction, sclerodactly of fingers and toes, and telangiectasis. A common and early symptom of scleroderma is poor circulation. Dressing in layers compensate for this problem. Dressing in light clothes would not address the person's need for warmth. The other options are only indicated if the disease progresses and the individual develops contractures.

An individual incurred a SCI at the C5 level. During an OT session focused on developing the ability to feed independently using AE, the OT notes that the client is flushed and sweating excessively. The client requests that the session end early due to a pounding headache. Which action is best for the OT to take first in response to this situation? a. end the session and call the transporter to return the client to the client's room b. empty the client's filled catheter bag and maintain the client's upright position c. return the client to the unit and report symptoms to the head nurse d. stop the session and recline the individual in the wheelchair for a rest break

b. empty the client's filled catheter bag and maintain the client's upright position The client's symptoms are indicative of autonomic dysreflexia. This is deemed a medical emergency and should be treated immediately. Common stimuli are blocked catheters or sitting on sharp objects.

A teenage girl with juvenile rheumatoid arthritis identifies a goal of applying her own makeup. Which adaptation Is most beneficial for the therapist to recommend? a. silver ring splints to hold makeup applicators b. enlarged soft foam handles on makeup applicators c. long thin handles on makeup applicators d. universal cuff to hold makeup applicators

b. enlarged soft foam handles on makeup applicators these will facilitate independent grasp and increase independence. Silver ring splints are not designed to hold objects, and a universal cuff is an adaptation for a person with no ability to grasp which is not indicated in this case.

an OT leads a transitional planning group for high school students with conduct disorders. The school fire alarm goes off 5 min before the group's scheduled termination. There have been 6 false alarms during the past 3 days at the school. Several of the students laugh and say "there it goes again." Which is the OT's best response to this situation? a. call the school's main office to determine the validity of this alarm b. escort the students to the nearest fire exit c. conclude the group with a discussion about the implication of false alarms d. escort the students back to their homeroom classrooms to await directions.

b. escort the students to the nearest fire exit

an OT administrator implements a quality improvement program at a large private hand therapy clinic. The administrator determines that the clinic's OTAs are not completing their assigned initial screenings in a timely manner. This has resulted in scheduling delays for complete functional evaluations. Which initial action is most effective for the administer to take in response to this situation? a. counsel the OTAs on the need to adhere to screening schedules b. examine the organizational structure of the screening process c. assign the OT to complete all screenings d. redesign the screening to simplify the process

b. examine the organizational structure of the screening process a fundamental part of quality improvement is to explore organizational needs. Counseling and reassigning may not effectively address the underlying reasons for the delays. Before redesigning the process, the administrator must first examine the organizational structure.

An OT leads a work group at a vocational rehab program for persons with TBIs. 1 member begins to make sexually suggestive comments to other group members. The therapist redirects the client to the work in progress, but the member continues to make sexually suggestive statements. Which is the therapist's best initial response to this situation? a. explain to the client that such statements are not tolerated at work and call security to have the client removed from the group b. explain to the client that such statements are not tolerated at work and the client must stop or leave the group c. end the group before the situation escalates and reschedule the group to meet without the disruptive client d. set the client up at a different work station so the client is not in contact with other group members and cannot disrupt the group's work

b. explain to the client that such statements are not tolerated at work and the client must stop or leave the group An important aspect of vocational rehab for people with TBIs is the development of appropriate social interaction skills.

an adolescent with Duchenne Muscular Dystrophy refuses to use mobile arm supports (MAS) because "they look so big and stupid." which action should the OT take first in response to the client's statement? a. collaborate with a rehab engineer to design a more compact device b. explore other options with the client to perform activities that do not use MAS c. provide several logical reasons for using MAS to enhance functional performance d. Discharge the client and follow up with after 1 month to reassess interest in the MAS

b. explore other options with the client to perform activities that do not use MAS This response is an example of therapeutic use of self and client centeredness. Developing a different design is a long-term option but may not be feasible. Providing logical reasons is not the best initial response as it ignores the client's feelings of frustration.

An elder resident of a SNF becomes tearful during an OT session. The resident has a diagnosis of advanced OA and relates that pain is causing discomfort during sexual activities. The resident expresses fear about losing a valued intimate relationship and asks the OT for advice. Which is the most beneficial action for the therapist to take in response to the resident's expressed concerns? a. refer the resident to social work for individual counseling b. explore the resident's goals for sexual expression c. refer the resident and the resident's significant other to social work for couples counseling d. advise the nursing of the resident's statements to ensure that sexual behavior is monitored.

b. explore the resident's goals for sexual expression Sexuality and sexual expression are in the practice of domain of OT. Once the individual's goals are established, strategies to attain these goals can be explored. Strategies can include the use of activity analysis, gradation, modification, simplification, non-medical methods to manage main and stiffness, positioning alternatives, AE, energy conservation techniques, and referrals to other professionals.

The residents of a halfway house plan a community leisure activity for a Saturday. Two residents state that they cannot participate in Saturday activities due to religious observances. The other residents express strong interest in the activity. Which is the OT's best response to this situation? a. schedule an in-house Saturday leisure activity for the two observant residents b. explore with the group an alternative schedule for a community leisure activity c. schedule an in-house leisure activity for all residents d. recommend the 2 observant members seek approval from their religious leadership to attend the Saturday activity

b. explore with the group an alternative schedule for a community leisure activity All residents should be provided with the opportunity to engage in the community leisure activity.

An OTR is working with a family who has a child with reactive attachment disorder. The child has been stealing, hoarding food, and does not comply with directions to complete chores around the house. The child is detached and unaffectionate with his parents, but very nice to his teacher and soccer coach. What is the MOST APPROPRIATE intervention to do INITIALLY to address this child's behavior? a. prepare and only eat meals in the kitchen and keep locks on the kitchen cabinets b. family education about reactive attachment disorder and setting realistic goals c. a family meeting to discuss progress on goals and bring about positive therapeutic outcomes d. a family chore list with rewards and consequences for competed/not completed items

b. family education about reactive attachment disorder and setting realistic goals Education and realistic goals are the primary actions needed. A family meeting to discuss progress on goals is important but as at an initial meeting no progress has been made yet because goals have not been set. A family chore list is the next step but cannot be done until the family meeting has occurred.

A female graduate student in her mid-20s was recently involved in a MVA and suffered multiple cervical fractures. She received a cervical fusion and was recently transferred from acute care to an inpatient rehabilitation facility. The OTR conducted an initial evaluation revealing that the client has very limited cervical ROM with ongoing UE weakness, pain, paresthesias, and difficulty performing basic ADLs. She is currently unable to perform IADLs. Which of the following would be the MOST APPROPRIATE INITIAL intervention approach? a. focus on restoration of full ROM and strength for cervical spine and UEs b. focus on adaptive strategies to facilitate independence with dressing, bathing, grooming, and hygiene c. focus on facilitating independence with IADLs d. Focus on returning to grad school

b. focus on adaptive strategies to facilitate independence with dressing, bathing, grooming, and hygiene This adaptation approach gets the client back to engaging in ADLS. At this time ROM and strengthening activities would be contraindicated. IADLs and return to school aren't to be addressed until after ADLs.

Several adolescents with behavior problems attend an after-school program in a mental health outpatient program. They work at an egocentric-cooperative level in a group dealing with issues related to peer pressure. The participants would be most likely to: a. actively taking on roles such as energizer, coordinator, or opinion giver b. focusing on the group tasks rather than the feelings of the participants c. making decisions with minimal to no supervision from the group leader d. performing group skills consistent with the developmental level of 15 to 18 years of age

b. focusing on the group tasks rather than the feelings of the participants the egocentric-cooperative group tends to focus on the tasks to be completed with little attention devoted to the feelings of the participants.

An OTR in the school system is implementing a transition plan for a group of developmentally disabled 17-year old students. The long-term goals for the students are gainful employment and supervised independent living in a group home. What activity BEST addresses the long-term goals related to transition for these students? a. role playing ordering food in the classroom b. going out for lunch in a fast-food restaurant c. ordering take-out lunch by phone d. identifying lunch items from a picture menu in the classroom.

b. going out for lunch in a fast-food restaurant This addresses community participation and inclusion which are two major principles of transition. Role playing is good, but it does not allow for the practice of skills in the real environment. Ordering take-out and identifying lunch items from a picture menu are interventions that missing the community integration component.

An OT implements a transitional program for a high school student with a history of numerous school-related failures. Which is the most important principle of intervention for the therapist to use with this student? a. utilize activities that are typically at the developmental level of a middle school student to ensure successful completion b. grade an activity of interest into achievable steps to facilitate successful completion c. introduce several activities during each session and change them frequently to decrease boredom d. terminate the activity during a treatment session when there is difficulty with activity completion to eliminate frustration

b. grade an activity of interest into achievable steps to facilitate successful completion

An OTR is working with a child with oral hypersensitivity. The child has difficulty with food tastes and gags frequently. The child refuses food at mealtimes and only has three preferred foods. What approach or intervention is MOST beneficial for the OTR to use to reduce oral hypersensitivity and decrease food selectivity? a. jaw strengthening and repetitive chewing activities to build oral motor skills b. gradual oral sensory exploration allowing the child to explore new foods using play c. adaptive positioning to provide a firm BOS to the trunk and feet during feeding d. high-dose vitamin supplements to address concerns for nutritional deficiencies.

b. gradual oral sensory exploration allowing the child to explore new foods using play A play activity with food provides context for greater motivation, control, and engagement in the feeding process. Oral motor skills was not the focus so jaw strengthening is not indicated. Adaptive positioning to increase BOS is for children with postural instability and neuromuscular impairments.

An OTR is working with a 12-year-old boy who will not play on the swings or slides at school. He also has difficulty climbing up the second-floor staircase at school and cannot sit in the bleachers. Which choice BEST depicts this child's area of concern? a. tactile defensiveness b. gravitational insecurity c. dyspraxia d. auditory processing problems

b. gravitational insecurity This is a form of over-responsiveness to vestibular sensations. Tactile defensiveness is the dislike of certain textures or sensations. Children with dyspraxia will appear clumsy.

an adult diagnosed with bipolar disorder has been taking lithium for 5 years. Prior to a weekly occupational therapy vocational planning group, the client reports noticeable functional changes since the last group session. In describing these changes, the client reports symptoms that may be indicative of a possible lithium overdose. With the client's permission, the therapist contacts the psychiatrist to describe the client's concerns. Which symptom would most likely be reported as indicative of this problem? a. reduction in mood swing b. gross hand tremors c. decreased velocity of speech d. fine hand tremors

b. gross hand tremors Reduced mood swings and decreased speech velocity are desired effects of taking lithium to decrease mania. Fine hand tremors are a common side effect and can be controlled by taking the medication propranolol. They usually don't represent an overdose on meds.

A cooking group meets for 1.5 hrs each week at a partial hospitalization program. During the group, members do not smoke, they wait for everyone to be served before eating and they clean up after the meal. When reporting these observations, which of the following is the most accurate statement for the therapist to make? a. the group protocol is clear b. group norms are being followed c. group sanctions are effective d. a diversity of group roles is evident

b. group norms are being followed sanctions are implemented only in a group if members behaviors fall outside of the group's norms and are considered deviant. The scenario does not provide sufficient information to determine the group member's roles.

an individual with a BMI of 35 is joining a community-based wellness program conducted by an OT. When formulating an individualized wellness plan, which condition should the OT take into consideration as an increased risk for this person? a. hypothermia during exertion b. hyperthermia during exertion c. rapid weight loss during the initial weeks d. increased anxiety and depression

b. hyperthermia during exertion an individualized wellness program should decrease anxiety and depression, not increase them

a patient who incurred a right CVA asks for a bottle of water to drink. the OT gives the patient a bottle of water but the patient is unable to open it. The therapist provides instruction on opening the bottle but the patient remains unable to complete the task. After the intervention session, the OT observes the patient independently open the bottle and drink from it. Which deficit is the most relevant for the OT to further eval? a. anosognosia b. ideomotor apraxia c. unilateral neglect d. asomatognosia

b. ideomotor apraxia with ideomotor apraxia, a patient cannot perform a task upon direction but can do the task on his/her own. Anosognosia is a more severe form of neglect demonstrated by a lack of awareness and denial of one's severity of paralysis. Asomatognosia is a body scheme disorder resulting in diminished awareness of body structure and failure to recognize body parts as one's own.

During a wheelchair evaluation, an individual with limited functional mobility ecxpresses concern about the ability to continue voulunteer work at a local church. The church's doorways are 31 inches wide. The client knows that 32 inches is the minimum width recommended for wheelchair access. Which recommendation should the OT make to the client to most effectively address the client's concerns and functional mobility needs? a. have the church widen it's doorways to comply with ADA requirements b. order a wheelchair with wraparound arm rests c. have the client explore alternative volunteer activities in accessible locations d. order a customized narrow adult wheelchair

b. order a wheelchair with wraparound armrests. wraparound armrests are often called space saver armrests. they reduce the overall width of a wheelchair by 1 in.

An individual who successfully completed an inpatient drug rehab program returns to the setting to thank staff members for their assistance with recovery. The individual offers to share personal insights and experiences with the members of the OT values clarification group scheduled for the next hour. After thanking the individual, which is the best response for the OT to make in response to the offer? a. welcome the individual as a guest to the group to share personal perspectives on recovery and inspire group members b. inform the individual that former clients cannot visit the group due to the need to maintain member confidentiality c. ask the individual to prepare a presentation on how values clarification aided in recovery for the next scheduled group session d. Tell the individual that the therapist will discuss this offer with the group and afterwards inform the individual regarding the group decision.

b. inform the individual that former clients cannot visit the group due to the need to maintain member confidentiality confidentiality of all members of a group on an inpatient unit must be preserved at all times.

An OT designs an activity group for 12 adults with multiple physical disabilities and or illnesses who attend a day hospital family respite program. Which method is most effective for the therapist to use to determine the members' needs upon which to base the program? a. interview family caregivers b. interview each member c. interview the day hospital staff d. research the program's population characteristics

b. interview each member

an individual is newly admitted to an acute inpatient psych hospital. The OT observes that the patient is able to follow the unit routines and construct a simple craft project by following written directions with diagrams. However, the patient is not able to complete the project if the instructions are missing. According to Allen's cognitive Disability FOR, which level is most accurate for the therapist to document as descriptive of the individual's functional level? a. level 1 b. level 4 c. level 3 d. level 5

b. level 4 at level 4, individuals require visual cues to complete tasks. Individuals at levels 2 and 3 cannot follow written directions to complete a task. Individuals at level 5 can complete a simple craft without written directions.

An individual recovering from a flexor tendon repair surgery is 2 days s/p. The surgeon refers the client to OT with a prescription to use the Kleinert protocol to guide intervention. the client is now 7 weeks s/p. Which are the most appropriate intervention activities for the OT to use with this client a. home management activities such as doing laundry b. light ADLs such as grooming c. strengthening exercises using a TheraBand d. passive exercises using a dynamic splint

b. light ADLs such as grooming According to the Kleinert protocol, light ADLs are introduced 6-8 wks post-op. Strengthening activities and heavier activities are 8-12 weeks post-op. The use of a dynamic splint is indicated immediately and up to 4 weeks post-op.

a client is being discharged after recovery from hip replacement surgery to live at home alone. Which is the most important equipment for the therapist to review with the client prior to discharge? a. rolling walker b. long-handled reacher c. a bedside commode d. an emergency call system

b. long-handled reacher this client will have hip precautions including no flexion. A long handled reacher will help the client retrieve items from the ground or may even help him dress.

an old adult with RA is provided with a functional hand splint to help prevent deformity. The OT also advises the individual about actions that can be taken to help prevent deformity. Which of the following is most important for the therapist to advise the individual to do? a. avoid fatigue b. maintain AROM c. increased muscle strength d. do PROM exercises

b. maintain AROM PROM and resistive exercise to increase strength are contraindicated for this population. Avoiding fatigue is helpful for energy conservation but does not prevent joint deformity.

a home care hospice OT works with a client with end-stage non-Hodgkin's lymphoma. The client is very knowledgeable about the illness and has been active in all aspects of its treatment. The client has requested activity ideas to fill the hours while family members are at work and school. Which activity is best for the OT to suggest to this client? a. complete a series of progressive resistive exercises to maintain UE strength and endurance b. make personalized memory scrapbooks for each member c. research alternative and complementary medicine approaches on the internet d. prepare an entrée and dessert for the family's evening meal

b. make personalized scrap books for each member one of the main goals of hospice care is to encourage positive life review and support the sharing of the legacy that each person lives. Researching alternative and complementary healthcare can be beneficial for a person with a deadly illness, but does not address the need of the person in hospice to have closure with significant others.

repetitionsa carpenter complains of tingling of the left thumb, index, and middle finger; weakened grasp; and night pain secondary to carpal tunnel syndrome. The left thenar eminence appears smaller and more flattened compared to the right thenar eminence. OT collaborates with the client to develop an intervention plan. Which recommendation is best to include in this plan? a. wrapping wrists with elastic bandages to provide support b. modification of techniques used to hold a hammer c. application of hot packs upon walking to decrease pain d. performance of wrist flexion and extension exercises with progressively increasing repititions

b. modification of techniques used to hold a hammer Carpal tunnel includes sensory and motor deficits associated with median nerve compression. Elastic wraps are not supportive enough. Administrating a pam such as heat, should be done with supervision from OT.

an individual recovering from a head trauma exhibits a motor pattern indicative of being influenced by the symmetrical tonic neck reflex. Which is most likely for the OT to observe the client having difficulty during functional mobility? a. moving both arms to midline when supine b. moving from lying supine to sitting c. flexing the head from the supine position d. extending the head from prone position

b. moving from lying supine to sitting moving from lying to sitting is initiated by flexion of the neck. The presence of STNR causes flexion to result in increased hip extension, making it difficult to assume a sitting position. STNR can decrease the ability to bring both arms to midline when supine.

a non-English speaking family attends a discharge planning session. The assigned OT does not share the language of the family. Which action should the therapist take first? a. make a referral for a home-care therapist b. obtain a translator to communicate with the family during the session c. attempt to communicate with the family through non-verbal communication d. consult with the case manager to develop a discharge plan

b. obtain a translator to communicate with the family during the session

An OT works with an individual recovering from a TBI in a rehab hospital. The therapist uses a transfer of training approach to help the patient develop and carry out a daily schedule of activities upon the patient's return home. What is the most effective activity for the therapist to use during an intervention session with this client? a. preparation of a simple meal b. organization of a list of daily activities c. composition of a shopping list d. completion of an interest checklist

b. organization of a list of daily activities a transfer training approach is a restorative approach that focuses on restoration of components to increase skill. It is deficit specific and utilizes tabletop and computer activities as treatment modalities. According to this approach, the activity of organizing a list of daily activities will help with the ability to formulate a schedule in one's home environment. The other activities are discrete activities that would develop skills related to the performance of activities, but do not address the skills needed to schedule the multiple activities in a typical day.

An OT conducts a communication skills group in a wellness program for a large accounting company. In this mature level group, what should the therapist do? a. help to develop the group norms b. participate as a member c. actively resolve group conflicts d. maintain a leader role

b. participate as a member In a mature group, the therapist participates at the level of a member and does not serve as a group leader except in special circumstances such as a member becoming destructive to the group process. The therapist does not usually participate in conflict resolution except to facilitate the members' participation in extreme situations.

an individual diagnosed with schizophrenia. undifferentiated type, is referred to partial hospitalization program. During the initial eval interview, the OT observes that the client answers each question by consistently returning to the focus of the first question. Each time the therapist introduces a new topic to discuss in the interview, the client ignores this topic and returns to the original topic of focus. When documenting the client's behavior, which of the following is most accurate for the therapist to report the client is demonstrating? a. thought blocking b. perseveration c. obsessive thinking d. poverty of speech

b. perseveration perseveration is a persistent focus on a previous topic or behavior after a new topic or behavior is introduced. Thought blocking is the interruption of a thought process before it is carried to completion. Obsessive thinking involves the persistence of an illogical thought.

The OTR works with a 19-month-old child with significant visual and motor impairments in an early intervention playgroup. Which activity is MOST BENEFICIAL to increase the child's participation in the playgroup? a. learning to play an instrument b. playing in a sand/water table c. listening to stories at circle time d. participating in a puppet theather

b. playing in a sand/water table Children with visual and motor impairments benefit from a variety of movement exercises to develop body awareness and directionality. Learning instruments requires fine motor skills. Stories only stimulates the auditory senses and is a passive activity. Puppets may be too visually and motor challenging for this child.

The population of an urban homeless shelter includes individuals with histories of chronic alcohol abuse who are at risk for developing peripheral neuropathy. The OT consulting at this shelter monitors the residents' status to ensure early detection of this problem. Which is the most important observed status change for the therapist to report? a. progressive deterioration in visual acuity b. progressive deterioration of sensorimotor functions of the LEs c. rapid onset of intention tremors d. rapid loss of sensorimotor functions of the facial and neck muscles

b. progressive deterioration of sensorimotor functions of the LEs Peripheral neuropathy is a syndrome of sensory, motor, reflex, and vasomotor symptoms. It does not result in a rapid loss of function, deterioration of visual acuity, or intention tremors.

During a classroom screening, an 8 y/o is observed holding a pencil with a tight grip. The student appears to rely heavily on visual cues to assist during both find and gross motor tasks. During gross motor activities, the student moves in an uncoordinated manner. The OT uses a sensory integrative FOR to interpret evaluation data. Which impairment should the therapist document as needing further evaluation? a. vestibular processing dysfunction b. proprioceptive system dysfunction c. hypo-responsive tactile system d. hyper-responsive tactile system

b. proprioceptive system dysfunction the tight pencil group, incoordination, and use of visual cues could indicate proprioceptive deficits. Vision is often used to compensate for proprioceptive deficits. Hyper-responsiveness would be demonstrated by tactile-defensiveness, and hypo-responsiveness would be demonstrated by tactile seeking through excessive touch.

An individual is transferred from an acute care hospital to a sub-acute rehab unit in a long term care facility. The patient incurred a left CVA in the middle cerebral artery 1 week ago. The individual is referred to OT. The referral states that the patient has right hemiplegia and a subluxed right shoulder. the OT meets with the nursing staff that will be providing primary care to the patient at the patient's bedside. The OT recommends that the direct care staff position the patient in left sidelying. Which is the best bed position for the therapist to recommend for placement of the patient's right arm? a. in 90* of humeral abduction and internally rotated b. protracted with arm forward on a pillow and the elbow extended or slightly flexed c. on the person's side adducted an internally rotated d. in 90* of abduction of the humerus with neutral rotation

b. protracted with arm forward on a pillow and the elbow extended or slightly flexed the best position of the UE for sleeping is to place the affected arm on a pillow in the position which ensures that the shoulder is approximated and that the extremity is well supported. Excess abduction can cause the joint capsule to loosen and reduce the stability of the humeral head in the glenoid fossa.

The OTR has been working with the speech-language pathologist (SLP) and team to support an augmentative communication system for a child on the autism spectrum. The child is effectively using a tablet-based system with core vocabulary integrated into the system. The child is able to communicate needs and wants using the system. However, the child has broken the glass on the tablet and it has been replaced twice. What is the BEST solution the OTR might recommend? a. attach the tablet to a protective notebook b. provide a child-sized protective case c. suggest a different augmentative communication device that wont break d. provide the child the device only when communication is anticipated

b. provide a child-sized protective case The child is using this tablet successfully, so changing the tablet is not a good option. Limiting a child's ability to communicate is unethical. The durable case is the best option.

An OT works with a child with pervasive developmental disabilities to improve self-care skills. In teaching the child to brush teeth, the therapist places the toothbrush in the child's hand and guides it to the mouth. To help the child learn to complete the activity, the therapist uses the somatosensory system. Which of the following is the most effective for the therapist to use next during interventions with this child? a. tell the child to brush up and down b. provide hand over hand assistance to brush the child's teeth c. touch the child's hand to prompt hand to mouth movements d. instruct the child to follow a pictorial sequence card depicting tooth-brushing

b. provide hand over hand assistance to brush the child's teeth by doing this, the OT is providing tactile, proprioceptive, and movement stimuli to cue the child. Providing auditory, tactile, or visual input does not provide sufficient input for the child to learn the task.

The family of a 2 y/o in a spica cast asks the OT to modify the child's car seat. The child cannot fit safely in the car seat due to the cast. Which is the best action for the OT to take in response to the request? a. pad the area between the car seat and the child's back with a pillow to accommodate for the lack of hip flexion b. recommend the family purchase a car seat designed for a child with a spica cast c. cut down the sides of the car seat to allow the cast to hand out of the sides of the car seat d. tell the family to use the current car seat and tighten up the straps to hold in the child

b. recommend the family purchase a car seat designed for a child with a spica cast The child needs a car seat that has been crash tested for children with spica casts. Padding the area would compromise the integrity and make the seat unsafe.

The parents of a 5 y/o with ADHD express difficulty managing the child's aggressive behavior towards other siblings. Which is the most effective strategy for the OT to recommend to the parents? a. allow the child to vent aggressive feelings on a stuffed animal or doll b. redirect the child's energy into acceptable and safe play activities c. provide consistent punishment for aggressive behavior d. send the child to stay with a family member or close friend for an extended "time out"

b. redirect the child's energy into acceptable and safe play activities this can be an effective strategy to manage aggressive behaviors. It would also be appropriate to have the parents observe and record the precipitants to these behaviors to determine potential environmental modifications.

following the performance of a HEP prescribed 1 wk ago, an individual with bilateral upper extremity muscle weakness reports experiencing pain in both shoulders and elbows. The pain is consistent for up to 8 hrs. The individual's OT is on vacation for 2 weeks and a recently hired entry-level therapist has been assigned to cover the vacationing therapist's caseload. Which is the most appropriate action for the covering therapist to take in response to this individual's reporting symptoms? a. stop exercising completely until the primary T returns and can re-eval the persons status. b. reduce the intensity of the exercise by 50% and reassess the person during the next intervention session c. continue with the current exercise program to develop tolerance. d. advise the individual to take a pain relief medication 30 min prior to exercising.

b. reduce the intensity of the exercise by 50% and reassess the person during the next intervention session.

An OT meets with a patient with fibromyalgia who has had difficulty meeting intervention goals. The patient complains of being hurt and frustrated in attempts to resolve pain and fatigue issues. Which is the most effective technique for the therapist to use to help the patient increase insight into this situation? a. offer a variety of options for pain management b. reflect the patient's verbal expressions back to the patient c. refer the patient to a specialized pain management center d. repeat the patient's exact words back to the patient

b. reflect the patient's verbal expressions back to the patient this involves expressing the feeling behind the patient's words and is an effective technique to facilitate self-reflection and develop insight. Repeating the exact words or parroting is not effective because this means merely stating the words without focusing on the emotions behind the words. Offering options for pain treatment and a referral to a pain management center can be helpful/ but do not address the question's stated focus on increasing the client's insight.

A client with arthritis in the dominant hand and wrist has been referred for OT in order to obtain a splint. The client is experiencing extreme pain with use of the hand. Which splint would BEST meet this client's needs? a. ulnar deviation splint b. resting hand splint c. serial static splint d. dynamic serial splint

b. resting hand splint This will decrease inflammation and pain, especially at night. Ulnar deviation splints are used when the MCP joints are painful and misaligned. Serial and dynamic static splints are used to regain ROM.

An OT consults with a home care agency interested in starting a falls prevention program. Which should the OT do first? a. review available evaluation protocols b. review available screening tools c. create a new screening tool d. design an evaluation protocol

b. review available screening tools The home care agency's population will need to be screened to determine who would benefit from a falls prevention program. Creating a new screening tool is not cost-effective because there may be several tools readily available that meet the program's needs.

an OT accepts a position at an adult daycare and respite program for elders with a variety of physical and cognitive disabilities. The therapist only has clinical experience in school-based practice. Which is the most effective way for the therapist to prepare for the professional responsibilities this new position will entail? a. attend caregiver support groups b. review current literature on evidence-based elder care c. review area demographic info on elders with disabilities d. confer with the program's administrative director

b. review current literature on evidence-based elder care a review of literature can provide quality information about evaluation and intervention approaches. Info about demographics is too broad. The program administrator can provide relevant info about the setting's policies but would not be able to provide info on the practice of OT.

an individual prepares for discharge home following rehab for a left CVA. Residual difficulties include fair dynamic balance and decreased proximal UE strength. The individual's stated priority is to be able to ambulate safely to the senior center located in the client's apartment building. Which ambulatory aid would be most effective for the OT to recommend to this client? a. a hemi-walker b. a rolling walker c. a side-stepper walker d. a standard walker

b. rolling walker a rolling walker is indicated for a person who cannot lift a standard walker due to impaired balance or UE weakness. A hemi-walker and side-stepper are indicated for individuals who don't have use of both hands.

a 6 y/o has thumb weakness, noted mostly in poor ability to perform thumb opposition. during eval, which activity will the therapist most likely observe the child having difficulty performing? a. holding a penny on the ulnar side of the hand while moving a nickel from the palm to the tips of the thumb and the index finger b. rotating a pencil 180* c. sliding the fingers up and down a pencil while holding the pencil in the tripod grasp d. moving a ring from pad-to-pad pinch position to the palm.

b. rotating a pencil 180* this is considered simple rotation, because complex rotation is 360*. The child with poor thumb opposition has the most difficulty performing this skill of all the in hand manipulation skills.

an OT observes that a child can open a combination lock, open a lock with a key, and turn a pencil over to erase. In documenting the child's in hand manipulation, which is most accurate for the OT to report that the child can correctly preform? a. finger-to-palm translation b. rotation c. palm-to-finger translation d. shift

b. rotation these activities involve turning or rolling. The translation answers are incorrect because the palm is not used in these activities. Shift is incorrect because the activities are not using linear movements.

An adult who incurred a TBI 3 months ago is referred to a home care agency to receive OT services. The client's referral states that cognition is at a level VII of the Rancho Los Amigos scale and that grasp and shoulder mobility are limited. During the initial interview, the client reports being frustrated by the inability to independently engage in the previously enjoyed and personally meaningful activity of cooking. The client reports frequently losing place when reading recipes and an inability to find things in the kitchen Upon eval, the client exhibits difficulty with the letter cancellation task. Which visual deficit should the OT document as present? a. imagery b. scanning c. cognition d. memory

b. scanning Visual imagery is the process of making a mental picture of info so that it can be remembered. Visual cognition is the ability to mentally manipulate visual info and integrate it with other sensory info. Visual memory is the retrieval and recall of info that has been stored and encoded.

a non-English speaking individual with a recent left sided CVA comes to OT for the first eval session. The assigned therapist cannot communicate verbally with the person. Which action is best for the therapist to take first? a. perform a screening to determine the specific areas to evaluate b. seek out a translator to assist with the eval c. communicate with the individual using non-vernal communication d. eval the individual according to established protocols

b. seek out a translator to assist with the eval This action is consistent with Principle 3: autonomy of the OT code of ethics.

a single parent of 2 school-aged children is employed as a truck driver working the 11 pm-7pm shife. The client was hospitalized for depression following the sudden death of his spouse 1 year ago. The deceased spouse's primary role had been home maintainer. The client was re-hospitalized this past weekend for exacerbation of depression. Which skills are most relevant for the OT to evaluate? a. interpersonal b. self-management c. cognitive d. leisure

b. self-management the individual has experienced major role disruptions. These major life changes require significant self-management skills. Leisure is an area of occupation, not performance skill.

an adult diagnosed with MS over 10 years ago experiences an exacerbation of symptoms. The individual's principle complaint is decreased strength and endurance. The person can ambulate short distances with a cane in the home and uses a wheelchair outside the home. The client asks for suggestions to enable independent home maintenance. Which is the best positioning recommendation for the OT to suggest the person to use during meal prep? a. sitting in the wheelchair with a tray table b. sitting at the kitchen table c. leaning against the counter while standing d. leaning against a tall stool while standing

b. sitting at the kitchen table The avoidance of fatigue is important in the management of MS. Doing meal prep while seated is safer and less fatiguing. A wheelchair is not indicated because it is currently too low level for her.

an individual recovering from a CVA has received extensive motor learning intervention. The client can now transfer a learned motor skill to different contexts. The client also demonstrates the ability to successfully motor problem solve during activities in different contexts. In documenting the client's progress, which stage of motor learning is most accurate for the OT to document the individual has achieved? a. skill acquisition b. skill retention c. practice context d. general context stage

b. skill retention

An ambulatory elder adult with hemiparesis and presbycusis is moving in with an adult child's family. The family hires an OT to provide information to help them maintain the elder's functional ability in the home. Which should the OT recommend? a. remove knobs from the stove when the elder is home alone b. speak directly, clearly, and slowly to the elder c. add bright color strips to the edge of each stair tread d. provide lists of the sequence of routine tasks.

b. speak directly, clearly, and slowly to the elder. Presbycusis is an age-related sensorineural loss that results in decreased hearing. There is nothing in the situation to indicate the need for any other suggestions.

An OT becomes aware of the practice of a colleague who teaches an energy conservation class to people with arthritis. This colleague has been sending the names of class participants to a vendor who sells adaptive equipment. Which action is the best action for the OT to take? a. ignore the situation for it doesn't harm anyone b. speak to the therapist privately and tell him/her this action is unethical c. advise the therapist to disclose this practice and if he/she refuses report the therapist to the state regulatory board d. report the therapist's unethical behavior to the state regulator board

b. speak to the therapist privately and tell him/her this action is unethical The OT should let the colleague have a change to self-correct the behavior. Reporting the therapist at this time is over-reactive and would result in little action for these boards. Providing a vendor with names of potential clients may be questionable ethically, but it is not illegal.

an individual with RA has developed several boutonniere deformities. Which of the following is the most accurate description for the OT to include in documentation of the individual's presenting signs? a. hyperextension of the PIP joint and flexion of the DIP joint b. ulnar deviation and subluxation of the MCP joints c. flexion of the PIP joint and hyperextension of the DIP joint d. Herberden's nodes at the DIP joints and Bouchard's nodes at the PIP joints

c. flexion of the PIP joint and hyperextension of the DIP joint deformity resulting in the hyperextension of the PIP joint and flexion of the DIP joint is called the swan neck deformity. Herberden's and Bouchard's nodes are types of bone spurs that result from OA.

A client recently lost significant function in the dominant hand following extensor tendon injury and presents with boutonniere deformity. Which is the BEST immediate course of action for the OTR to take in response to this client's hand function and impairment? a. dorsal blocking hand splint b. splint PIP in full extension c. splint PIP in full palm flexion d. resting position hand splint

b. splint PIP in full extension Boutonniere deformity results from injury in zone III of the hand and occurs when the common extensor tendon is ruptured resulting in flexion of the PIP. Dosal blocking hand splints are used for a flexor tendon repair. Splinting the PIP in full palm flexion would promote further deformity and loss of function. Resting splints are used to support the natural position of the hand will not affect or correct the deformity.

A patient is recovering from a right total hip replacement (posterolateral incision, cementless fixation). Which is the best type of bed-to-wheelchair transfer for the OT to teach the patient to use? a. stand-pivot transfer to the surgical side b. stand-pivot transfer to the non-surgical side c. lateral slide transfer using a transfer board d. squat-pivot transfer to the surgical side

b. stand-pivot transfer to the non-surgical side with a posterolateral incision, excessive flexion & extension, IR, and adduction past neutral are contraindicated.

an elementary school student with hypotonic CP receives school-based OT to improve fine motor skills. The child holds a thick marker with a static tripod grasp and holds a #2 pencil with a gross grasp. The OT previses the intervention plan based on the child's attained skills. Which grasp would be best for the therapist to address next in the revised OT intervention plan? a. dynamic tripod with the thick marker b. static tripod with a pencil c. lateral pinch with a thick marker d. dynamic tripod with a pencil

b. static tripod with the pencil the best way to progressively grade the grasp is to work on static tripod with a thinner object before going to a dynamic tripod. A lateral pinch is not an effective grasp for a marker.

A 4 month old with arthrogryposis remains in position when placed and shows little spontaneous movement. The OT implements intervention to work on rolling. Which positional changes should the therapist include in the intervention session? a. prone to supine b. supine to side lying c. prone to side lying d. supine to prone

b. supine to side lying when children exhibit developmental delay, the OT should begin intervention by working on the first skill that typically occurs which is rolling from supine to side lying. Rolling from prone to supine is the next stage and the other answers occur later.

An OT wants to develop an after-school program for obese adolescents who have diabetes or who are at risk for developing diabetes. Which program development action should the therapist take first? a. obtain statistical data about adolescent obesity to support the need for the program to the school administrators b. survey the adolescents about their occupational performance to determine program focus c. survey the OT practitioners about services they provide to obese adolescents to determine group focus d. review professional literature to obtain ideas for activities to include in the program

b. survey adolescents about their occupational performance to determine program focus The development of new services would require a needs assessment to determine the necessity and focus of services. While statistics can support the program, the target population is more relevant.

An OT accepts a job at an after-school program. The program provides services for adolescents at risk for mental health problems due to their history of being victims of abuse. The therapist decides that an activity group to elicit the adolescents' thoughts and feelings in safe atmosphere would be instrumental to their recovery. The therapist designs a/an: a. instrumental group b. task-oriented group c. topical group d. thematic group

b. task-oriented group the purpose of this type of group is to increase member's awareness of feelings, thoughts, needs, values, and behaviors through the process of choosing, planning, and implementing group activity. A topical group is a verbal group that focuses on the discussion of activities members are interested in outside of group. An instrumental group is designed for individuals with chronic disabilities who are functioning at their highest level with no anticipation of improvement. A thematic group assists members in acquiring knowledge or skills to perform a specific set of skills independently.

an adult incurred severe lacerations to the extrinsic flexor tendons in Zone V of the hand. The tendons were surgically repaired and have healed. The surgeon's most recent orders have upgraded the patient to active ROM w/out restrictions. The OT is preparing for discharge and performs a final eval, which reveals limitations in active finger flexion 10-20* of full active ROM of the MCP, PIP, and DIP joints of all fingers. The therapist collaborates with the patient to prepare a home program. Which intervention is most important for the therapist to include in this program? a. use of a resting splint b. tendon gliding exercises c. weight-bearing activities d. home management tasks

b. tendon gliding exercises tendon gliding exercises help to prevent adhesions of the tendons in the healing process.

a 14 y/o with spastic quadriplegia CP and mild intellectual disability begins high school. The OT determines that the adolescent's academic needs are being adequately addressed by the use of the equipment and strategies that the adolescent had acquired during the middle school years. Which recommendation is the best for the therapist to make for the IEP team? a. adaptation of the middle school strategies and equipment to the adolescent's high school classes b. the development of a transition plan with the adolescent for desired post-school life c. the development of peer relationships through engagement in extracurricular activities d. a referral to the local center for independent living for community-based activities

b. the IEP must include transitional planning to help a student identify desired post-school goals and develop a plan for achieving these goals in adult life. This plan must begin at 14 (or younger if indicated) with transition services implemented at 16 (or younger if indicated).

An OTR has a client who needs a seating option that provides the most access to play and table-top tasks. The child requires a stable base to sit on, but does not need back support to participate. The OTR has found the cube chair does not fully slide under the table, so the child cannot access materials. What might be the BEST option for the OTR to consider? a. corner chair b. therapy bench c. Rifton modular wooden chair with pommel d. Rocker's Pediatric chill out chair

b. therapy bench These are used to work on trunk control while working on table-top tasks. Using a therapy bench allows the child to use the full ROM of his upper body. A modular chair may be too specific and individualized to the client in this setting. A rocker chair is designed for individuals that are not in their wheelchair or adaptive seating, and it would not fit under the table.

A 6-year-old client has a medical diagnosis of autism and is working on playing cooperatively with peers. His play tends to be self-directed; however, he has begun to enter play with peers during preferred tasks, but tends to disrupt the play group. What is the BEST goal to increase cooperative play with peers and support playfulness once engaged in a play group? a. the child will maintain organized play sessions with peers successfully 100% of the time for a 20 min play session b. the child will enter a play session without disrupting play with his peers at least 2 times during a 20 min play session c. the child will play with peers for 5 min during an organized play group determined by the therapist d. during a therapist-supported play group, the child will play with peers as demonstrated by 2 exchanges of toys with peers during a 20 min play session

b. the child will enter a play session without disrupting play with his peers at least 2 times during a 20 min play session This outlines a goal that supports the child's area of challenge, entering play without disruption. Its unrealistic to expect most children to succeed 100% of the time. Asking a child to play for 5 minutes focuses on length of time, not entering a play group. Focusing on therapist-directed play is not as effective as sharing during play.

an OT plans individual and group activities for a child with oppositional defiant disorder. Which is most important for the therapist to address during group activities? a. the child's willingness to take on a variety of group roles b. the child's ability to attend to and complete a task c. the child's distorted body image d. the child's self-regulation of energy and activity levels.

b. the child's ability to attend to and complete a task children with this disorder tend to have difficulties with impulse control, attention span, and short-term memory. They exhibit argumentative and resentful behaviors.

an OT documents an individual's performance during a stress management group. According to established documentation standards, which statement is best for the therapist to include in the daily progress note. a. the client completed the checklist of stressors in an appropriate amount of time b. the client was able to identify 3 current life stressors c. the client appeared upset and tense throughout the session d. the client stated walking is a relaxing and enjoyable activity.

b. the client was able to identify 3 current life stressors documentation must be specific, measurable, and behavioral. In this scenario, it must provide information that is objective and related to the individual's performance in group. Timely completion of an assessment does not include sufficient information about the client's performance. The identification of an enjoyable and relaxing activity can be relevant, but this statement does not directly address the person's performance in group.

An OT works with an individual recovering from a TBI who demonstrates behaviors consistent with Rancho level VII. The client is a resident in a transitional living program. Which is the most important focus for the therapist to include in the client's intervention plan? a. the provision of a high degree of environmental structure to decrease confusion and ensure safety b. the development of strategies to accurately and safely complete IADLs with min assist. c. the development of adaptive techniques to accurately complete BADL with mod assist d. the provision of max assist to accurately and safely complete IADLs

b. the development of strategies to accurately and safely complete IADLs with min assist. A person at this Rancho level can complete highly familiar tasks such as BADL with minimal assistance.

an individual recently discharged from an acute psychiatric unit interviews for a position in a transitional employment program. The person answers the interviewing therapist's questions in a direct yet subdued manner and rarely looks at the therapist. Which is the most accurate for the therapist to document in the summary of the interview? a. the individual should have medications evaluated before starting the TEP b. the individual demonstrated eye contact c. the individual exhibited poor interaction skills d. the individual appeared depressed

b. the individual demonstrated limited eye contact this is the only factual answer. The other answers are based upon conjecture.

an OT observes an 18 month old child is not able to creep more than a few steps. When the child looks up, both hips and knees flex, and the child ends up W sitting with both arms extended and propped forward. When documenting this observation, which is most accurate for the OT to report the child is demonstrating? a. typical development of locomotion skills b. the influence of the symmetrical tonic neck reflex (STNR) resulting in delayed gross motor skills c. An obligatory asymmetrical tonic neck reflex (ATNR) resulting in delayed gross motor skills d. an intact tonic labyrinth reflex which facilitates balance responses

b. the influence of the STNR resulting in delayed gross motor skills A persistent STNR would cause extension of the UEs. ATNR would cause the child to collaspse on 1 side. Tonic labyrinth reflex which facilitates balance response.

an assisted-living facility has received a grant to purchase capital items that can be used to increase the independence of its residents. Which is best for the OT consultant to recommend for the use of grant funds? a. the purchase of AE that residents can borrow when needed b. the purchase of a computer-based driver rehab program c. the hiring of more personal care attendants d. the construction of an ADL training apartment

b. the purchase of a computer-based driver rehab program a capital expense is an item that is above a fixed amount, typically $1,000. Since the residents of an assisted-living facility tend to be elderly, the need to eval driving abilities and provide appropriate interventions to ensure safe driving is an important focus. AE and staff are considered direct expenses. An ADL house meets the criteria of a capital expense but would not be needed because the residents live in their own apartments which have kitchens.

During an early intervention planning meeting, an OT explains the results of a play assessment to the parents of an 18 mo old toddler with multiple DDs. The child has been assessed as delayed by 6-8 mo in all developmental parameters. Which intervention approach is most effective for the therapist to recommend using to first develop the toddler's play skills? a. the use of toys that encourage creative and imaginative play b. the use of toys that are visually and auditorily stimulating c. participation in small parallel play groups with others d. engagement in activities that use sensorimotor skills prerequisite to play.

b. the use of toys that are visually and auditorily stimulating creative play doesn't occur until 4-7 years, and parallel play isn't appropriate yet.

A 14-year-old client is working with a OTR to expand his social engagement and participation with peers. He presents with challenges connecting with peers and making friends. The client has indicated a personal goal to increase social engagement and friendships. What is the BEST way for the OTR to gain a perspective on the client's needs? a. use a parent report to determine targeted areas to improve social engagement b. use a client report to determine the youth's feelings and experiences in contexts c. use parent report to determine the cognitive and social engagement skills d. use standardized testing to determine targeted areas to improve cognitive skills and work preferences

b. use a client report to determine the youth's feelings and experiences in contexts There is no standardized testing for social relationships and a parent report may give skewed information. It is best practice to take into consideration the clients wants, needs, and feelings.

An OT provides caregiver education to the spouse of a client with Alzheimer's disease and a secondary diagnosis of a left CVA. The client is dependent upon a wheelchair for mobility and has been deemed cognitively incompetent. The spouse reports that the client becomes restless at meal times, consistently, undoes the lap belt, and tries to get up from the wheelchair. The spouse reports that the need to constantly say "sit down" is personally exhausting and often increases the client's agitation. Frequently, neither one eats dinner. Which is the most effective recommendation for the therapist to make to the spouse? a. allow the spouse to get up when restless and provide dinner to the client at a later time b. use a wheelchair lap tray to serve several smaller meals to the client in intervals throughout the day c. hire a home care attendant to assist the client at meal times and provide some respite to the spouse. d, use a wheelchair lap tray to serve the client large meals at breakfast, lunch, and dinner.

b. use a wheelchair lap tray to serve several smaller meals to the client in intervals throughout the day A lap tray is a permissible and reasonable restraint if it is necessary to maintain the individuals safety. A lap tray with food on it can provide physical and sensory cues necessary to keep the client seated for a time that is sufficient for eating a smaller meal. Hiring a home care attendant can relieve the spouse's caregiver stress, but it does not address the client's risk of falling when attempting to get up.

a patient with a complete SCI at the C5 level completed inpatient rehab which concentrated on increasing independence in ADLs. In the d/c documentation, which is most likely for the OT to state that the client is able to do? a. button buttons using a button hook b. use mobile arm supports for feeding c. tie shoes using thick shoelaces d. brush teeth using a tenodesis splint

b. use mobile arm supports for feeding mobile arm supports are likely to be able to be used by a person with this level of injury. The ability to complete grooming tasks with a tenodesis splint and buttoning with a button hook are likely skills for a person with a SCI at C6. A person with C8 level SCI would likely be able to tie shoes.

An individual who is scheduled for a right total hip arthroplasty is referred to OT. A posterolateral surgical approach will be used. The client has expressive aphasia resulting from a CVA incurred 4 years ago. Medicare is the client's only health care benefit. The client states that private payment for any health care services or equipment is not possible at this time. Following surgery, which is the most appropriate bed-positioning intervention for the OT to recommend? a. side-lying with LEs adducted b. use of an abductor pillow between the LEs c. use of hospital bed to elevate the LEs to 90* d. change of position from supine to prone every 2 hrs.

b. use of an abductor wedge between the LEs the abductor pillow will prevent adduction of the operated hip, which is an important postsurgery precaution. All other positions are contraindicated or cautioned.

an elder expresses concerns about the ability to perform daily tasks. The OT assess that the individual has somatosensory deficits consistent with the normal aging process. The therapist recommends AE to assist with task performance. Which AE should the therapist recommend the person use during meal prep and feeding? a. utensils with narrow, smooth grips b. utensils with wide, textured grips c. a rocker knife d. dycem pads

b. utensils with wide, textured grips these provide augmented sensory feedback and will be easier to grasp than smooth, thin grips. A rocker knife and dycem would not address these deficits. A rocker knife is suitable for one-handed cutting. Dycem pads are used to provide stability to an object.

Following an acute hospitalization for the medical management of a CVA, an individual receives home-based OT services. The OT is working on dressing skills with the patient. During one session, the therapist has the individual dress in the bedroom and during the next session the therapist has the client dress in the bathroom. During the following session, the therapist has the client don and doff a sweater and coat in the living room. Which motor learning technique is the therapist using? a. variable activities b. variable conditions c. repetition d. generalization

b. variable conditions this involves the practice and performance of skills in various contexts to improve the transfer of learning and retention of skills. Dressing during three different sessions is not repetitive. Generalization is not a treatment technique, but rather a desired outcome of an intervention.

An OT works on feeding with a toddler who has a hyperactive gag reflex. What should the therapist do to decrease the gag reflex? a. have the child suck through straws of progressively longer lengths b. walk a tongue depressor from the front of the tongue to the back c. quickly ice the childs throat laterally d. have the child blow bubbles

b. walk a tongue depressor from the front of the tongue to the back this can desensitize a hyperactive gag reflex. Sipping on a straw can increase a sucking reflex. Quick icing is a traditional rood technique that theoretically stimulates a muscle group. This technique has no evidence to support its use and should not be used in current practice.

As the result of a trauma 10 months ago, a 13 y/o incurred a unilateral below-elbow amputation. Due to a recent growth spurt, the adolescent is being re-evaluated for a new prosthesis. During the eval, the adolescents angrily tells the OT that a prosthesis is "annoying to have to wear everyday." The adolescent reports not liking the "claw" and as a result often takes off the current prosthesis. However, the adolescent acknowledges that the prosthesis is "sometimes needed" to perform desired bilateral activities. Which is the best action for the OT? a. refer the adolescent to the child psychologist to deal with adjustment to disability b. work on developing unilateral skills for completion of meaningful activities c. refer the adolescent to a prosthetist for a prosthesis with a cosmetic hand d. recommend the adolescent attend a teen amputee support group

b. work on developing unilateral skills for completion of meaningful activities most individuals with unilateral below the elbow amputations are able to achieve functional independence in desired activities using their intact UE for skilled task functions with their residual limn serving as a stabilizer. The adolescents response is normal and does not indicate a need for referral to a psychologist or support group. A cosmetic prothesis requires more muscle function to operate and it does not enhance functional abilities.

a client with chronic schizophrenia attends a transitional employment program. The individuals has a secondary diagnosis of Class 1 heart disease. The OT meets the client and the client's work supervisor to discuss the work activities the client can safely complete. Which of the following most accurately describes client's capacity for work? a. work with minimum limitations b. work with no limitations c. work with reasonable accommodations of frequent rest breaks d. work with reasonable accommodation of no heavy lifting

b. work with no limitations This class requires no limitations on activites.

an OT working in a private clinic receives a referral for a client who incurred a nerve laceration while working as a cable installer and repair person. Upon eval, the therapist determines that the client exhibits maximum motor and sensory losses consistent with radial nerve laceration below the supinator. The OT documents the results of the eval. Which deformity should the therapist note that the client is exhibiting? a. claw hand b. wrist drop c. ape hand d. Saturday night palsy

b. wrist drop The presenting signs of a radial nerve laceration are weakness or paralysis of the extensors in the wrist, MCPs, and thumb with a characteristic wrist drop. Ape hand is the flattening of the thenar eminence indicative of a median nerve palsy. A claw hand is indicative of ulnar nerve palsy. Saturday night palsy denotes a radial nerve palsy that results from a position that compresses the radial nerve.

an OT is working in a SNF. The therapist is documenting current patient progress for the past 14 days and realizes that a required 7-day progress report to the client's insurance carrier had not been documented. Which should the therapist do in response to this omission? a. document the patient's status as of 14 days and backdate the note 1 week b. write a note describing the patient's progress after 7 days of treatment and sign with the current date c. call the insurance company to explain the documentation was lost due to a computer failure d. described the patient's progress after 7 days of treatment and backdate the note 1 week

b. write a note describing the patient's progress after 7 days of treatment and sign with the current date it is important to accurately document the therapy progress in a timely manner. It is possible for a therapist to inadvertently miss a documentation deadline. If this occurs, the therapist should follow the AOTA code of ethics for veracity and document the patient's progress for the first 7 days and date it with the current date. Notes should never be backdated.

An entry level OT implements a therapeutic feeding program in a SNF. During a therapeutic feeding session, and elderly resident with dementia, non-Alzheimers type, becomes upset and cries for his mother. Which should the therapist say in response to the resident's statements? a. remember that you are now in a nursing home and your mother is not here b. you must miss your mother, tell me about her c. remember your mother passed away years ago d. I will tell the nurse that you want your mother contacted

b. you must miss your mother, tell me about her This response validates the person's feelings and provides him with the opportunity to reminisce about a pleasant memory. Reminding the individual that his mother is deceased is not appropriate because it is no longer part of his reality. Telling the person there is potential to talk to her offers false hope and does not validate his feelings in the moment.

A local pharmacy hires an OT to consult on redesigning of the pharmacy's customer service area. The counter should be no higher than how many inches? a. 32 inches b. 34 inches c. 36 inches d. 36 inches

c. 36 inches according to the ADA, the recommendation for maximal height for an accessible service counter is 36 inches.

A OTR is interpreting scores of a developmental test that was administered to a 4-year-old child. The child scored at the 63rd percentile for the child's age and gender group. What can the OTR conclude based on this score? a. the child displays below-average developmental skills compared to similar children b. these scores are sensitive for measuring small changes in the child's development c. 37% of children in the same sample scored higher than this child d. the child has moderate developmental delays compared to the normative sample

c. 37% of children in the same sample scored higher than this child Below average would be less than 50%. It is difficult to assess sensitivity from this scenario, and it is difficult to infer whether the test is a norm-referenced test which compares scores to a normaitive sample.

A religious congregation obtained private funding to build a ramp so that members with disabilities can attend services. The entrance to the congregation's building has 6 steps with a rise of 7 inches each. Which is best for the OT to recommend for the construction of this ramp? a. 42 ft long b. 48 ft long c. 42 ft long with a 5'x5' landing at the ramps midpoint. d. 48 ft long with a 5'x5' landing at the ramps midpoint.

c. 42 ft long with a 5x5 landing at the ramps midpoint.

The administrator of a large rehab hospital reviews the OT staffing schedules. The administrator determines that of the 12 full-time therapists, 6 work full-time in the inpatient department, 2 work full-time in the outpatient department, and 2 divide their hours equally between both departments, 2 divide their hours equally between the administrative/managerial work and direct care provision on the inpatient unit. Which is the most accurate staffing for the administrator to document for the OT inpatient department in the annual report? a. 12 full-time equivalent employees b. 10 full-time equivalent employees c. 8 full-time equivalent employees d. 6 full-time equivalent employees

c. 8 full-time equivalent employees a Full-time equivalent is the amount of work time assigned to 1 full-time staff member in a year. In this scenario, 2 employees have part-time inpatient responsibilities, so they would each be considered .5 FTE. The other 2 only devote half their time to direct patient care, so they are also considered .5 FTE. .5 x 4 = 2 and 2+6=8

An individual recently had a left transfemoral amputation as a result of the complications of diabetes. The client is 2 yrs s/p a right transtibial amputation as a result of the same precipitant. The client has a right prosthesis. The client is referred to OT for preprosthetic interventions for the left residual limb. Which intervention is best for the OT to implement during the first session? a. UE strengthening with emphasis on the biceps b. percussion to the left LE's residual limb c. UE strengthening with emphasis on the tricep d. LE dressing with emphasis on donning and doffing prostheses

c. UE strengthening with emphasis on the tricep Strengthening the UE, especially triceps will facilitate the independence in transfers. Neuromas are nerve endings that adhered to scar tissue. Since neuromas can be very painful, percussion to the left residual limb and donning/doffing of the left prosthesis is contraindicated at this time.

a toddler with spastic quadriplegic CP demonstrates a consistent gag reflex. Which technique should the OT which technique should the OT use to help inhibit the reflex? a. move a spoon from side to side on the tongue b. walk a spoon down the tongue, going proximal to distal with even pressure c. press a spoon down firmly on the center of the tongue d. stroke the tongue in a circular motion with a firm object

c. press a spoon down firmly on the center of the tongue apply pressure from distal to proximal when doing this. Avoid lateral and circular movements, as they can facilitate a gag reflex

An individual who is scheduled for a right total hip arthroplasty is referred to OT. A posterolateral surgical approach will be used. The client has expressive aphasia resulting from a CVA incurred 4 years ago. Medicare is the client's only health care benefit. The client states that private payment for any health care services or equipment is not possible at this time. the client is being discharged home. Which equipment should the OT recommend to ensure safety and independence in toileting? a. a raised toilet seat b. grab bars c. a 3 in 1 commode d. non-skid mats

c. a 3 in 1 commode the client will need to observe hip precautions for several weeks. The 3 in 1 commode will provide the additional height needed. It is also the only item identified that is reimbursable by Medicare.

An OTR is working with a 4-year-old boy with Down syndrome at preschool. His gross motor skills are awkward, he falls frequently, and shows difficulty moving around obstacles. He parallel plays but does not interact with his peers at school. Which activity is MOST EFFECTIVE to improve gross motor and social skills in this child at preschool? a. circle-time songs with teacher in the classroom b. a craft activity at the table with other peers c. a 4 step obstacle course in the classroom d. riding a tricycle at recess time on the playground

c. a 4 step obstacle course in the classroom An obstacle course can consist of various gross motor skills and be completed with peers to promote social participation. Circle-time songs address cognitive and language development. A craft activity focuses on fine motor activity. Riding a tricycle is a solitary activity that only addresses the gross motor skills component.

An OT working in a school system conducts a series of educational workshops for parents of children with attention deficit disorders and sensory processing disorders. The series focuses on principles of SI. Which of the following is most beneficial for the therapist to recommend the parents provide in the home environment? a. a wide range of sensory stimuli including auditory, visual, and tactile to increase awareness and responsivity b. minimal auditory, visual, and tactile sensory stimuli to decrease distractibility and responsivity c. a balance between structure and freedom so the child can direct his/her own actions d. clear directions and structured limits to organize behavior and promote adaptive responses.

c. a balance between structure and freedom so the child can direct his/her own actions a key principle of SI theory is to structure the environment to match the child's capabilities. The design of an environment to increase or decrease distractibility is more consistent with a compensatory remediation approach. Providing structure and limits is more consistent with a behavioral approach

an OT provides recommendations for play activities that a parent with a complete SCI at the C7 level can do with children ages 8 and 10. Which is the best adapted activity for the therapist to recommend? a. an arts and crafts project using a mouthstick brush to paint b. a woodworking project using a universal cuff to hold tools c. a board game using a tenodesis grasp to move game pieces d. a computer game using a typing stick to keyboard

c. a board game using a tenodesis grasp to move game pieces a person with this level of SCI has a tenodesis grasp that can be useful. The other activities are appropriate for individuals with higher SCIs

a child with congenital anomalies has severe developmental delay. The child demonstrates motor and cognitive skills at a 9-month level. Which is the best adaptation for the OT to use during intervention to develop the child's visual and auditory awareness? a. a hand-held rattle of the child's favorite cartoon character b. a wrist bracelet with blinking lights that makes noise when moved c. a button switch that activates a CD player when the switch is pressed d. a communication device that offers selection of "yes" and "no"

c. a button switch that activates a CD player when the switch is pressed The button switch encourages the child to develop the appropriate skill of cause and affect. The rattle and bracelet is appropriate at a 3-6 month level, and the communication device is appropriate for 12-18 months

An OT provides intervention for an individual with a swallowing disorder. To elicit a swallow reflex, the OT provides sensory input to the inferior faucial arches. Which should the therapist use to provide this intervention? a. a tongue depressor b. a moistened cotton swab c. a chilled dental examination mirror d. a warmed metal teaspoon

c. a chilled dental examination mirror the use of cold stimulation of the inferior faucial arches via this item will elicit a swallow reflex while others will not

an individual with mild cognitive deficits takes meds for multiple medical conditions. The OT works with the individual to develop the ability to safely self-administer medications. Which equipment and/or strategy should the therapist train the client to use? a. easy-open caps on the medication bottles b. a chart listing medication dosages and administration times on the refrigerator c. a daily pill holder with the time-labeled slots for each dosage d. family caregiver supervision of medication administration

c. a daily pill holder with the time-labeled slots for each dosage a chart on the fridge is not as useful because the individual cannot take the chart with him or her during daily activities. Easy open caps do not provide organizational structure for cognitive deficits. Supervision could be needed if the person was unable to benefit from organizational structures.

To develop social interaction skills, an OT implements a group program for students with Asperger's Syndrome. Which group is the best for the therapist to include in this program? a. a directive group b. a topical group c. a developmental group d. a task-oriented group

c. a developmental group A developmental group's focus is to teach the social interaction skills needed for group participation in a sequential manner. Individuals with Asperger's often have normal intelligence but significant deficits in social interaction skills. A directive group is for lower functioning clients. A topical group is a discussion group that focuses on activities performed outside of the group. A task-oriented group increases members' awareness of their values, feelings, and ideas as revealed through group activity.

A carpenter incurred a short below-elbow amputation. The client plans to return to work. Which components would be most important for the OT to recommend for the client's prosthesis? a. a fixed-elbow socket and a lightweight Teflon-coated terminal device b. a cable-driven elbow socket and a heavy-duty serrated grip terminal device c. a fixed-elbow socket and a heavy-duty serrated grid terminal device d. a cable-driven elbow socket and lightweight Teflon-coated terminal device.

c. a fixed-elbow socket and a heavy duty serrated grid terminal device An individual with this kind of amputation will require a fixed-elbow socket to provide stability because natural forearm rotation is not possible. A carpenter will need a heavy-duty serrated grid terminal device to hold tools and nails.

OT services are provided to the clients of a psychogeriatric unit in a SNF. An OT presents an in-service on restraint reduction to the unit's direct care staff. Which of the following would the therapist identify as a permissible use of a restraint? a. a bed guardrail to prevent a confused resident from wandering in the evening b. prescribed medication to control a resident's agitated behavior c. a lap board to enhance a resident's self directed functional behavior d. a wheelchair with a lap belt to prevent a person with ataxic gait from falling.

c. a lap board to enhance a resident's self directed functional behavior a restraint is defined as anything that prevents access to the environment or to one's self. A lap board can enhance functional performance and is permissible with the resident's informed consent.

An OTR is working with a 2.5-year-old child in an early intervention program who was extremely premature at 25 weeks' gestation, presenting with upper extremity weakness and poor grasping. The child's mother would like her child to self-feed independently. Currently, the child is not using a spoon and finger feeds cereal using a raking grasp. The meal process is slow and the mother continues to feed the child in order to meet the nutritional needs. What is the MOST appropriate intervention to promote self-feeding independence? a. small diameter or smooth handle spoon to improve child's success and tolerance of food quantity. b. a dycem mat to stabilize a dish to compensate for uncoordinated arm movements c. a light-weight, built-up handle on the spoon or universal cuff placing the hand in a stable position d. a napkin or moist cloth to use if the feeding activity becomes messy

c. a light-weight, built-up handle on the spoon or universal cuff placing the hand in a stable position a small diameter smooth spoon would be more difficult for the child to hold. A dycem mat helps children with neuromuscular deficits, but it doesn't address the poor grasping and UE weakness.

An OT working for a home care agency provides an in-service to new employees on Medicare reimbursement guidelines for DME. Which item would the therapist describe as reimbursable by Medicare? a. a raised toilet seat for a person after a hip replacement b. a reacher for a person with arthritis in both hips c. a walker for a person who cannot ambulate in the home without one d. grab bars In the bathroom for a person who cannot bathe or toilet without them

c. a walker for a person who cannot ambulate in the home without one. the criteria for DME to be reimbursable by Medicare are that the item must be necessary and reasonable to treat an illness or incidence of decreased functioning.

A client with a diagnosis of paranoid schizophrenia is participating in an initial evaluation session at a psychiatric day treatment program. Halfway through the completion of an activities configuration, the client states the referral to this day program is inappropriate and unnecessary because it was made by an incompetent psychiatrist. The client becomes visibly upset and loud when talking about the unfounded referral and the psychiatrist's incompetence. Which is the best initial action for the OT to take? a. end the evaluation session and tell the client to call to re-schedule when feeling better b. assure the client of the referring psychiatrist's competence and advise the client to discuss concerns with the doctor. c. acknowledge that the client appears upset and ask if the client is able to focus on the remaining evaluation. d. contact the day program's chief psychiatrist to report the client's stated concerns about the referring psychiatrist's competence.

c. acknowledge that the client appears upset and ask if the client is able to focus on the remaining evaluation. a simple acknowledgement will valuate the individual's concerns in a non-threatening way. There is no need to report the client's concerns for there is no concrete evidence of physician incompetence at this time.

A clubhouse program hires an OT as a consultant. The clubhouse board of directors requests that the OT consultant focus on the development of evening and weekend leisure activities. Which recommendations should the therapist make for the selection and completion of the group activities? a. activities to be selected and completed by clubhouse members according to guidelines provided by the therapist b. activities to be selected by the therapist with written instructions provided to the group attendees for activity completion c. activities to be selected by clubhouse members with activity completion led by the clubhouse members d. activities to be selected by the therapist and completed by the clubhouse members after demonstration by the therapist

c. activities to be selected by clubhouse members with activity completion led by the clubhouse members clubhouse programs utilize a consumer empowerment model that emphasizes active involvement of all participants in the decision-making and implementation processes of the clubhouse. Other choices are too directive for this model.

A 5-year-old child with cerebral palsy exhibits a digital pronate grasp pattern during writing activities. The child's individual education program (IEP) goal is to use a static tripod grasp while writing her first name in a legible manner on primer paper upon request 80% of the time. What are the MOST APPROPRIATE methods to promote the desired grasp pattern? a. activities to increase hand tone and stability b. activities to facilitate the palmar-raking grasp c. activities to promote supination and radial-ulnar dissociation d. activities to increase proximal control and shoulder strength

c. activities to promote supination and radial-ulnar dissociation These activities promote the developmental had patterns necessary for the desired grasp. Activities for tone and stability do not address the grasping pattern itself. Palmar-raking grasp is an even more primitive grasp. Activities to increase control and shoulder strength are important, but they do not directly assess the components for the grasp.

A 5-year-old child with cerebral palsy exhibits a digital pronate grasp pattern during writing activities. The child's individual education program (IEP) goal is to use a static tripod grasp while writing her first name in a legible manner on primer paper upon request 80% of the time. What are the MOST APPROPRIATE methods to promote the desired grasp pattern? a. activities to increase hand tone and stability b. activities to facilitate the palmar-raking grasp c. activities to promote supination and radial-ulnar dissociation d. activities to increase proximal control and shoulder strength.

c. activities to promote supination and radial-ulnar dissociation This grasp promotes a static tripod grasp because it involves the developmental hand patterns necessary for the desired grasp. Activities to address tone and stability address strength and stability and not the grip itself. Activities to facilitate the palmar-raking grasp is developed at 5-7 months and is a precursor to the grasp the child is already using. Shoulder control and strength are important but they do not directly address the movements necessary for the proper grasp.

An OTR is completing a feeding evaluation of a 3-year-old child who has mild hypotonia, immature oral motor control, and oral hypersensitivity. The child sits in a standard dining chair during meals and requires moderate to maximum assistance from a caregiver for feeding. When attempting to swallow food, the child hyperextends the neck, elevates both shoulders, and has poor lip closure. What information should the OTR include in the INITIAL caregiver instructions based on this observation? a. methods for using cryotherapy to simulate facial muscles prior to feeding the child b. handling techniques for facilitating full forward neck flexion during feeding c. adaptive positioning techniques for promoting trunk alignment d. neuromuscular facilitating techniques for promoting head and trunk stability.

c. adaptive positioning techniques for promoting trunk alignment this provides a firm BOS to the trunk and feet. Handling for facilitating full forward flexion during feeding is important, but not the initial action needed for feeding. Neuromuscular facilitation techniques for promoting head and trunk stability are good but these are specialized and take a while to learn.

an individual hospitalized for the first time due to a brief psychotic episode attends OT group. During task performance, the OT, notices that the person is restless with hand tremors and shaking legs. Which of the following should the therapist document that the person is exhibiting? a. akinesisa b. pseudo-parkinsonism c. akathisia d. tardive dyskinesia

c. akathisia akathisia is a side effect of antipsychotic meds that is exhibited by restlessness, hand tremors, and shaky legs. Akinesia is also a potential side effect, but this is evident by a lack of movement. Psuedo-parkinsonism is a side effect that appears as rigidity, pill rolling tremors, masked face, and a shuffling gait. Tardive dyskinesis is an irreversible neurological condition caused by prolonged use of neuroleptic medications.

A young adult with a T9-10 SCI wishes to engage in sports activities. Which wheelchair features are best for the OT to recommend to this client? a. a heavy-duty foldable frame with a high back b. an ultra-light foldable frame with a high back c. an ultra-light rigid frame with a low back d. a heavy-duty rigid frame with a low back

c. an ultra-light rigid frame with a low back sports competition wheelchairs are usually made with rigid construction, because a folding wheelchair doesn't provide the necessary stability. A low seat back enhances the user's upper body movements.

A person is diagnosed with COPD. The OT instructs the individual on breathing exercises to use control respiration rate during activities. The therapist tells the person to inhale as if smelling roses. How should the therapist tell the person to exhale? a. as if blowing out 20 lit candles on a birthday cake b. as if blowing forcibly to relight a campfire c. as if flickering a lit candle d. in quick, short multiple breaths

c. as if flickering a lit candle when one exhales to flicker a candle, it uses pursed lip breathing which is a method of controlled breathing. This type of breathing decreases rate of breathing and prevents airway collapse.

An adult who incurred a TBI 3 months ago is referred to a home care agency to receive OT services. The client's referral states that cognition is at a level VII of the Rancho Los Amigos scale and that grasp and shoulder mobility are limited. During the initial interview, the client reports being frustrated by the inability to independently engage in the previously enjoyed and personally meaningful activity of cooking. The client reports frequently losing place when reading recipes and an inability to find things in the kitchen. The client responds well to the compensatory approach when performing countertop meal preparation tasks. The therapist decides to use a dynamic interactional approach as the theoretical foundation for the cognitive-perceptual intervention for other meal prep activities. During a session to train the client in scanning strategies, the client accurately finds two items in a fridge. Which is the therapist's most appropriate initial response to the client's task success? a. praise the client for successful task completion b. ask the client to now find 3 items in the fridge c. ask the client how he/she knows the items are correct d. ask the client to now find two items in the pantry

c. ask the client how he/she knows the items are correct A fundamental technique in the dynamic interactional approach is awareness questioning to help the individual identify successes, detect errors, estimate task difficulty, and predict outcomes.

An adult with a right CVA begins OT to improve grooming and dressing skills. The client refuses to work with a female therapist and insists on working with a male therapist. The OT supervisor reviews the department's case load and finds that the department's sole male therapist has limited experience working with individuals with CVA. Which action is best for the supervisor to take next in response to this situation? a. encourage the individual to work with the female therapist currently assigned and assure the client of the therapist's skill and competence. b. contract with a per diem male therapist to work with the client and provide the needed interventions c. assign the male therapist to work with the individual and provide supervision for the therapist d. modify the intervention goals to include activities that are more comfortable for the individual when working with the female therapist

c. assign the male therapist to work with the individual and provide supervision for the therapist This is the choice that respects the persons autonomy. Encouraging the client to do something that is uncomfortable violates the client's rights. Modifying the goals is not appropriate as the goal must be based on the assessment of the clients abilities. In addition, the established goals are relevant and should be addressed.

An OTR is working with a 3-year-old child who has spastic quadriparesis and needs a new seating and positioning device. What approach is MOST APPROPRIATE to use when selecting the new device for this child and family? a. sensorimotor approach b. motor-learning c. biomechanical approach d. neurodevelopmental approach

c. biomechanical approach This approach focuses on postural alignment, joint stability, and musculoskeletal problems. A sensorimotor approach is used for righting, equilibrium, reflex integration, and movement patterns. A motor-learning approach focuses on normalizing muscle tone, remediating sensory dysfunction, and improving dyspraxia. An NDT approach uses handling techniques to allow for typical movement.

A child with a short attention span and processing difficulties is referred to OT. Based on the results of a screening, the therapist decides to focus the evaluation on sensory processing performance skills. Which should the therapist evaluate? a. form-constancy b. right-left discrimination c. proprioception d. spatial relations

c. proprioception proprioception is a component of sensory processing performance skills. Form-constancy, right-left discrimination, and spatial relations are all part of perceptual processing.

A 9 y/o child identifies the assembly of a model as the most desired play activity. The OT determines that the child would have difficulty completing the selected model. Which action is most effective for the therapist to take during the next intervention session? a. allow the child to work on the model and provide max assistance as the child completes the project. b. explore with the child why completing the model is desired by the child and provide alternative project choices. c. break the project down into accomplishable segments and instruct the child to complete one segment at a time. d. explain to the child several reasons why the selected model is not the best choice for the child and provide alternative project choices.

c. break the project down into accomplishable segments and instruct the child to complete one segment at a time.

An OTR is working with a 7-year-old with autism spectrum disorder who has hypersensitivity to almost every sensory system. The child does not like tactile play, fears loud noises, and dislikes playing on high surfaces like the slide or jungle gym on the playground. What intervention is MOST EFFECTIVE to improve comfort and reduce fear and anxiety in this child with autism spectrum disorder? a. imitate the child and wait for the child to initiate interaction b. reduce the use of direct eye contact during the session c. carefully grade the introduction of novelty into the session d. discuss ways to use objects based on their properties.

c. carefully grade the introduction of novelty into the session This strategy allows for the just right challenge. Option A&B are most effective for improving engagement and interaction in a child with ASD. Discussing use of objects improves play and ideation skills in this population.

An OTR is working in early intervention with a 15-month-old with sensory processing problems who is at risk for autism spectrum disorder. The early intervention team is working with the child and family on the goal of increasing caregiver-child interaction. Which intervention is MOST EFFECTIVE in addressing this goal? a. limit nighttime sounds and lights to promote a better sleep routine b. teach caregivers massage for calming and increased self-regulation c. coach parents on how to read child's cues for greater responsiveness d. play computer or tablet activities with the child in the home

c. coach parents on how to read child's cues for greater responsiveness This strategy allows parents to learn to positively respond to communication/needs to they provide the necessary input for optimal sensory processing. Massage is a strategy for sensory regulation to children with ASD, but it is not the most effective intervention for increased child/caregiver interaction

An adolescent student with Ducheene MD and depression is being evaled for a power chair. Which is the most important area to be considered during the OT eval of the student to determine the student's readiness for the wheelchair? a. level of interest b. fine motor skills c. cognitive skills d. postural control

c. cognitive skills Cognitive skills include alertness, spatial operations, judgement, decision making, and problem solving. Since these abilities are needed for the safe operation of a power wheelchair, it is essential that the OT assesses the student's cognitive level.

An OTR is working with a speech-language pathologist (SLP) to create an augmentative communication system for a 7-year-old client with limited mobility or motor control in the upper and lower extremities. She is seated in a wheelchair and is able to use her head to turn from side to side. What is the OTR's FIRST role in this case? a. recommend the type of device b. recommend the positioning of the device c. collaborate with SLP to determine the type of device d. create a visual schedule to promote communication

c. collaborate with SLP to determine the type of device When working with other disciplines, it is important to recognize each others roles and responsibilities. OT often has a key role in determining the best positioning for the device, but in this case the client has not received recommendations for the type of device. Therefore, this is not the BEST option YET. Visual schedules are generally not used as an overall communication device.

A person fell and sustained bilateral Colle's Fractures. The client wore bilateral short-arm casts for 6 weeks. After cast removal, the client began OT sessions to increase endurance and strength prior to returning to work. The client tends to work hard when performing resistive exercises with both wrists. The therapist monitors the client for observation. Which behavior indicates overexertion? a. decreased respiration rate during resistive wrist flexion b. increased ability to achieve full ROM of the wrist c. complaints of pain in the wrist extensors d. consistent strength in wrist extension activities

c. complaints of pain in the wrist extensors Other factors are signs of adequate performance, not exertion.

a client has developed edema in the right hand after it was immobilized in a cast. She is right-hand dominant and has difficulty using her left hand for tasks that require dexterity. She lives alone and needs to use both hands for IADLs such as meal preparation and cleaning. The OTR wants to recommend a temporary method to provide compression. Which of the following compression materials is MOST appropriate for the client? a. low-stretch bandage b. coban wraps c. compression glove d. custom-made garment

c. compression glove compression gloves provide light compression without the need for custom measurements. The client will be able to easily don and doff the gloves without the dexterity needed to apply bandages or coban wraps.

s new outpatient mental health facility offering OT services opens in a community of 100,000. The OT director of an established outpatient program is concerned that the new program will affect service delivery. Which action is best for the director of the established program to take in response to this potential competition? a. wait for the new facility to implement services and develop services in the area not covered by that facility b. offer the same services that the new facility offers and advertise a competitive price c. conduct a program evaluation and develop a marketing plan based off the results d. design an intensive marketing campaign to local consumers and third party payers

c. conduct a program evaluation and develop a marketing plan based off the results The best approach is to conduct a program eval to ID the strengths and limitations of the existing program and then develop a promotional plan that highlights the program's strongest aspects.

an individual with left hemiplegia who is right-hand dominant receives training to resume independent driving. Which adaptation is best for the OT to recommend the client use? a. palming the steering wheel b. hand controls for brake and gas pedals c. a spinner knob on the steering wheel d. life-sided accelerator pedal

c. spinner knob on the steering wheel a person who is right-hand dominant with left hemiplegia can drive one-handed using a spinner knob. Palming the steering wheel is not recommended for one-handed drivers, for it is easier to lose control of the vehicle. There is no functional need to change a car's existing pedal arrangement.

2 months ago, a child received full thickness circumferential burns of both UEs. The child shows progressively decreasing ROM of the right elbow with a 25* loss to date with a stiff endpoint. The child complains of increased pain in the elbow. Which action should the OT take? a. begin use of a continuous passive motion device b. apply heat prior to active ROM c. contact the child's physician d. splint the child's elbow in static splints

c. contact the child's physician The child shows probable symptoms of heterotopic ossification, a calcium deposit that may occur in or near a joint after burns. Circumferential burns are most susceptible to this condition. Symptoms include decreased joint excursion, a stiff endpoint, and increased pain. All other options are contraindicated at this time, and a physician must be the one to determine the next course of action.

A single parent of 2 children, a 4 y/o and a 6 y/o, is referred to OT services in a rehab center. The client had a brain tumor removed 1 month ago. The referral states that the client has residual cognitive-perceptual deficits. Screening indicates that the client's sensori-motor abilities are WFL. The OT administers a standardized cognitive-perceptual assessment to the client. The client demonstrates difficulty performing the first 2 tasks of the eval. Which is the most appropriate action for the therapist to take? a. continue the eval and provide verbal cues during task performance b. continue the eval and model each evaluation task c. continue the eval according to the established administration protocol d. discontinue the eval to avoid frustrating the client

c. continue the eval according to the established administration protocol A standardized assessment must be administered a certain way in order to be reliable. Providing cues or modeling would compromise the reliability of the assessment tool, and discontinuing the assessment would mean the therapist would not obtain the necessary info about the patient's status.

an OT leads a community integration group for individuals with mild intellectual disabilities who reside in a group home. During a travel training session, a member of the group slips while going up the stairs of the bus. The client quickly gets up, pays the fare, sits down and jokingly states "good thing I bounce well." Which action should the OT take after assessing that the person is not injured? a. cancel the planned activity and return the group home to file an occurrence report b. ask the bus driver to radio for an ambulance to obtain a medical assessment to validate that the client was not injured c. continue with the planned activity and file an occurrence report upon return to the group home. d. continue with the activity and ask the client to report any development of symptoms related to the fall

c. continue with the planned activity and file an occurrence report upon return to the group home Upon determining there was no injury, there is no need to cancel the activity or call an ambulance. It is standard policy to file a report, but in minor events it does not need to happen immediately. Asking the client to report additional symptoms is appropriate, but it is not the most important action for the OT to take. Proper documentation is a professional and legal requirement.

An OTR is working with a kindergarten classroom teacher in developing seating options for all students in her classroom. The teacher shared that her students are seated on the floor during several academic instruction times. They are positioned in a circle and often students struggle with keeping their hands to themselves and their bodies in their own space. What type of chair might the OTR recommend to the teacher? a. corner chair b. infant sitter c. cube chair d. Howda Hug chair

c. cube chair A cube chair is a universally designed chair that can support all children in the kindergarten class. It provides support as well as cues on personal space. A corner chair provides external support for children with inadequate postural control of the head, neck, and trunk. Infant sitters are designed for smaller children. The Howda Hug chair provides proprioceptive input to those needing extra sensory input. Not all students may benefit from this.

a 5 y/o is referred to OT. Upon the completion of a standardized test eval, the OT determines that the child demonstrates age-appropriate cognitive and fine motor skills. Which activity would the child be able to complete at this developmental level? a. cutting long thin strips with scissors b. holding and snipping with scissors c. cutting simple figure shapes with scissors d. opening and closing scissors in a controlled fashion

c. cutting simple figure shapes with scissors Cutting strips is a 3-4 y/o skill. Holding and snipping and opening and closing scissors is a 2-3 y/o skill.

An individual with post-polio syndrome receives an OT re-eval of functional status. How should the therapist initiate sensory testing with this client? a. demonstrate the test with the individual's vision occluded b. proceed proximal to distal c. demonstrate the test with the client's vision not occluded d. proceed distal to proximal

c. demonstrate the test with the client's vision not occluded Sensory testing must begin with a demonstration of the test with the client being able to visually observe the demonstration. After the demonstration is complete, the testing proceeds with the vision occluded.

An OT is conducting a community transportation group with individuals attending a TBI day treatment in an urban area. Which should the group do first? a. read a subway bus or map b. take a subway or bus as a group c. determine a destination d. purchase a subway or bus fare card

c. determine a destination The group has to determine the destination before they can decide what type of fare is necessary to get them to the destination

a young adult recently diagnosed with depression and anorexia nervosa is a consumer of services at a psychosocial clubhouse. The client attends individual OT sessions once a week and several evening and weekend groups. When completing an activities schedule and questionnaire, the client identifies the clubhouse groups and part-time work as an editorial assistant as the only activities completed each week. In high school the client was captain of the swim team, played tennis, and worked in an after-school activities program for young children. During the clubhouse vocational support group, the client reports difficulty keeping track of the job tasks that need to be completed each day. Which is the most appropriate recommendation for the OT to make the client? a. write down directions for each task that needs to be completed b. keep a daily log of completed tasks c. develop and use a checklist of tasks to be completed each day d. ask the work supervisor to rovide verbal cues through the workday.

c. develop and use a checklist of tasks to be completed each day this provides a clear visual cue of what needs to be accomplished. There is nothing in the scenario to indicate that the person does not know how to perform the tasks, therefore, there is no need for the person to write down directions for each task.

a child with ADHD and conduct disorder attends an after school program that utilizes sensory integrative and behavioral management approaches to achieve intervention goals. Snacks are provided and occasionally used as rewards. A parent insists that a child not be given any foods containing sugar. Which is the OT's best response to this request? a. discontinue providing sugary snacks but continue their use as rewards in the behavioral management program b. provide the parent with recent research that refutes the link between sugar and problem behaviors c. discontinue providing sugary snacks for the child to comply with the parents request d. inform the parent that the therapist will discuss the issue with the program's administrator to determine the best course of action

c. discontinue providing sugary snacks for the child to comply with the parent's request The parents request must be respected and honored .

a hospital-based OT is committed to promoting the profession to hospital staff and the general public. Which is the most effective action for the therapist to take to accomplish this goal? a. write an article about recent accomplishments of new OT staff members in the hospital's employee newsletter b. include photos of patients participating in the new driver rehab client in the hospital's annual report c. display info sheets about major areas of OT practice on the hospital's cafeteria bulletin board d. write a monthly column in a local newspaper about OT approaches for a diversity of clinical conditions

c. display info sheets about major areas of OT practice on the hospital's cafeteria bulletin board a hospital cafeteria services both hospital staff and the general public so this would be the communication venue that would be readily accessible to both audiences. The other choices do include activities which can help promote OT but they do not target both hospital staff and the general public.

a patient has been discharged from a rehab facility 6 mo ago. A staff therapist who works at the facility sees the former patient and the senior therapist who treated the patient in a dating situation. The senior therapist confirms involvement in a personal relationship with the former patient. Which is the staff therapist's best response to this situation? a. report the therapist to NBCOT b. advise the faculty director c. do nothing d. report the therapist to the OT supervisor

c. do nothing a health care practitioner can date a former but not current patient.

an intervention plan for a person with a complete lesion of the spinal cord at the C6 level has been developed by the client and OT. Which activity should be included in this plan as a goal for the client to develop the ability to independently perform? a. typing with a mouth stick b. transferring using depression transfers c. donning pants while in bed d. feeding using a suspension sling or mobile arm support

c. donning pants while in bed a person with this SCI can independently don underwear and pants while laying in bed. Min assist is needed to don socks and shoes. A person with this level will use a slide board transfer. A depression transfer will be used at C7 level.

An OT works in a school system with a child with developmental delays. One of the goals of treatment is to develop pre-writing skills. The child exhibits the ability to grasp a pencil proximally with crude approximation of the thumb, index and middle fingers and the right and little fingers slightly flexed. The therapist develops an intervention plan. Which grasp should be the focus for the implementation of intervention.? a. digital pronate grasp b. static tripod grasp c. dynamic tripod grasp d. palmar supinate grasp

c. dynamic tripod grasp The grasp pattern described in this case is the static tripod grasp. The next grasp to be mastered after this is the dynamic tripod grasp. Other grasps listed are precursors to the static tripod grasp.

an individual with CRPS type 1 presents with severe pain and pitting edema in the right hand. The individual has a secondary diagnosis of degenerative joint disease. Which should the OT initially recommend to address these concerns? a. PROM of wrist and fingers b. retrograde massage from distal to proximal c. elevation of the affected hand above the heart d. retrograde massage from proximal to distal

c. elevation of the affected hand above the heart elevation will promote venous and lymphatic drainage and decrease the hydrostatic pressure in the blood vessels. It is not the initial treatment when severe pain is present. PROM is not advisable for people with degenerative joint disease.

an individual who has PD presents with poor trunk rotation during ambulation and while performing ADLs. according to the neurophysiologic approaches, which is the most effective therapeutic intervention for the OT to use with this person. a. facilitation of trunk rotation using NDT b. slow rolling with the person supine with knees and hips flexed c. engagement in ADLs using diagonal patterns d. provision of a rolling walker to compensate for limited rotation and enhance mobility.

c. engagement in ADLs using diagonal patterns according to neurophysiologic approaches, the most appropriate approach is to use a technique to facilitate rotation during activity performance. PNF diagonals are the best choice because many activities can be used with these patterns. NDT may facilitate rotation, but its not functional.

an OTA recently attended a 2-day splinting workshop. The OTA asks the supervising OT for a caseload that includes more clients that require splinting interventions. Which action is best for the OT o to take in response to this request? a. decline the request because splinting is an advanced practice skill b. ask the OTA to give a splinting in-service to demonstrate acquired knowledge c. establish the OTA's competency in splinting d. distribute the department's caseload to meet the OTA's request

c. establish the OTA's competency in splinting the establishment of service competency is required before the OTA takes on a new task. This ensures that the OTA will achieve the same intervention outcome as the OT.

an older adult is referred to OT with a diagnosis of OA in both knees and elbows. The OT interviews the patient and learns that the patient desires to return home to live alone independently in a bi-level home. Which should the therapist do next? a. provide suggestions for adaptations to the patient's bathroom to increase safety b. train the patient in energy conservation techniques during IADL tasks c. evaluate the patient's performance of daily activities in a simulated setting d. provide the patient with a HEP to build strength and ROM

c. eval the patient's performance of daily activities in a simulated setting the person was just referred to OT and the interview indicated a need for further eval. By observing these activities in a simulated setting, the OT can assess the demands of the activities and the person's capabilities. Once info is obtained the OT can make recommendations.

a therapist in a rehab hospital receives a physician's order to eval a patient who has metastatic bone cancer with pathologic fracture of the ribs. The physician requests information about the patient's ability to tolerate resistive activities of the UEs. Which action is best for the OT to take in response to this request? a. complete a MMT with less than full resistance b. provide isometric strengthening exercises for UE c. eval tolerance during functional ADL tasks d. initiate TheraBand stretches and report tolerance to physician

c. eval tolerance during functional ADL tasks a patient with cancer with pathological fractures is at high risk for fractures from higher resistance activities. In this situation, MMT should be deferred and information about tolerating resistive activities can be gained through lower resistance ADLs. The therapist should not conduct MMT even with less than full resistance. Resistive exercises, whether isometric or isotonic, are not ideal in this situation because it can cause fracture and because a patient with a rib fracture should not engage in high-resistive upper body exercises.

A school-based OT consults with a teacher regarding a non-speaking student who uses a wheelchair and an augmentative communication device. The teacher reports that the student has been making many errors on the communication device but that no difficulties had been observed when the student used the device in the past. Which is the most effective initial action for the therapist to take in response to the teacher's report? a. advise the teacher to contact the student's parents and recommend that they bring the child to a physician for an examination b. reassess the student's motor and communication abilities to determine needed modifications c. evaluate the position of the student in the wheelchair and the device on the wheelchair d. reposition the communication device on the wheelchair to facilitate access and increase accuracy

c. evaluate the position of the student in the wheelchair and the device on the wheelchair. even minor changes in the person's positioning can impact access and use of an assistive device.

An OT working in an outpatient peds clinic has been seeing a child for sensory processing deficits. After 12 treatments a denial letter for services has been received. The letter states that the diagnosis and the treatments do not meet the requirements of the policy. The parents who are aware of the denial want the therapist to change the child's diagnosis code in order to receive reimbursement. What is the best way for the therapist to respond? a. discharge the child from OT and provide a home SI program for follow-up by the parents b. encourage the parents to seek an alternate insurance carrier to receive improved coverage for services c. explain to the parents that the current diagnosis cannot be changed in order to receive reimbursement d. encourage the parents to set up a payment plan with the clinic's accounting department

c. explain to the parents that the current diagnosis cannot be changed in order to receive reimbursement it is against the AOTA code of ethics of veracity. If there was an error made by mistake, it could be corrected and resubmitted with a letter of explanation. However, with this scenario, the code was correct. It is not the therapist's place to suggest changes in insurance and that would not solve the current situation anyways. While setting up a payment plan may be necessary, it does not address the parent's request to change the codes.

a client with chronic depression and a spouse attend a discharge meeting with the OT following the client's 6 week hospitalization for a major depressive episode. They state that they have few activities in common and spend little time together. The client retired 2 months ago and the spouse continues to work full time. Which of the following should the therapist encourage this couple to do to address this concern? a. immediately participate in 1 activity together b. become involved in their own individualized activities during the day c. explore activities they enjoyed together and alone d. delay planning activities until the depression is totally resolved

c. explore activities they enjoyed together and alone It is important for the client to explore activities enjoyed alone so that retirement and time separated from his spouse will be enjoyable and meaningful.

A 16-year-old student had a traumatic brain injury as a result of an all-terrain vehicle (ATV) accident 2 months ago. The OTR is providing an intervention session with the student that focuses on an expressed interest by the student to obtain a small part-time job locally. What is the INITIAL action that the OTR should take based on this information? a. locate a job on social media or a website the student may like b. explain that having a job with this injury would require adaptations c. explore realistic choices based on the student's interests and abilities d. document the student's interest in obtaining a job in the medical record

c. explore realistic choices based on the student's interests and abilities Locating a job online does not consider the student's interests. Documenting the interest does not provide an intervention and should have already been done.

A OTR is working with at-risk adolescents with a history of depression and suicide attempts on an in-patient acute psychiatric unit. What OT intervention is MOST BENEFICIAL for this population? a. facilitating a social-skills group focusing on past events and traumas b. planning a menu for a healthy cooking group c. exploring new leisure interests and activities in a coping skills group d. developing a school in service for suicide prevention

c. exploring new leisure interests and activities in a coping skills group this can help stop or divert suicidal thought. Focusing on the past traumas is not the most beneficial intervention during acute stages because it can give the individuals a sense of hopelessness. Developing a suicide prevention in service is valuable but not at this point in recovery.

a preschool aged child is hypersensitive to touch. The therapist decides to use the SI FOR to help the child develop the ability to adequately modulate sensory stimuli. Which technique is best for the OT to use initially with this child? a. fast brushing to the child's arms in a direction opposite to the hair growth b. lightly moving touch to the child's abdomen and extremities c. firm pressure where the child can see the source of the stimuli d. quick icing where the child cannot see the source of the stimuli

c. firm pressure where the child can see the source of the stimuli firm pressure and deep touch where the child can see the stimuli is an effective approach for children with tactile hypersensitivity. This technique tends to be more tolerable than light touch which tends to be aversive especially when applied to the face, abdomen, or in a direction opposite of hair growth.

A child in fourth grade is struggling academically and his teacher cannot read his handwriting. The teacher asks the OTR at the school to evaluate his handwriting abilities. What is the FIRST step when evaluating a child's handwriting? a. conducting a standardized handwriting test b. examining the child's visual and motor functions through clinical observations c. gathering the data on a child's writing compared with classroom standards d. review the child's medical records

c. gathering the data on a child's writing compared with classroom standards Initially, the child's performance in the context of the classroom should be the focus as this data gives the OTR an integrated picture of the child's written communication. A standardized test is only necessary after data and clinical observations are gathered, and clinical judgment is used to determine if further eval is warranted. Examining visual and motor functions are important but not initially, this is typically done after the occupational profile and interview are completed. Medical records are not often available in school-based OT.

An OT and an OTA co-lead a work adjustment group. 1 member has become progressively more dependent on the OTA for directions, praise, and input throughout the group activities. Which action should the group leaders initially take in response to these behaviors? a. schedule several individual sessions with the OTA and group member to examine the issues of dependency and transference b. inform the attending psychiatrist that the group member is exhibiting signs of transference and dependency c. have the OTA work with the person during group sessions to develop independence in task completion d. have another therapist co-lead the group with the OT and reassign the OTA to another group.

c. have the OTA work with the person during group sessions to develop independence in task completion This question addresses therapeutic use of self and effective group process. The best answer is to utilize the group situation and have the OTA function as a change agent during the group's activities. It is not necessary to devote individual sessions to this issue.

a child with spinal muscle atrophy can no longer reach beyond 90* of shoulder abduction and 90* of shoulder flexion. The parents state that the child can no longer don or doff a t-shirt. Which is the best approach for the OT to recommend the child to use for dressing? a. place the t-shirt directly on the child's lap, have the child don the arms first, then don the head of the t-shirt. b. have the child learn to don and doff front-opening shirts instead of t-shirts c. have the child support the elbows on a table at chest height to don the t-shirt over arms, then don over head. d. have the child sit with the trunk well-supported, lean to the right and don the right arm, repeat to the left, then don the head of the t-shirt.

c. have the child support the elbows on a table at chest height to don the t-shirt over arms, then don over head. spinal muscle atrophy is a progressive disorder, and the therapist needs to prepare the child and family for progressive loss of skills.

a young adult recently diagnosed with undifferentiated schizophrenia is referred to an OT day treatment program. Which should the OT do first with the client? a. determine short-term and long-term goals for program participation b. model desired behaviors during OT groups c. have the client complete an occupational interest checklist d. encourage the client to maintain a daily log of medication intake

c. have the client complete an occupational interest checklist upon referral, the first step in the OT process is screening. Determining the person's occupational interests can help identify areas requiring further eval. one cannot determine goals until an eval is done.

An OT works with adolescents who have been victims of child abuse. Treatment can be carried out in groups or on an individual basis. Which of the following would indicate to the therapist that intervention should be provided to an adolescent on an individual basis rather than in a group? a. the adolescent wants more socialization experiences b. the adolescent could benefit from feedback from peers c. the adolescent desires greater control over the environment d. the adolescent needs an opportunity to gain situational perspective

c. he adolescent desires greater control over the environment this type of person would benefit from working on an individual basis because groups are unpredictable and an effective group process requires trust and the sharing of control among all members.

An OTR is working with a 2 year, 3-month-old child with Down syndrome who is receiving early intervention services. The family and therapy team are working on ways to further develop the child's social play skills. What intervention is MOST BENEFICIAL to increase social play skills? a. support play using adapted toys for engagement b. role play turn taking with child in a therapy session c. help parents create a play group for the child d. praise engagement in imaginative play in therapy

c. help parents create a play group for the child This is the most beneficial intervention because it includes a family-centered approach in a natural environment. Adapted toys are more appropriate for children with low postural stability and limited extremity movement. While teaching the skill of turn taking is beneficial, it is best conducted with another child or children and in the moment while the play occurring. Praising engagement in imaginative play is a good skill to reinforce but this does not include peers so it does not promote social interaction.

An individual is s/p carpal tunnel release. When the OT conducts a sensory test for sharp/dull pain, the person reports dull as sharp on the palmar surface of the thumb and index finger. All other responses were correct. Which is accurate for the therapist to document about the client's sensation? a. impaired for pain along C5 & C6 dermatomes b. hypersensitive along the ulnar nerve distribution of the palmar surface of the hand c. hypersensitive along the median nerve distribution of the thumb and index fingers c. absent for pain along the median nerve distribution

c. hypersensitive along the median nerve distribution of the thumb and index fingers The sensation is not considered absent because the individual does feel a sensation, it is just the incorrect sensation. Ulnar nerve distribution involves the ring and little fingers.

several newly homeless veterans with a variety of mental health diagnoses attend an OT community re-entry group conducted in a community-based shelter. Which should be the primary focus of the initial group session? a. development of home management skills such as meal prep b. determination of financial assets and money management skills c. identification of local resources such as soup kitchens and thrift stores d. exploration of vocational interests and employment opportunities

c. identification of local resources such as soup kitchens and thrift stores locating basic resources is the most essential survival skill listed for people who have recently become homeless. Initial sessions at the homeless shelter would likely focus on basic survival and personal self-care skills prior to focusing on vocational interests, ADLs, and IADLs.

A single parent with RA and two school-aged children reports difficulty completing a HEP. The parent states that multiple familial, work, and home management responsibilities fill the day and additional activities cannot fit into the day. Which is the best action for the OT to take in response to the se realities? a. explain and reinforce the importance of active ROM exercises of remediation of dysfunction b. provide intervention to develop time management skills and enable temporal adaptation c. incorporate the parent's engagement in a diversity of role activities into the home program d. increase the frequency of OT sessions to compensate for lack of follow-through with the home program.

c. incorporate the parent's engagement in a diversity of role activities into the home program

A level 2 fieldwork student's first assigned case is an individual with right hemiplegia. The supervising therapist reminds the student that primitive reflexes can emerge when someone incurs a CVA. The therapist demonstrates this point by rotating the client's head to the right and stating that the observed responds demonstrates a subtle ATNR reflex. The therapist asks the student to describe the client's reaction that resulted in the therapist's interpretation. Which is most accurate for the student to state the client is exhibiting based on this observation? a. increased flexor tone of the right UE b. increased extensor tone of the left UE c. increased extensor tone of the right UE d. increased extensor tone in both UEs

c. increased extensor tone of the right UE. When observing ATNT, one will see flexion of the skill side of the body and extension of the face side.

a young adult recently diagnosed with depression and anorexia nervosa is a consumer of services at a psychosocial clubhouse. The client attends individual OT sessions once a week and several evening and weekend groups. When completing an activities schedule and questionnaire, the client identifies the clubhouse groups and part-time work as an editorial assistant as the only activities completed each week. In high school the client was captain of the swim team, played tennis, and worked in an after-school activities program for young children. During an individual session with the OT, the client states, "I don't know what I want to work on. I don't really know what my goals are." Which is the most appropriate action for the OT to take? a. defer the development of an intervention plan until the individual has determined personal goals b. establish a short-term goal related to improving the individual's goal setting skills c. initiate a discussion with the individual about what is personally important d. contact the individual's psychiatrist to request a medication eval

c. initiate a discussion with the individual about what is personally important This answer is client-centered, employs therapeutic use of self, and incorporates the patients rights. It is also helpful for establishing rapport with the client. Setting up a short term goal to improve goal-setting without the input of the individual is vague and would not contribute to a client-centered intervention plan.

a person with recently diagnosed MS begins an outpatient program. During the initial intervention session, the client expresses difficulty concentrating on daily tasks due to chronic fatigue. Which should the OT do in response to the client's concerns? a. reassure the client that these are typical symptoms of the diagnosis b. reassure the client that medications will ease these symptoms c. inquire about the client's fatigue level during different tasks d. eval the clients endurance and cognition

c. inquire about the client's fatigue level during different tasks the therapist must obtain further info. This info is essential to plan intervention for energy conservation. Reassurance does not deal with the stated problem. in addition, meds might not ease the client's symptoms. There is no info provided in this example to indicate the need for revaluation of the clients status.

A home-care OT seeks to enhance an elder's compliance with the OT intervention program. After discussing the goals of the program with the person, which intervention is most effective for the OT to use? a. provide the individual with limited opportunities for practice of skills to decrease boredom b. use multiple, variable instructions to ensure retention of new learning c. integrate previously learned strategies into new activities to facilitate generalization d. teach family members positive techniques to reinforce activity performance in the home

c. integrate previously learned strategies into new activities to facilitate generalization Discussing the person's goals with his or her family may be helpful but can only be done with his or her permission. Additionally, it does not directly address the individual which is the person that the therapist needs to engage.

An OT is treating an individual with PD in an outpatient setting. The therapist observes that the person attends regularly but has little energy and does not seem to be performing the prescribed HEP. Which is the best action for the therapist to take in response to these observations? a. advise the physician to compete a referral for a psych eval b. tell the person that the completion of the HEP is vital to recovery c. interview the person and complete a standardized depression scale d. defer intervention until the person's depression is treated

c. interview the person and complete a standardized depression scale this give the OT standardized, objective data via the depression scale and the interview gives subjective data. A referral to psych would be premature. Telling a person the importance does not directly solve the issue.

A caregiver support group meets weekly at a senior center. A new member attends the group for the third time and listens intently. The person nods in agreement when others speak, but does not participate verbally. Which action is most effective for the OT to take to facilitate the individual's engagement in the group? a. reiterate the group's norms that active participation is expected from all group members b. ask the individual several questions to encourage verbal participation c. invite the individual to join in the discussion, if the person would like d. refer the individual to the center's social worker for individual, non-group counseling

c. invite the individual to join the discussion, if the person would like this choice acknowledges his/her membership but does not pressure the person speak before he/she is ready.

a toddler attends an early intervention program as a result of developmental delay. Over the past 2 weeks, the toddler has successfully completed the activities the OT has provided in order to develop a palmar grasp. Which action should the therapist take next in response to the child's progress? a. continue providing the child with the activities to refine palmar grasp b. review the initial eval to determine new goals c. provide activities to develop a radial palmar grasp d. provide activities to develop an ulnar palmar grasp

c. provide activities to develop a radial palmar grasp ulnar palmar grasp --> palmar grasp --> radial palmar grasp If the initial eval determined there was a need to work on development of grasp, intervention can proceed to the next level without re-eval or establishing new goals.

During a topical work preparation group for individuals recovering from mental illness, a member expresses concern about answering questions related to personal psychiatric history during a job interview. Which action is best for the OT to take in response to these concerns? a. refer the client to a vocational rehab counselor b. encourage the other members of the group to share their experiences c. lead a group discussion on the legal rights afforded in the interview process d. support the client in not disclosing past psychiatric history

c. lead a group discussion on the legal rights afforded in the interview process Since members of the group may not be aware of their legal rights, it's most important to discuss this issue. ADA protects the medical histories of individuals. If the person can perform the essential functions of a job, there is no compelling reason to disclose past medical history. While encouraging members to share experiences and supporting the client's decision are relevant, it is not the MOST important action.

A client was referred to an outpatient therapy center after her hand began to swell. The occupational therapist noticed a significant difference in the size between the client's right and left hands. Which of the following is TRUE of hand volume assessment for clients with edema of the hand? a. hand volume should be measured at the same time of the day since edema varies with rest and activity b. the volumeter is used to measure edema of an individual finger of the hand c. measurements should be made before and after the intervention to measure the effects of therapy d. the therapist measures the water remaining in the volumeter after the hand is immersed to measure hand volume

c. measurements should be made before and after the intervention to measure the effects of therapy hand volume should be measured at different times of the day to measure the effects of intervention or rest. Circumferential measurement is used to assess edema in individual fingers. The client is instructed to immerse the dang. the water is displaced into a graduated cylinder. The therapist reads the graduated cylinder to get the measurement.

an individual with cyclothymic disorder has successfully completed a transitional employment program and is competitively employed in a busy real estate office as an administrative assistant. The person reports feeling overwhelmed by the number of part-time real estate agents who drop off work late in the afternoon to be completed within 1-2 days. The individual contacts the transitional employment program OT for suggestions on dealing with this work stress. Which action is most effective for the therapist to recommend the person take in response to this situation? a. make an appointment for a vocational skills reeval to determine an intervention plan to develop needed skills b. make an appointment with the psychiatrist for a medication eval to manage anxiety c. meet with the work site's direct supervisor at the end of each day to prioritize the next day's workload d. organize the next day's workload at the end of each day according to the deadlines set by each agent

c. meet with the work site's direct supervisor at the end of each day to prioritize the next day's workload input from one's direct supervisor on the prioritization of the next day's work can be very helpful for someone with multiple indiviudlas giving daily work with expectations for quick completion of this work. Orginizing the next days work according to each agent's stated priority can result in multople expectations for work being completed at the same time which would be difficult to fulfil and not decrease stress.

during the initial ADL eval, the therapist notes that the patient consistently spills food due to an inability to adjust movements while cutting food and moving the food from the plate to the mouth. Which deficit does this behavior most likely? a. ideational apraxia b. asomatognosia c. motor apraxia d. tactile agnosia

c. motor apraxia this is the loss of access to kinesthetic memory so that purposeful movement cannot be achieved due to ineffective motor planning; although sensation, movement, and coordination are intact. Ideational apraxia is a breakdown in the knowledge of what is to be done of how to perform an action. Asomatognosia is a body scheme disorder that results in diminished awareness of body structure and a failure to recognize body parts as one's own.

An OT is evaluating a child with DD characterized by hyootonicity. According to the Rood approach, which is the first stability pattern that the therapist should facilitate during intervention? a. roll over b. quadruped c. neck co-contractions d. prone on elbows

c. neck co-contractions according to Rood, motor development follows a sequence termed 'ontogenic motor patterns' that includes 8 different patterns in sequence (supine withdrawal, rollover, prone extension, neck co-contractions, prone on elbows, quadruped, standing). Neck co-contraction is essential for head control.

An OTR is working with a child in the foster system who was born prematurely. The OTR observes that the child easily over reacts to touch, sounds, odors, and tastes. The child also has emotional outbursts of anxiety when climbing on the playground equipment or unpredictable surfaces. Which sensory integration problems BEST describe what this child appears to be experiencing? a. under-responsiveness and poor visual perception b. over-responsiveness and dyspraxia c. over-responsiveness and gravitational insecurity d. under-responsiveness and vestibular-bilateral problems

c. over-responsiveness and gravitational insecurity A child who is under-responsive would not react to this stimuli. Dyspraxia is a motor planning issue and is not indicated in this scenario.

An elementary aged child has recently been discharged to home after open heart surgery. Following the surgery, the child now requires oxygen to maintain appropriate oxygen saturations. The child was provided a large tank of oxygen for the home with a long tube to allow movement about in his bedroom but he cannot quite get into the bathroom. The child is not able to access the dining room or living room. The family states that they are able to order smaller tanks but didn't know why they might need them. What might the OTR recommended during a home visit to promote increased accessibility to the home environment? a. oxygen cylinder cart for smaller tanks b. longer tubing for the large tank c. oxygen backpack carrier for smaller tanks d. remove oxygen during family time

c. oxygen backpack carrier for smaller tanks An oxygen backpack allows the child to increase independence and access to his environment. The large tank can still be used when the child does not need to be mobile. A cart would be more difficult for a child to navigate. Longer tubing may get caught or cause an accident.

an OT employed in a pediatric clinic participates in an initial performance appraisal. The supervisor identifies the area needing improvement as handling skills in working with children with various types of CP. Which is the most effective way for the occupational therapist to improve handling skills? a. observe and experience occupational or physical therapist using handling techniques b. complete an evidence-based practice literature review on the use of handling techniques for children with CP c. participate in an advanced level experiential course on handling techniques d. participate in a teleconference on handling techniques for children with CP

c. participate in an advanced level experiential course on handling techniques this provides opportunities to learn and practice these techniques. It would also provide opportunities to interact with other therapists and benefit from visual and kinesthetic learning. observing a scale therapists is helpful, but it does not offer the opportunity to develop Hands-On skills. A lit review and a teleconference can cover a variety of information about best practices, however, these offer for a little opportunity for hands-on skills.

An individual recovering from a flexor tendon repair surgery is 2 days s/p. The surgeon refers the client to OT with a prescription to use the Kleinert protocol to guide intervention. The OT plans the client's early mobilization program. Which is the most appropriate exercise routine for the OT to use within the limits of a dorsal block splint? a. active flexion/passive extension b. active flexion/active extension c. active extension/passive flexion d. passive flexion/passive extension

c. passive extension/passive flexion the Kleinert protocol in the early phase (0-4 wks) has uses this method. The Duran protocol, which can also be used, uses passive flexion and extension. Active flexion is contraindicated for this type of repair because the tendon could rupture.

An OT completes a cognitive screening for a person with chronic schizophrenia, undifferentiated type. The therapist uses Allen's cognitive disabilities model to guide the eval process. During the screening, the person is able to imitate the whipstitch but cannot imitate the single cordovan stich. Based on these results, the OT determines that further eval is indicated. Which ability is most relevant for the therapist to asses? a. performance of multi-step tasks using overt trial and error problem solving b. performance of multi-step tasks using analytical reasoning c. performance of simple tasks independently using visual cues d. performance of simple tasks with long-term repetitive training.

c. performance of simple tasks using visual cues A person who completes this on the ACL is a level 4. According to the ACL, this person is able to complete simple tasks independently. However, the person relies heavily on visual cues. Since the screening indicated that the person is likely at this level, further eval is needed to provide more specifics about the person's abilities and limitations.

An OT employed at a day treatment center for clients with psychiatric disorders is conducting a leisure planning group. The members of the group decide to take a day trip to the local sculpture garden. Which side effect of psychotropic medications is most important for the therapist to discuss preventative precautions with the group? a. orthostatic hypotension b. akathisia c. photosensitivity d. tremors

c. photosensitivity Photosensitivity results in severe sunburn which can occur during the outdoor trip

following a left CVA, an individual receives OT services at a subacute rehab facility. The patient's personal goal is to be independent with dressing. The patient demonstrates decreased memory, poor sequencing, and ideational apraxia. Which of the following is most effective for the therapist to provide when teaching one-handed dressing techniques to the patient? a. step-by-step verbal instructions b. sequenced photos of the dressing steps c. physical prompts to initiate the steps in dressing. d. a full length mirror for the client to observe self-dressing performance.

c. physical prompts to initiate the steps in dressing ideational apraxia is the breakdown in the knowledge of what should be done and how to perform specific activities. this means one cannot perform the task either spontaneously or on request. providing physical prompts may be enough to cue the individual to begin and complete the task.

An OT working in an outpatient clinic observes the clinic's administrative assistance leaving patient records open on the clinic's reception counter. The assistant has left the clinic for lunch. Which action is best for the therapist to take in response to the observation? a. remind the administrative assistant of the need to keep patient records private when she returns from lunch. b. Immediately contact the administrative assistant's direct supervisor to report this observation c. pick up the records and place them in a location out of public view d. discuss the issue with the clinic's director during their next scheduled supervision session

c. pick up the records and place them in a location out of public view The therapist must protect HIPPA. All other actions do not address the immediate issue.

A 9 y/o girl with the diagnosis of CF is hospitalized in a small rural hospital. Currently, there are no other children in the hospital and the hospital does not have a pediatric play area. The head nurse asks the OT to suggest appropriate play activities that hospital volunteers can provide for the child. Which is the most age appropriate activity for the therapist to suggest? a. dressing paper dolls b. coloring in coloring books c. playing card games d. cutting and pasting pictures onto cards .

c. playing card games children aged 7-12 are developmentally able to participate in games with rules, competition, and social interaction. The other activities reflect creative play for ages 4-7.

an OT providing home-based OT services implements a bed positioning plan for a person recovering from a CVA. The person is receiving care from family members and personal care assistants employed by a home care agency. Which action should the therapist take to ensure the accurate implementation of this plan by the caregivers? a. provide verbal step-by-step directions of the desired positions to the client's caregivers b. post written step-by-step directions on the wall by the client's bed c. post pictures of the positions next to the headboard of the client's bed d. require each caregiver to demonstrate the replication of the desired positions .

c. post pictures of the positions next to the headboard of the client's bed. A visual representation can decrease any misinterpretation of verbal or written instruction. Requiring caregiver training for all would be helpful but it Is highly unlikely that the therapist would be able to access every personnel or caregiver providing care to this client.

When performing a chart audit for an on-site accreditation visit, an OT realizes that a date of service was documented wrong. Which are the best actions for the therapist to take? a. use whiteout to remove the incorrect date, write the correct date of service, and then initial and date the correction b. write the correct date over the incorrect date and then write the rehab director's initials c. put a single line through the incorrect date, write the correct date of service, and then initial the date and correction d. meet with the rehab department director to discuss the need to correct this documentation

c. put a single line through the incorrect date, write the correct date of service, and then initial the date and correction Medical charts are legal documents that cannot be altered without accountability. Therefore, the error found must be acknowledged with the date or correction and initials of the person making the correction.

An OTR is working with a family to determine an appropriate chair for a 6-year-old child with spastic diplegia cerebral palsy. The child has indicated a desire to begin independent toileting. Presently the child is able to walk with a reverse walker and can don and doff elastic pants and underpants. Which is the BEST support to help this child with independent toileting? a. special tomato seat b. bath chair c. raised toilet seat d. plastic reducer ring

c. raised toilet seat The client may need trunk support during toileting to prevent tipping. A raised toilet seat would help with this. A plastic reducer ring is used to prevent splashes and provides a small seating option for small children. In this case It may be too restrictive for this client.

An older teenager with a congenital right below-elbow amputation had never wanted a prosthesis before. Now the teen wants a prothesis "to look good at the prom and more 'normal' when doing things with his friends." Which action would be most beneficial for the OT to take to meet the client's expressed needs? a. recommend a prosthesis with a cosmetic passive hand b. recommend a prosthesis with a voluntary opening hook c. recommend a prosthesis with a myoelectrically controlled hand d. recommend counseling to explore the client's sudden preoccupation with body image

c. recommend a prosthesis with a myoelectrically controlled hand the best choice is a prosthesis that meets the teen's expressed need for a cosmetically appealing device which can also be used to perform functional age-appropriate bilateral fine motor activities such as texting or playing video games. A passive hand can grab items but cannot release it so it would not be the most functional choice. A hook would not respect the clients wishes for an appealing device, and recommending counseling would be judgemental.

An OT leads an outpatient wellness program. An individual with OCD asks for suggestions to manage symptoms that are interfering with life satisfaction. Which is the most beneficial recommendation for the therapist to make to the individual? a. approach activities in a nonchalant manner without high expectations b. engage in concrete activities that can be broken down into simple steps c. redirect thoughts and energies into meaningful activities d. set limits on the number of activities done in a day

c. redirect thoughts and energies into meaningful activities The focus of OT in a wellness program is to help individuals attain and maintain life satisfaction through the engagement in meaningful activities. Engaging in activities that can be broken down into simple steps is more helpful for individuals with cognitive deficits.

An OT working in a school must incorporate the individuals with Disabilities Education Act in the program. In which of the following locations should the therapist provide intervention? a. regular classroom while general education classes are not in session b. special education classroom while other children with disabilities are present c. regular classroom while general education classes are in session d. private OT room designed for children with disabilities

c. regular classroom while general education classes are in session The guidelines of IDEA say that children must be served inclusively and have full access to the general education cirriculum

An OTR is working with a child with cerebral palsy. The OTR facilitates the child's performance by physically or manually guiding movement. Although this manual guidance approach can guide selective attention and help the child organize and plan the movement, what principle is MOST IMPORTANT for the OTR to implement when using manual guidance with this child? a. use verbal instructions while physically guiding the child's movement b. provide light touch rather than deep pressure throughout the session c. remove physical guidance quickly decrease dependence on the technique d. never use manual guidance, it is not appropriate for a child with CP

c. remove physical guidance quickly decrease dependence on the technique Remove this type of guidance asap so that the child does not begin to rely on it. Using verbal instruction complicates the handling guidance. Deep pressure is more beneficial.

an individual who had a CVA one year ago continues to demonstrate unilateral inattention. The individual drives daily to therapy despite several suggestions from the OT to discontinue this activity. The therapist has determined that the client is an unsafe driver. What is the best action for the therapist? a. report the individual to the DMV b. suggest that the individual attends a driver training program c. report the information to the physician d. tell the individual's family that the client is at risk for injuring self and others while driving.

c. report the information to the physician The physician is the only person that can take action on a persons ability to drive. It would also be good to inform the family tho.

an individual with a SCI at the level of T-1 is practicing a stand pivot transfer in the OT department of a rehab center. The patient complains of dizziness and nausea. Which action is most important for the OT to take first? a. call for help according to facility procedures b. return the patient to the wheelchair for a 5 minute rest break c. return the person to the wheelchair and immediately recline it d. return the patient to the wheelchair and transport the patient back to rest in bed

c. return the person to the wheelchair and immediately recline it Individuals with a SCI are at risk for orthostatic hypotension. Complaints of nausea and dizziness are indicative of orthostatic hypotension. Reclining the individual will return the BP to a normal range.

a child with a tactile defensive sensory disorder attends a private early intervention clinic. The OT collaborates with the child's parents to develop strategies and guidelines to help the child handle symptoms of this disorder at home. Which is the best recommendation for the OT to make? a. avoid use of swings and other moving equipment during play activities b. encourage the use of swings and other movement equipment during play activities c. soften the child's clothing by repeated laundering and removing tags d. provide a variety of textures in the clothing the child wears

c. soften the child's clothing by repeated laundering and removing tags

a 10 y/o with congenital anomalies wears bilateral ankle-foot orthoses. The parents want the child to be able to don and doff shoes independently, but the child cannot tie shoes. Which is the best footwear recommendation for the therapist to make for the child to wear? a. leather slip-on loafers b. slip-on tennis shoes with no laces c. running shoes with Velcro shoe closures d. hi-rise sneakers with sliding adapters on the laces

c. running shoes with Velcro shoe closures running shoes are the best option for use with AFOs and Velcro closures will help the child to be independent until tying is learned. Leather slip-on loafers will not correctly support the AFO. Slip-on tennis shoes without laces don't have adequate support in the upper part of the foot to maintain the AFOs. The sliding adapters are a good option to replace laces, but the high-rise sneakers will not likely allow for placement of the AFOs on the feet.

The OT administers the Allen Cognitive Level screen to an individual. During this eval, the most complex behavior the individual can perform is to do the three running stiches, imitating the therapist's example. Based on these results, the therapist selects an activity to use for intervention that is consistent with the individuals functional level. According to the cognitive disabilities model, which activity is best for the therapist to use when implementing intervention with this patient? a. exercises that require the imitation of another's posture b. sorting laundry by matching the colors of clothing items c. sanding wooden bookends d. planning a 3-course meal

c. sanding wooden bookends According to the rating criteria, ability to imitate the running stitch indicates level 3 of cognitive performance. People at level 3 can use their hands to manipulate objects, and they are able to perform a limited number of simple tasks that are repetitive.

during a parallel task group, one of the members appears agitated and fidgety. The member gets up and looks out the window for a few minutes and then sits back down and quietly returns to the task. This behavior continues throughout the group. Which is the OTs best response when the group member stands again? a. tell the group member to remain seated or leave the group b. ask the other group members if the group member is bothering them c. say nothing to the group member and proceed with the group d. inform the group member that the observed behaviors indicate a lack of readiness for this group

c. say nothing to the group member and proceed with the group many individuals, due to the symptoms of their illness or medications have a difficult time sitting still. They can, however, benefit from attending a group. A parallel group does not require any interaction for task completion. Therefore, if the group members behavior is not disturbing or disruptive, he or she should be allowed to benefit from this form of treatment.

an OT provides consultation services to a nursing home and is developing a restraint reduction program. The therapist is also a faculty member in an OT professional education program. The therapist uses this consultation experience to explain to the students the role of OT. Which of the following is most accurate for the therapist to state this consultation experience represents? a. direct service b. primary prevention c. secondary prevention d. needs assessment

c. secondary prevention This involves the early detection of problems in a population that have diagnoses that place them at risk of developing complicating secondary conditions. A restraint reduction program's aim is to prevent the development of secondary conditions, such as deconditioning. Primary prevention targets individuals with no preexisting conditions.

an OT works with the new foster parent of a 2 y/o child diagnosed with major DDs and severe hypotonia. The therapist advises the foster parent to position the head in midline during feeding. Which additional positioning recommendations for feeding are best for this child? a. sitting with hips and knees at 90* flexion, neck in neutral b. sitting with hips and knees at 90* flexion, neck in extension c. semi-reclined with neck in neutral d. semi-reclined with neck in extension

c. semi-reclined with neck in neutral This position can be easily maintained with the use of a commercially available child seat and allows for correct postural alignment during the feeding activity. Feeding in a semi-reclined position is the least restrictive and most comfortable option for the child. It is not safe to feed anyone in neck extension because it can result in choking.

An individual with Parkinson's disease exhibits difficulty moving from sitting in a chair to standing. Which technique is best for the therapist to recommend the person use to help successful complete the functional mobility activity? a. rise from the chair while sitting with buttocks against the back of the chair b. extend both legs so that both feet are away from the chair while rising c. sit at the edge of the chair and rock back and forth before rising d. rise while weight-bearing on one foot and pushing up with both arms

c. sit at the edge of the chair and rock back and forth before rising One of the most common problems with Parkinson's is the initiation of movement. Rocking back and forth prior to moving from sit-to-stand provides the person with vestibular and proprioceptive input that can help facilitate movement.

A house painter is referred to OT after a re-occurrence of rotator cuff tendinitis. The physician prescribes a conservative intervention approach. Which recommendation is the best for the OT to make? a. continue performing above shoulder activities to build rotator cuff strength b. use an extension handle in the paint roller when painting ceilings c. sleep with the shoulder extended and adducted d. sleep with the shoulder fully flexed and adducted

c. sleep with the shoulder extended and adducted this is an acceptable position for this condition because all other activities and positions are contraindicated for people with rotator cuff injuries.

An OTR in the school system is implementing a transition plan for a group of developmentally disabled 17-year old students. The long-term goals for the students are gainful employment and supervised independent living in a group home. What activity BEST addresses the long-term goals related to transition for these students? a. cuts a circle, cuts a line, snips, cuts a square b. cuts on a line, snips, cuts a square, cuts a circle c. snips, cuts on a line, cuts a circle, cuts a square d. snips, cuts a square, cuts a circle, cuts on a line.

c. snips, cuts on a line, cuts a circle, cuts on a square Each of these tasks build on themselves.

The accountant asks the private practitioners to present their budget for anticipated direct expenses of their growing practice. Which is the most appropriate item to include in this budget request? a. the rent and utilities of the practice's primary office b. an integrated computer system for paperless documentation by all staff c. staff vacation and sick time d. supplies of items used in in-home therapy sessions.

c. staff vacation and sick time Direct expenses include costs related to OT service provision such as salaries and benefits. Vacation and sick time are benefits that must be budgeted. Supplies are considered a variable expense since it will change depending on the amount of services provided. Rent is a fixed expense, and utilities are indirect expenses.

The OT completes an intake interview for a work hardening program. As the individual is leaving, the person gives the therapist a hug and expresses much gratitude. The individual then tries to kiss the therapist on the lips. Which action is best for the therapist to take in response to the situation? a. forcibly push the individual away while telling the person that the behavior is inappropriate and unacceptable b. say nothing but decline the person's admission to the work hardening program based upon the person's inappropriate behavior c. state that the individual's behavior oversteps professional boundaries and makes the therapist uncomfortable d. tell the person the behavior is inappropriate and unacceptable and decline the person's admission to the program

c. state that the individual's behavior oversteps professional boundaries and makes the therapist uncomfortable Informing the person of the boundaries is an appropriate action. There is no need to admonish the individual or decline the person's admission to the program.

A school-age child who is right hand dominant complains of numbness and tingling after writing for more than 15 min. A neuro exam shows no reason for the numbness and tingling. Which action would be most beneficial for the OT to recommend? a. use a pencil held in a universal cuff to complete writing activities b. elevate the right UE at night and whenever possible during the day c. stretch the right UE every 15-20 min during writing activities d. use a custom -molded pencil grip made of splinting material when writing

c. stretch the right UE every 15-20 min during writing activities educate the child in active ROM and stretching to increase blood circulation and attempt to prevent the numbness and tingling. The cuff and grip do not address the numbness and tingling.

an OT conducts the preadministration screening for a supported housing program with several levels of care. The OT receives a referral for an individual with chronic schizophrenia, disorganized type, with residual symptoms of circumstantiality, flat affect, and decreased attention. Which screening tool is best for the therapist to use with this client? a. a semi-structured interview b. an ADL checklist c. a structured cooking task d. a weekly activity schedule

c. structured cooking task a structured cooking task can be used to screen for a diversity of cognitive skills and home management abilities. A semi-structured interview can be helpful in determining the person's interests and goals but it is not an effective screening for functional skill level. In addition, a person with circumstantiality may have difficulty providing useful answers to the question. An ADL checklist can assess knowledge of an activity but does not assess performance. A weekly schedule can provide info about an individual's time use and ability to complete structured tasks, but it is a paper and pen task that has limited applicability to screening for housing placement recommendations.

an OT steps in , at the last minute, to assist another therapist with a problem-solving skills group. Which is the most helpful action for the OT to take? a. split the group in 2 and have each therapist work with his/her own group b. participate as a member of the group and supply the desired responses c. support the leader with the comments and questions that keep the group on focus d. act as an observer and take notes for documentation

c. support the leader with the comments and questions that keep the group on focus The role of assisting a group leader is to facilitate participation of the members and the achievement of the goals of the group. Splitting members into 2 groups would result in the assisting therapist having no knowledge of the groups history, process, or goals. The benefit of receiving feedback from a co-leader is likely the precipitant for the group leader asking the therapist to participate. The other options do not provide any co-leadership benefits.

A child with myelomeningocele meets the short-term goals of achieving functional gross grasp and lateral pinch. After several additional weeks of OT, the child does not meet the goals of demonstrating pincer grasp and three jaw chuck. The therapist should modify the treatment plan to: a. splint the index finger in 30* PIP flexion and 30* DIP flexion to achieve pincer grasp. b. increase strength of lateral pinch as a basis to develop pincer grasp and three jaw chuck. c. teach the child to use gross grasp and lateral pinch for functional activities. d. teach the child to use ulnar grasp for functional activities.

c. teach the child to use gross grasp and lateral pinch for functional activities

An individual with an incomplete C6 SCI has a secondary diagnosis of thromboangitis obliterans. The OT conducts a pre-discharge home eval of the client's rented apartment. Which is the most important area for the therapist to asses? a. the apartment's electrical capacity for an environmental control unit b. the apartment's electrical capacity for an emergency call system c. the apartment's water temperature d. the landlord's willingness to modify the bathroom

c. the apartment's water temperature Thromboangitis obliterans, aka Buerger's disease, results in diminished temperature sensation, parasthesias, pain, and cold extremities. It is most common in young men who smoke. Poor or absent temperature sensation can be a serious safety risk. A person with a C6 SCI can independently access a phone to call 911. Structural bathroom modifications arent needed because this person would be able to bathe with Min Assist using a tub bench, slide board transfer, and handheld shower. None of these mods require a landlord's permission.

A child with autism receives home care OT services. The parents identify a primary goal of developing the child's independent toileting skills. The child is completely dependent and the parent reports not attempting toilet training for several years. The therapist collaborates with the parent to establish the first intervention goal for the child. Which behavior should this goal address? a. the child's ability to sit on the toilet b. the child's ability to verbally tell someone of the need to go to the bathroom c. the child's ability to indicate when the diaper is wet or soiled d. the child's ability to non-verbally indicate the need to go to the bathroom

c. the child's ability to indicate when the diaper is wet or soiled the first toileting skill that must be developed is the child's recognition of being wet or soiled. This typically occurs at 12 months. Other skills can develop after this skill.

An individual recovering from a flexor tendon repair surgery is 2 days s/p. The surgeon refers the client to OT with a prescription to use the Kleinert protocol to guide intervention. the OT meets with the client to ensure compliance with the prescribed splitting protocol. Which is the most important outcome of this session? a. the client's adherence to a written splint wearing schedule b. the client's ability to independently don and doff the splint c. the client's understanding of the purpose and procedures of the splint protocol d. the completion of functional training in the use of the splint

c. the client's understanding of the purpose and procedures of the splint protocol if the client understands the purposes and procedures of the splint protocol, he/she will become a collaborative partner in the interventions. The other choices are important components of a splinting intervention and may be required by accrediting bodies. However, the success of these interventions and attainment of compliance rely on the client's understanding of the splinting protocols proposes and procedures.

an individual with borderline personality disorder incurred a back injury while working as a stock person for a large warehouse. The individual attends a work hardening program. The OT evaluates the individual and determines that the client's level of productivity is just below the warehouse minimum standards. The individual complains of pain when lifting the heaviest boxes. The client frequently becomes angry and verbally abusive in response to directions or feedback. Which is the most important initial focus of the work hardening program for this individual? a. an increase in productivity to meet minimum standards b. an increase in productivity to exceed minimum standards c. the development of affective work behavior skills d. the development of strength and ergonomic lifting abilities

c. the development of affective work behavior skills. These behavior skills include social responsiveness, attitude toward the job, and relationships with supervisors and co-workers. The clients current behaviors put him at risk for not being able to maintain employment upon return to work. Increasing productivity and developing ergonomic lifting abilities and strength can be addressed during the course of the work hardening program, but the inappropriate behavior must immediately be addressed for the individual to benefit from the program.

An OT develops a protocol for a topical group to be implemented on an acute inpatient unit for individuals recovering from substance abuse. Which intervention focus is best for the OT to identify for this topical group? a. the improvement of self-esteem b. the development of coping skills c. the identification of leisure pursuits d. the practice of assertive behaviors

c. the identification of leisure pursuits A topical group is a verbal discussion group focused on a specific activity engaged in outside of the group. Identifying leisure activities that can be pursued in substance-free environment is a relevant focus for individuals on an inpatient unit who need to plan concretely for discharge. Improving self-esteem, developing coping skills, and practicing assertive behaviors are very broad and aren't related to a specific activity. Topical groups are activity focused.

an OT conducts a caregiver education workshop on positioning techniques for family caregivers. At the conclusion of the class, the caregivers will be expected to utilize the skills taught. Which is the most effective method for the therapist to use when teaching these techniques? a. therapist demonstration of general techniques followed by an individualized discussion with each caregiver. b. an oral multimedia presentation including powerpoint slides and handouts of positioning techniques for diverse disorders c. therapist demonstration of techniques followed by a lab with caregivers practicing on each other d. a Q & A session to address the specific individual positioning concerns of the caregivers

c. therapist demonstration of techniques followed by a lab with caregivers practicing on each other A variety of methods including demonstration, practice, and discussion would give the best chance for learning across the group. Only using oral teaching methods would not likely enable all participants to be successful. Psychomotor skills are best learned by practice.

an OT uses a motor learning intervention approach to develop prehension patterns with a child recovering from a brain tumor. The therapist places small toys and put them into a storage box that is also on the table. The therapist uses random practice during this activity. Which types and arrangement of toys are most effective for the therapist to provide according to the motor learning approach? a. toys that are exactly the same shape, size, and weight in a mixed arrangement on a table b. toys that are exactly the same shape, size, and weight in a straight line on the table c. toys of different shapes, sizes, and weights in a mixed arrangement on the table d. age-appropriate toys arranged in a developmental sequence according to the child's developmental age

c. toys of different shapes, sizes, and weights in a mixed arrangement on the table according to this approach, random practice involves the performance of several motor tasks in the random order to encourage the reformulation of the solution to the presented motor problem. Each time the child picks up a different toy, he/she will need to use a different grasp pattern. Having children pick up toys that are of the same shape, size, and weight involves repeated practice of the same motor skill. which is blocked practice. The motor learning approach does not use a developmental sequence.

A school-based OT is teaching orientation and mobility skills to an adolescent with a degenerative visual disorder. Which is the most effective motivational technique for the therapist to use with this student? a. provide concrete structure and frequent feedback to ensure accurate orientation and safe functional mobility. b. keep sessions short to allow time for emotional adjustment to orientation and mobility challenges c. treat the student as an adult and incorporate the student's orientation and mobility goals into intervention sessions d. limit anxiety by practicing the techniques in a quiet and self-contained environment.

c. treat the student as an adult and incorporate the student's orientation and mobility goals into intervention sessions Adolescents prefer to be treated as an adult. Too much structure will limit the student's trial and error learning which is vital to learning and retaining orientation and functional mobility skills. The length of the session should be determined by the student's established goals. Using a quiet room may be a good initial strategy, but it is not a motivational strategy.

An OTR just recently changed work settings from the acute care hospital setting to the school system. The OTR's caseload is mostly children with cerebral palsy and she desires to learn more about the research relevant to the area of practice but is very overwhelmed by the large volume of evidence available. What is the MOST beneficial method for the OTR to use to implement evidence in practice-based decisions? a. search scholarly databases for articles using levels of evidence b. test a variety of approaches in practice and analyze their effectiveness c. utilize systematic reviews and resources that synthesize study results d. subscribe to clinical blogs and LISTSERVs to stay up to date on current trends

c. utilize systematic reviews and resources that synthesize study results this is the best way to stay current on relevant research without getting consumed with weeding out unnecessary info. Scholarly searches based on levels will take a long time because there are up to 5 levels of evidence. Usually, blogs are opinion-based.

An OT provides OT services in a patient's room. The patient has left hemiplegia and is unable to recognize the faces of family members when they enter the room to visit or the items that the family members bring for the patient. The family members become upset by this behavior. Which deficit should the therapist explain to the family members as the most likely reason for the patient's behavior? a. ideational apraxia b. anosonosia c. visual agnosia d. asomatognosia

c. visual agnosia Visual agnosia is the inability to recognize familiar objects despite normal function of the eyes and optic tracts. Anosognosia is the inability to perform a purposeful motor act, either automatically or on command.

A person diagnosed with dementia, Alzheimer's type is evaluated by an OT. Although the client demonstrates diminished memory skills, the therapist determines that the patient is still able to live at home with supportive structure. The therapist collaborates with the client to identify activities to include in a structured routine that enables the client's continued occupational performance. Which activity is best for the OT to recommend to the client? a. cooking dinner b. doing laundry c. walking with a neighbor d. watching TV

c. walking with a neighbor this can meet the person's needs for social participation and exercise. This is an activity that can be safely pursued even with memory deficits. Cooking is unsafe because appliances can be left on. Doing laundry requires memory of multiple steps that would make this task difficult. Watching TV is safe but does not support use of intact abilities.

A single parent of 2 children, a 4 y/o and a 6 y/o, is referred to OT services in a rehab center. The client had a brain tumor removed 1 month ago. The referral states that the client has residual cognitive-perceptual deficits. Screening indicates that the client's sensori-motor abilities are WFL. The OT re-evaluates the client prior to discharge from the rehab center. The client exhibits residual cognitive deficits in problem solving. To develop requisite problem solving skills for independent functioning at home, which ability should the OT work on with the client during OT intervention? a. performing routine morning self-care b. making a shopping list of grocery staples c. washing the family's laundry d. reading a bedtime story to the client's children

c. washing the family's laundry Doing laundry can present a number of potential problems that must be solved such as stain removal and appropriate care for different textured/colored fabrics. The other tasks have less problem solving components.

a child with moderate arthrogyrposis complains of profuse sweating when wearing bilateral night resting splints. Which of the following should the OT suggest to the child? a. wear only one splint each night, rotating left to right b. wear volar cock-up splint c. wear a cotton stockinet liner under the splints d. wear a splint with several 1-cm perforations in the splinting material.

c. wear a cotton stockinet liner under the splints This will help absorb the sweat. Wearing only one splint does not address the sweating. The volar cock-up splints do not address the position of the MPs and IPs and might result in contractures. Perforations might decrease the strength of and integrity of the splinting materials.

an OT working in a private clinic receives a referral for a client who incurred a nerve laceration while working as a cable installer and repair person. Upon eval, the therapist determines that the client exhibits maximum motor and sensory losses consistent with radial nerve laceration below the supinator. the cliet successfully completes the work-hardeining program. However, the client has residual moderate impairment in temperature perception. During the discharge planning session, the therapist discusses how this impairment may impact areas of occupation and suggests activity modifications to facilitate the client's occupational performance. what is the most appropriate recommendation for the therapist to make to the client? a. request reassignment to work activities that do not involve exposure to extreme temperatures b. mark all potentially hot objects at home and work with bright stickers c. wear work gloves for activities involving extremes or variations in temperature d. wear a protective splint during the work day and at home during maintenance tasks.

c. wear work gloves for activities involving extremes or variations in temperature the client should wear gloves because of the danger of incurring a burn due to diminished temperature sensation. The client's job will frequently require him to work in extreme temperatures so it may not be reasonable to ask to be reassigned.

a 4 y/o child with a complete myelomeningocele at the T12 level is referred for outpatient OT and PT. In setting goals with the family, the OT suggests increasing independence in dressing skills and the PT recommends improving ambulation. The family wants to work on ambulation but refuse interventions to develop dressing skills. Which action is best for the OT to take first to address the family's stated preference? a. concur with the family that the child does not need to receive OT services b. refer the family to counseling to assist them in accepting the child's functional limitations c. work with the family to determine a different focus for OT intervention d. reinforce the importance of independence in dressing for the child to the family

c. work with the family to determine a different focus for OT intervention this addresses the child's and family's rights and allows the therapist to use therapeutic use of self. The family's decision should be respected, but the child will have other occupational needs that can be worked on. There is no information to indicate the need for a counseling referral.

An OTR is working with a family in an early intervention setting. The mother reports that her child is having difficulty transitioning from the bottle to the cup and often pushes the cup away. What are the MOST appropriate interventions to use INITIALLY to prepare the child for cup-drinking activities? a. ignoring these behaviors and redirecting the child without focusing on the disruptive behaviors b. working oral-sensory input such as massage or vibration intermittently throughout the day c. working on jaw stability, lip closure, tongue movements, and oral sensitivity through positioning, handeling, and oral motor activities d. experiment with thickening liquids to see if it eases the transition and increases nutritional intake

c. working on jaw stability, lip closure, tongue movements, and oral sensitivity through positioning, handeling, and oral motor activities These activities will provide stability for the child to maintain adequate head alignment and enhance proper liquid management to eventually allow the child to drink safely from a cup. Ignoring these behaviors is appropriate when dealing with children who have selective eating or food refusal and need behavioral management. Oral-sensory input is for children with sensory processing disorders. Experimenting with thick liquids may occur later in therapy if other activities do not work.

Upon evaluating a client for a wheelchair, the OT determines that a standard narrow adult chair would be suitable for the individual. The dimension of this chair will be: a. 18" wide x 18" deep x 20" high b. 16" wide x 16" deep x 18.5" high c. 14" wide x 16" deep x 18.5" high d. 16" wide x 16" deep x 20" high

d. 16" wide x 16" deep x 20" high A regular chair is 18" wide x 16" deep x 20" high, and a slim adult chair is 14" wide x 16" deep x 20" high.

An OT measures a person for a wheelchair. The widest point across the person's hips and thighs is 16 inches and the greatest length from the person's posterior portion of the buttocks to the popliteal fossa is 18 inches. Which wheelchair seat dimensions should the therapist recommend. a. 18 inches wide by 20 inches deep b. 18 inches wide by 18 inches deep c. 16 inches wide by 18 inches deep d. 18 inches wide by 16 inches deep

d. 18 inches wide by 16 deep You must add 2 inches to the width of the hips and subtract 2 inches from the popliteal fossa to prevent skin breakdown.

An OT working for a partial hospital program arranges the seats for a topical group. The room is small and there are 8 people in the group. 2 of the members have been diagnosed with schizophrenia, paranoid type. Which is the best arrangement of the room for this group composition? a. 8 chairs placed around a table, with no assigned seats and the therapist at the front of the room b. 9 chairs placed in a circle, with assigned seats and the therapist in front of the room c. 2 semi-circles of 4 chairs facing the therapist, with the 2 patients with schizophrenia in the front row d. 9 chairs placed in a circle, with no assigned seats for the patients or therapist

d. 9 chairs placed in a circle, with no assigned seats for the patients or therapist in groups, members are typically more comfortable if they are allowed to sit where they wish. This is respectful of their choices and autonomy

An OT provides home-based services to a homemaker who incurred a right CVA 8 months ago. The individual and the therapist have chosen to focus on kitchen activities during the intervention session. The therapist has the client stand in front of the counter with an open dishwasher to the left. The therapist asks the client to put the clean dishes into an overhead cabinet to the right of the client using the affected UE. By setting up the activity in this manner, which proprioceptive neuromuscular facilitation technique is the therapist using? a. heavy work/mobility superimposed on stability b. reciprocal inhibition/innervation c. diagonal patterns of D2 flexion/extension d. diagonal patterns of D1 flexion/extension

d. Diagonal patterns of D1 flexion/extension The D1 flexion pattern moves the person's left UE "up and away" as the person moves the dishes from the dishwasher to the cabinet. D2 is the PNF extension diagonal which moves the UE "down and in"

An OTR is working with a classroom teacher in developing seating options for a student who requires additional proprioceptive input to fully engage in school-based tasks. The child is most challenged during circle and calendar (when children are seated on the floor) and has difficulty participating which often results in negative behaviors. What is the BEST option the OTR might recommend? a. corner chair b. infant sitter b. cube chair d. Howda Hug chair

d. Howda Hug chair This cahir creates a snug feeling for the user and allows the user to calm and focus on educational tasks. A corner chair prevents children from falling over when leaning, an infant chair is from smaller frame children, and a cube chair provides an organized workspace

an individual presents with intention tremor, dysmetria, decreased equilibrium, and nystagmus caused by a cerebellar lesion. The person expresses difficulty with routine tasks. Which intervention is best for the OT to provide? a. a cone and pegboard activity with wrist weights to control tremors b. quick stretch to lateral trunk muscles during functional activities c. a power wheelchair to prevent falls during routine activities d. UE weight bearing during self-care activities at the sink

d. UE weight bearing during self-care activities at the sink treatment goals for persons with cerebellar dysfunction are focused on strengthening proximal muscles., improving postural responses, and increasing stability. Weight bearing can increase shoulder girdle stability.

A client recently diagnosed with tardive dyskinesia reports that she has increased difficulty getting dressed in the morning due to rapid involuntary movement in her hands. Which assistive device should the OTR INITIALLY recommend for this client? a. zipper pull b. button hook c. reacher d. Velcro straps

d. Velcro straps these can replace shoelaces and other small fasteners for clients with diminished fine motor coordination. A zipper pull and button hook still requires some fine motor skill. This person would need coordination to squeeze the rigger on a reacher.

an elementary school-aged child incurred burns, lacerations, and a femur fracture in an MVA and was hospitalized for 3 weeks. During outpatient OT sessions, the parent expresses concern that the child has begun wetting the bed daily. Which professional is most relevant for the OT to advise the parent to contact? a. a rehab nurse to develop a bladder control program b. a child life therapist for intervention for DD c. a psychiatrics OT for behavior management strategies d. a clinical social worker to explore potential regression issues

d. a clinical social worker to explore potential regression issues The child has no reported DD and the injuries reported would not have likely affected the genito-urinary system. Therefore the symptom of bed wetting is likely a symptom of regression

The transition plan for a high school senior with DD includes a referral to a vocational rehab workshop job setting. The student has set a goal to live independent of family. Which is the best living environment for the OT to recommend? a. an apartment in a subsidized housing project b. a group home with case managers available on-call c. a supported apartment with a roommate d. a group home with daily on-site supervision

d. a group home with daily on-site supervision This person has likely relied on family for help with various tasks at home. It is important that they have supervision and assistance when they need help. As they learn to become more independent they could move to a group home were case managers are on call.

a patient is recovering from a right CVA resulting in severe left hemiplegia and visuospatial deficits. The person's left LE has pitting edema. Which wheelchair would be best for the OT to recommend? a. A powered wheelchair with a joystick control and dual elevating leg rests. b. a lightweight active duty wheelchair with dual elevating leg rests c. a 1 arm drive chair with an elevating leg rest on the left d. a hemiplegic chair with an elevating leg rest on the left

d. a hemiplegic chair with an elevating leg rest on the left Patients can propel these types of chairs with their unaffected arms and legs. there is no need for an elevating leg rest on the right side and a powerchair is unnecessary. A 1 arm drive chair is contraindicated for patients with cognitive or perceptual deficits as it can be confusing to learn to propel accurately.

an elementary school teacher has been recently diagnosed with MS. which adaptation is the best for the OT to recommend to the teacher to accommodate for the effects of MS on classroom teaching? a. large print written material to compensate for visual impairments b. a daily list of tasks to compensate for cognitive deficits. c. a motorized scooter to compensate for decreased performance. d. a high stool to compensate for lower extremity weakness

d. a high stool to compensate for lower extremity weakness lower extremity muscle weakness is common in the early stages of MS. A high stool would allow the teacher to sit while maintaining visibility over the classroom.

a young adult recently diagnosed with depression and anorexia nervosa is a consumer of services at a psychosocial clubhouse. The client attends individual OT sessions once a week and several evening and weekend groups. When completing an activities schedule and questionnaire, the client identifies the clubhouse groups and part-time work as an editorial assistant as the only activities completed each week. In high school the client was captain of the swim team, played tennis, and worked in an after-school activities program for young children. The client expresses interest in the pursuit of additional activities outside of the clubhouse and work. Which is the most appropriate avocational resource for the therapist to recommend that the client explore? a. a local fitness center for exercise classes b. the town swimming pool for open swimming sessions c. the area soup kitchen for volunteer opportunities d. a local community center for volunteer opportunities

d. a local community center for volunteer opportunities This suggestion can support the client's altruistic abilities while providing a diversity of potential activity pursuits. The other options can be contraindicated for people with anorexia because they often engage in these activities in excessive (sometimes abusive) quantities.

an individual with myasthenia gravis is being discharged home after a hospitalization for the treatment of pneumonia. The person's spouse has expressed concern about caregiving responsibilities and the client's ability to function in the home. Which is the most beneficial recommendation for the OT to make to address the spouse's concerns and the client's needs? a. the extension of the client's length of stay to allow for caregiver training b. the extension of the client's stay to provide intervention to develop the ADL skills c. a referral for the client to an adult day care program to relieve caregiver stress and to develop functional skills d. a referral to a home care agency for a functional eval and home assessment

d. a referral to a home care agency for a functional eval and home assessment This is the best choice to provide accurate info about the client's functional status and caregiver needs. An extension of length of stay is difficult to justify because the individual was hospitalized for the medical treatment of pneumonia. Once this illness is effectively treated, discharge must occur. Additionally, it is more effective to provide caregiver training in the home.

A sensory profile completed by a caregiver indicates that an elementary school-aged child has modulation impairments and sensation seeking patterns. The OT observes the child frequently wandering, bumping into objects in the room and fidgeting. Which is the best intervention approach for the therapist to use to address this child's deficits? a. strategies to increase random sensory input and encourage high physical activity at home b. an obstacle course which requires diverse movements for varied proprioceptive and tactile input c. sensory experiences on body awareness ad grading control during play activities d. a sensory diet which includes controlled sensory input integrated into the child's daily routine.

d. a sensory diet which includes controlled sensory input integrated into the child's daily routine Because these children create sensation for themselves, their behavior tells us what sensory input they need. Children who rock and fidget require vestibular input to help them attend and learn. Children with proprioceptive problems often rely on visual or verbal cues to know how to move their bodies, and they often appear clumsy, bumping into objects in their environment. Its important to provide caregivers with a written sensory diet to implement on a daily basis in the child's home and school environments to facilitate carryover.

a 5th grade student is having difficulty self-feeding and frequently spills beverages. The student has non-spastic CP resulting in fluctuating muscle tone and poor motor control. Which is best for the OT to recommend the student use to improve independence in self feeding and minimize spillage? a. a sippy cup b. both hands to hold the cup c. a cup with a handle d. a straw

d. a straw this is an easy and age-appropriate modification to prevent spills. A sippy cup is developmentally inappropriate for this age. Fluctuating muscle tone will result in random, involuntary contractions and relaxation of the muscles making purposeful movements difficult. Using both hands or using a cup with a handle would not prevent spills secondary to fluctuating muscle tone.

a 7 y/o with complete spina bifida at the T10 level attends outpatient OT weekly. The child's parent reports that the child is losing bladder control. Upon re-evaluating the child, the OT notes that the child shows a minimal decrease in strength of bilateral lower and upper extremities and an increase in the equinovarus position of the feet. The therapist contacts the physician to report that the child's change in status may indicate which of the following? a. shunt malformation b. a recent growth spurt c. Arnold-chari formation d. tethered cord

d. a tethered cord this is noted by all of the symptoms listed. The spinal cord of the child with spina bifida is sometimes attached to the spinal column and becomes taut as the child grows. Shunt malformation would be marked by intermittent headaches, short attention span, increased paralysis, decreased UE strength, noticeable decrease in school performance, and increased irritability.

the private practitioners meet with their accountant to plan fiscally for their expanded practice. How would the accountant classify the fees that the practice receives from their early intervention contract? a. accounts payable b. capital assets c. productivity standards d. accounts receivable

d. accounts receivable accounts receivable are payments received by the program or institution. These are assets in a budget.

A SNF wants to provide an activities program to meet the needs of each resident. The OT supervisor has been asked to write a job description for the position of an activities program director and delineate the qualifications needed for that position. What should the OT recommend for the position? a. an OTA who receives general supervision from an OT b. an OTA who receives close supervision from an OT c. an OTA who receives routine supervision from an OT d. an OTA who does not require supervision from an OT

d. an OTA who does not require supervision from an OT according to AOTA standards of practice, an OTA who works strictly as an activities program director is not providing OT, therefore they do not require supervision of an OT.

An OT working in a SNF observes a resident with cognitive disabilities don slippers by putting them on the wrong feet. The resident plans to go visit a friend on another floor and does not seem aware that the slippers are on the wrong feet. Which is the best action for the therapist to take in response to this situation? a. say nothing for this error may embarrass the resident b. say nothing but follow the resident to the friend's room to ensure a safe arrival c. ask the resident to look at the slippers to see if the error is noticed d. approach the resident and advise the resident to reverse the slippers

d. approach the resident and advise the resident to reverse the slippers the slippers must be immediately reversed to ensure safety and prevent a fall. Poor or inappropriate use of footwear is a primary cause of falls. Letting the resident walk with slippers on the wrong feet is unacceptable. Since the resident has cognitive deficits he/she may not be able to notice the error and self correct.

An OT works in a school setting with adolescents with Asperger's syndrome. The need for a social skills training group is identified. 1 activity that the therapist plans to use in the group is roleplaying. Which is the most effective way for the therapist to determine relevant scenarios for the role-play activities? a. survey the teachers on social difficulties displayed in class b. survey parents on social difficulties they have observed in the adolescents c. review literature on adolescent social skill development d. ask the group members about their social concerns

d. ask the group members about their social concerns this is the most client centered answer. Surveying others can be helpful, but it may also not be an accurate reflection of the individuals' needs.

An individual recovering from an exacerbation of MS is referred by a primary care physician to an outpatient OT clinic. The referral requests that the OT complete a functional capacity eval. Which is the most likely reason for this referral? a. determination of cognitive level b. determination of disability status c. assessment of IADLs d. assessment of return to work capabilities

d. assessment of return to work capabilities

An OT assesses a retired carpenter and war veteran who was newly admitted to an acute psychiatric unit due to an exacerbation of post-traumatic stress disorder. The individual is observed to initiate interactions with others during structured tasks but is withdrawn when not engaged in a task. What would be the best recommendation for the OT to make? a. write in a daily journal his thoughts and feelings b. engaging in meditation when alone in the hospital room c. write about his experiences in unit's newspaper d. assist another patient with a woodworking project.

d. assist another patient with a woodworking project this uses his carpentry skills and facilitates the curative factor of altruism. This will allow him to socialize. Self-expressive activities and meditation can be very helpful for many people, but they can be extremely stressful for people experiencing exacerbations of PTSD. Focusing on traumatic events are contraindicated during acute episodes.

An OT provides caregiver education to the family of an individual who is dependent in all self-care. During instruction on proper wheelchair positioning, where should the therapist advise the family to place the wheelchair seatbelt? a. at waist level b. midway between waist and trunk c. at the widest point of the individual's midsection d. at hip level

d. at hip level wheelchair seat belts are to extend across the hips and into the lap at a 45* angle.

An older adult recovering from an MI is referred to OT for a home care eval. The referral states that the client has high BP and medication-related orthostatic hypotension. Which precaution is most important for the OT to observe with this client? a. adherence to dietary restrictions during meal prep activities b. avoidance of activities that require movement against gravity c. delay of the OT eval until the client's medication is stabilized d. avoidance of activities that require sudden postural changes

d. avoidance of activities that require sudden postural changes OH is an excessive drop in BP that occurs upon assuming an upright position. All functional activities have components that occur against gravity so these cannot be avoided. Adherence to dietary restrictions during meal prep is important, but the question is about an evaluation session OT an intervention.

An OT is providing intervention for an individual recovering from a CVA who has residual body neglect. The therapist is using a deficit-specific approach to intervention. Which type of activities is best for the therapist to use with this client? a. unilateral using the affected UE b. unilateral using the non-affected UE c. tasks that require right/left discrimination d. bilateral using both UEs

d. bilateral using both UEs a basic principle of intervention for neglect, according to a deficit-specific approach, is to provide bilateral activities. During these activities, the therapist can guide the affected UE trough the activity. The unilateral activities do not work on the identified deficits.

an adult with arthritis of both hands has ulnar drift of myofascial pain syndrome during finger extension and flexion and at rest. The patient also has a lengthening of the central slips of the extensor digitorum communis tendons of the right index and middle fingers. Which of the following should the OT report the person is exhibiting? a. swan-neck deformities b. trigger finger deformities c. MP palmar sublux-dislocations d. boutonniere deformities

d. boutonniere deformities this is caused by a lengthening or rupture of the extensor digitorum communis tendons and is expressed by DIP hyperextension and PIP flexion. A swan-neck deformity can result from the rupture of the lateral slips of the extensor digitorum communis or flexor digitorum superficialis tendon and results in DIP flexion and PIP hyperextension.

an OT conducts an after school transition skills group with adolescents with a diversity of disabilities. One of the student's behavior is very different than in prior groups (e.g., difficulty focusing when typically serving as the group initiator, making statements that are irrelevant to the topic at hand). The therapist smells a strong alcohol sent on the student's breath. Which action should the therapist take in response to this situation? a. directly ask the student if he has been drinking b. initiate a group discussion about the effects of substance abuse on occupational performance c. proceed with the group and report suspicions of alcohol use at the next transition planning team meeting d. call for a school aide to escort the student to the school's onsite health care facility

d. call for a school aide to escort the student to the school's onsite health care facility OTs must ensure the student's safety. Thee medical staff can evaluate the student to determine the cause of the student's atypical behavior and alcohol smelling breath. These symptoms can be the result of alcohol use or an indication of ketoacidosis.

a parent receiving OT in a hand clinic asks the OT to adjust his/her child's hand splint. The parent reports that the splint leaves red marks after being removed for 20 minutes. The child received the splint from the school therapist. Which is the best action for the therapist to take in response to this request? a. modify the splint using a heat gun b. suggest that the parent have the child refrain from wearing the splint c. have the pediatric therapist in the department modify the splint d. call the child's therapist and have the parent express the concern

d. call the child's therapist and have the parent express the concern the best choice is to have the parent directly speak to the child's therapist. In the other choices, the therapist is providing treatment without a referral which is not in accordance with practice standards, and acting without knowledge of the diagnosis or plan of care would be unethical.

an elderly person has lost significant functional vision over the last 4 yrs and complains of blurred vision and difficulty reading. The patient frequently mistakes images directly in front, especially in bright light. When walking across a room, the patient is able to locate items in the environment using peripheral vision when items are located on both sides. Based on these findings, which visual deficit should the OT report the client is exhibiting? a. glaucoma b. presbyopia c. hemianopsia d. cataracts

d. cataracts Cataracts are a clouding of the lens which results in a gradual loss of vision. Central vision is lost first, then peripheral. Glaucoma presents the reverse symptoms: loss of peripheral vision (tunnel vision) and then central progressing to total blindness

an elementary school student is referred to OT to improve fine motor skills. The student is having difficulty drawing simple shapes (circle, square, & triangle) and cutting a straight line with scissors. Which of the following activities is best to help develop arches of the hand and increase hand strength to improve the student's ability to grasp a writing utensil or scissors. a. spell vocabulary words using 3D letter shapes b. identify simple shapes with vision occluded c. complete a puzzle lying prone on the floor d. chair push-ups before table top activities

d. chair push-ups before table top activities weight bearing activities, such as chair push-ups, help develop the arches of the hand and strengthen the intrinsic muscles of the hand to improve fine motor activities. Using 3D shapes and identifying shapes with vision occluded do nothing to help this. Lying on the floor to complete a puzzle will provide weight bearing through the shoulder and increase shoulder stability, but does not develop arches of the hand or increase strength.

The OTR is working with an adolescent, 16-year-old girl, diagnosed with depression. Her primary area of concern is socialization with her peers. Her strength is in the area of task skills. What is the BEST therapy goal to address INITIALLY for this client? a. client will increase socialization with peers by participating in task groups b. client will select and teach two peers a craft activity which she enjoys c. client will be provided with the opportunity to interact with peers while she demonstrates a familiar craft activity. d. client will select and teach a craft activity to the OTR and state two positive comments about her instruction

d. client will select and teach a craft activity to the OTR and state two positive comments about her instruction

an OT observes an OTA having difficulty transferring a client with athetoid movements from a mat to a wheelchair. Before the therapist can cross the room to help with the transfer, the OTA slides with the client to the floor. The therapist assists the OTA in safely returning the client to the wheelchair. They assess that the client appears to be unharmed and return the client to the unit for medical eval. Which action should the therapist take next? a. counsel the OTA on the need to ask for assistance with difficult transfers b. require the OTA to attend a transfer training in-service c. document the OTA's unsafe actions in a personnel file d. complete an occurrence report according to facility standards

d. complete an occurrence report according to facility standards immediately after an incident occurs, the therapist must complete documentation according to the settings standards. Counseling the OTA to ask for assistance and requiring attendance at a workshop can be appropriate aspects of risk management, but they are not the first steps.

a middle-school aged child with osteogenesis imperfecta reports feelings of low self-esteem, social isolation, boredom, and lethargy. the OT collaborates with the child to identify resources for after- school leisure activities to promote socialization and community participation. Which of the following activities is best for the therapist to explore with the child? a. team sports b. therapeutic horseback riding c. scouting programs d. computer clubs

d. computer clubs osteogenesis imperfecta results in brittle bones that fracture easily. The other occupations that involve more physically based activities would be difficult for the child to safely pursue.

In an outpatient rehab clinic, an OT is treating a high-school student with spina bifida resulting in full motor paralysis and sensory deficits below L1 spinal cord level. The client is a competitive swimmer and is able to transfer independently from the wheelchair to the pool without an assistive device. The client's goal is to learn how to mount and ride a horse. Which is best for the therapist to do initially to help the client attain this goal? a. implement a home-based transfer training program for the client's family to learn how to assist with mounting a horse b. develop an exercise program for the client to do at home on a daily basis to increase upper extremity strength. c. encourage the client to extend scheduled swimming sessions and practice mounting large-inflatable tubes in the pool d. consult with a stable owner to discuss alternative methods of mounting a horse

d. consulting with a stable owner to discuss alternative methods of mounting a horse consulting with a stable owner would be the best initial option. Based upon this info, the therapist can plan appropriate intervention. Since the client is a competitive swimmer and can transfer independently to the floor, UE strength is not a problem. Activity demands of mounting large inflatable tubes are not the same as those of mounting a horse.

An OT provides bed mobility training for an individual recovering from a left CVA. The therapist notes that the person's right calf is swollen and warm. The person complains that it is painful. which action should the therapist take initially? a. elevate the leg and provide retrograde massage b. advise the person to tell the physician about the symptoms during the physician's next bedside visit c. continue with the training and document the symptoms in the medical record d. contact the charge nurse immediately to report the symptoms

d. contact the charge nurse immediately to report the symptoms The signs are indicative of DVT. This is a medical emergency that must be handled immediately by the medical staff. Elevation would be appropriate but retrograde massage is contraindicated. The other answers delay the acquisition of medical care.

An older adult with Parkinson's disease has a secondary diagnosis of depression. During the home based OT service provision, the client exhibits increased confusion and disorientation. Which action is best for the home-care OT to advise the client's primary caregiver to take in response to the client's change in status? a. obtain an eval from a neurologist to determine if dementia has begun b. seek support from a caregiver's respite program c. discuss placement alternatives with the social worker d. contact the client's physician to request medical eval

d. contact the client's physician to request medical eval. If the physician finds no physical reason for the clients change in status, he can request a psychiatric consult. Depression can result in these symptoms, therefore, the first action should be to consult with the doctor. It is premature to seek respite.

a person with arthrogryposis undergoes serial casting with weekly cast changes of the right wrist. Upon cast removal during the fourth week, the therapist notes a small open area 1/4 cm by 1/4 cm and a red rash over the ulnar styloid. Which is the therapist's best response to these observation? a. pad the area and apply another b. refer the individual to the wound care team c. fabricate a static splint that doesn't impede on the ulnar styloid d. contact the physician and describe the observations

d. contact the physician and describe the observations The physician needs to know so he/she can make a decision whether to recast, dress the open area, refer to the wound care team, fabricate a new splint, or employ something else.

an older adult diagnosed 3 years ago with dementia, Alzheimer type has been admitted to the hospital for regulation of medication. The OT determines that the patient demonstrates diminished memory skills since the previous eval, but is still able to live independently at home with support and supervision. During the discharge planning meeting, which activity should the therapist recommend family members perform for the patient? a. weeding the garden b. sorting and folding the laundry c. preparing cold sandwiches d. cooking hot meals

d. cooking hot meals Cooking hot meals is a potential safety risk for a patient with decreased memory. Other activities can be continued with supervision.

A OTR is working with a child with autism spectrum disorder on improving engagement and interaction. Which intervention strategy is the MOST BENEFICIAL to improve these skills with this child? a. provide opportunities for deep pressure and proprioception b. provide clear boundaries regarding the intervention space and activities in it c. promoting imagination in the OT sessions (ex: what would an animal look like if it were part monkey and frog) d. creating fun problems to be solved (ex: a toy bear is trapped in quicksand- how do we save him?)

d. creating fun problems to be solved (ex: a toy bear is trapped in quicksand- how do we save him?) Providing deep pressure is used for improving comfort and reducing fear, not improving engagement. Clear boundaries are used to improve behavior and task completion. Promoting imagination is used to improve play.

the family of an individual being admitted to a rehab center offers the OT a cash gift. The therapist refuses the money but the family continues to insist that the therapist take the cash gift. After thanking the family for the generous gesture, which is the therapist's best response? a. donate the money to the hospital b. donate the money to charity c. use the money to purchase an item for the OT department d. decline the gift

d. decline the gift the other answers are not in accordance with the OT code of ethics.

an OT working in a school has been asked to recommend technological devices for a student with severe spastic quadriplegia and dysarthria. Which action should the therapist take prior to recommending specific equipment? a. determine access capabilities in collaboration with the SLP b. identify funding sources in collaboration with the social worker c. obtain family support in collaboration with the psychologist d. determine the intervention goals in collaboration with the student.

d. determine the intervention goals in collaboration with the student. this is essential so that all goals are meaningful and relevant to the clients needs and desired goals.

An OTR is working with a child in elementary school experiencing extreme poverty and homelessness. Which intervention strategy BEST describes a tier 2 intervention for this population? a. analyze the child's specific needs and collaborate with mental health team to coordinate care for those intensive needs b. provide a school-wide in-service training on adverse childhood effects and its effects on learning c. provide tips for promoting mental health and successful functioning in school d. develop and run groups to foster resiliency and social engagement and participation

d. develop and run groups to foster resiliency and social engagement and participation Tier 1 interventions are universal, Tier 2 are for some students in small, at risk groups, and Tier 3 interventions are intensive, assessment based for individuals. Answer A is a tier 3. and B & C are tier 1 interventions.

An OT working on an acute psych inpatient unit conducts a series of groups for clients newly admitted to the unit. Which group leadership style is the most effective for the therapist to assume when leading these groups? a. advisory b. facilitative c. laissez-faire d. directive

d. directive this involves structure, clear directions, and immediate consistent feedback. These qualities are needed in a group whose members are acutely ill with psych disorders.

an individual recently lost significant functional abilities due to post-polio syndrome. The OT works with the individual to develop compensation skills for performing daily tasks. During a meal prep session, the person angrily throws all of the AE onto the floor. At the next team meeting, which defense mechanism should the therapist report the individual seems to be reporting? a. acting out b. passive-aggressive behavior c. reaction formation d. displacement

d. displacement This occurs when an individual redirects an emotion from one "object" to another. In this case the anger over the progression of the disease was taken out on the AE. Acting out is a term used to describe behaviors that violate social norms. Reaction formation is the switching of an unacceptable impulse into its opposite. (ex: hugging someone you want to hit).

an OT working in a private clinic receives a referral for a client who incurred a nerve laceration while working as a cable installer and repair person. Upon eval, the therapist determines that the client exhibits maximum motor and sensory losses consistent with radial nerve laceration below the supinator. through active participation in individual OT sessions, the client regains functional motor skills. To facilitate return to work, the OT refers the client to a local work-hardening program. This program does not have the exact equipment that the client uses in the job setting. Which action is best for the therapist at the work-hardening program to take in response to this situation? a. refer the client to another work-hardening program that has the equipment b. order and install the equipment necessary to duplicate the work setting c. perform some necessary aspects of rehabilitation in the client's work setting d. duplicate the job task components as closely as possible

d. duplicate the job tasks as closely as possible Work-hardening programs use real or simulated tasks that duplicate as closely as possible the components of each job task. It is not realistic for the program to have all necessary equipment. Therefore, there is no reason to refer this person to another facility.

An adult is hospitalized in the recovery phase of Guillain-Barre syndrome. The patient complains of tingling, aching, and weakness in both hands, causing difficulty in grasping grooming supplies. The patient requests relief from hand symptoms. Which action should the OT take to address the patient's concerns? a. provide soft tissue massage to both hands prior to grooming activities b. apply hot packs to both hands and complete stretching exercises prior to grooming activities c. refer the cline to a neurologist for follow-up of possible condition regression d. educate the patient about sensory deficits and related adaptive ADL strategies

d. educate the patient about sensory deficits and related adaptive ADL strategies

an older adult who is recovering from a CVA attends OT twice per day. The intervention environment is highly structured and not over-stimulating, yet the OT observes that the client's mood often changes abruptly. Within 1 session, the client will laugh and then become tearful with no apparent precipitant. Which is the most accurate for the OT to document the client is exhibiting in the client's daily progress note? a. early Alzheimer's b. anhedonia c. a response to auditory hallucinations d. emotional lability

d. emotional liability this is often observed in people recovering from CVA. anhedonia is the inability to experience pleasure.

a young adult recently diagnosed with depression and anorexia nervosa is a consumer of services at a psychosocial clubhouse. The client attends individual OT sessions once a week and several evening and weekend groups. When completing an activities schedule and questionnaire, the client identifies the clubhouse groups and part-time work as an editorial assistant as the only activities completed each week. In high school the client was captain of the swim team, played tennis, and worked in an after-school activities program for young children. the clubhouse members attain a level of cohesion which enables them to perform at a cooperative level. Two members of a community integration group disagree with the others on the details of a group project. How should the OT leading this group respond to this conflict. a. clarify all viewpoints and facilitate the members in decision-making b. listen to all viewpoints and suggest that members vote to determine the project details c. mediate only when the members have reached a deadlock situation d. encourage the members to explore alternative methods to resolve the conflict

d. encourage the members to explore alternative methods to resolve the conflict in a cooperative group, the therapist acts as an advisor. Group members are mutually responsible for giving feedback and meeting group needs. The OTs intervention should facilitate group problem solving rather than direct a course of actions or decisions. Waiting until the group has deadlocked is not beneficial to the group cohesion.

An OT is working with a preschool student who was born with congenital Cytomegalovirus infection. As a result, the child has difficulty seeing. The child enjoys playing with classmates but has difficulty when the play activity is highly dependent on vision. Which of the following actions is best for the therapist to recommend for improving the child's play experiences with classmates? a. train a personal assistant to provide verbal cues during play activities b. have the child read books in Braille aloud to the classmates c. train a classmate to guide the child during play activities d. incorporate 3D objects into play activities

d. incorporate 3D objects into play activities This enables the child to use tactile senses. Strengthening the child's other senses is an effective intervention to compensate for low vision.

An OTR is making an early intervention home visit to a family in a rural community. The grandmother, mother, and mother's sister are present during the session, and the grandmother is bottle-feeding the 8-month-old infant when the OTR arrives. The OTR's goal for the child is to work with the family on further developing the child's motor skills. What INITIAL action should the OTR take in the session with this family? a. wait until the child finishes eating for the mother to begin therapy with the child in order to work on motor skill development b. re-schedule the child's therapy session due to concern that motor activities are contraindicated after bottle feeding c. focus the treatment session on feeding skills to incorporate the natural routine and habits of the family d. engage the caregivers in conversation about the child and progress made since last visit

d. engage the caregivers in conversation about the child and progress made since last visit waiting will waste precious time, re-scheduling is not necessary, and feeding is not the child's therapy goal so it would not be client-centered or ethical.

An OTR is working with a 1 year, 8-month-old child who was diagnosed with developmental delay. The child receives early intervention services because of delays in motor, self-care, adaptive, and social emotional domains. Findings from the initial evaluation also suggest that the child has self-regulatory difficulties and has difficulty falling asleep and staying asleep. Which intervention is MOST EFFECTIVE to address INITIALLY to assist the family with their sleeping concerns? a. limit nighttime sounds and lights to promote better sleep habits and routine b. use a weighted blanket during naps and nighttime sleeping times c. create a plan for reinforcing the child's independence in falling asleep d. establish a consistent quiet nap and nighttime routine with the family.

d. establish a consistent quiet nap and nighttime routine with the family. Limiting nighttime sounds and lights are good strategies to use as part of the process once a plan has been established. Weighted blankets may be beneficial but require monitoring for safety. Creating a plan is a component but not the most effective.

A graduate student with an anxiety disorder reports feeling confused about the future. During OT eval, the client relates decreased feelings of competence for a chosen field of study and overall personal causation. Which is the best initial action for the therapist to take in response to the client's concerns? a. administer a vocational interest inventory b. provide activities related to the client's chosen field of study c. refer the client to the state office of vocational and educational services d. establish short-term goals with high potential for attainment

d. establish short-term goals with high potential for attainment This provides the individual with successful experiences needed to develop a sense of competence. Once these skills are developed, need for further vocational exploration can be determined.

a teenager with congenital bilateral above-elbow amputation is referred to OT to increase independence in ADLs. te adolescent wants to be independent in donning and doffing shirts. Which action is best for the therapist to take first? a. refer the patient to a prosthetist b. evaluate abdominal strength c. evaluate the use of AE equipment in dressing d. evaluate trunk and LE ROM

d. evaluate trunk and LE ROM upon referral, the therapist must complete eval. Thus the first step is to evaluate trunk and LEs to determine if the patient can use the LE to dress, as many people with amelia do. Hip ROM is essential to be able to don shirts with the feet. After ruling out adapted techniques. the next step would be to attempt adapted equipment. Evaluating abdominal strength might be helpful but it is not the first action to take.

A newly hired OTA is instructed by the director of rehab to supervise two hospital volunteers as they learn how to assist patients in safely completing bed to wheelchair transfers. The OTA informs the supervising OT of the director's request. Which is the first action the OT should take in response to this request? a. advise the OTA to comply with the request b. advise the OTA to refuse the request c. observe the OTA to assess service competence in transfer training. d. explain to the director why the request is inappropriate

d. explain to the director why the request is inappropriate volunteers are not trained health care professionals and they cannot perform transfers with patients. Therefore, the OTA cannot comply with the directors request. Although inappropriate to decline a supervisor's request, it is important this is done for safety and to prevent other inappropriate requests from happening.

The parent of 2 elementary school aged children receives home care hospice services due to metastasized bone cancer. The client is in pain and has poor endurance and decreased muscle strength. The client requires mod assist with self-care and dressing. Which is the best intervention for the OT to incorporate into sessions with this parent. a. training in energy conservation techniques for self-care and dressing b. training in joint protection techniques for self-care and dressing c. using biofeedback to reduce the client's pain d. exploring play activities for the parent to do with the children

d. exploring play activities for the parent to do with the children a major focus of hospice is to maintain the individual's control over his/her life while enabling engagement in meaningful activities. There are many activities suitable for elementary aged children that can be completed by a person with decreased endurance and muscle strength.

An OTR is applying the procedures of the constraint-induced movement therapy with a client. The client is attempting to scoop food from a bowl with her right hand, which was affected by a stroke. The OTR has been providing her with verbal cues during the activity. At the end of the activity, the OTR engaged the client in a discussion regarding the task performance. The OTR is using which component of motor learning? a. repetition b. intrinsic feedback c. shaping d. extrinsic feedback

d. extrinsic feedback this is the external information from the environment received by the client while he or she is performing the task. Motor learning involves repetition. Intrinsic is the sensory experience felt by the client. During shaping, the desired behavior is or motor action is performed in small, successive increments.

An OTR is working with a 4-year-old child with developmental delays to promote hand function to allow the child to hold a crayon more efficiently. What activity is MOST EFFECTIVE to develop this hand skill? a. rolling clay into a ball b. finger painting c. crumpling paper d. eye dropper painting

d. eye dropper painting This helps promote a precision grasp which helps form the tripod grasp. Rolling clay works on regulation of pressure. Finger painting develops tactile awareness and finger isolation. Crumpling paper is an activity to develop in-hand manipulation skill of translation.

a young adult recently diagnosed with depression and anorexia nervosa is a consumer of services at a psychosocial clubhouse. The client attends individual OT sessions once a week and several evening and weekend groups. When completing an activities schedule and questionnaire, the client identifies the clubhouse groups and part-time work as an editorial assistant as the only activities completed each week. In high school the client was captain of the swim team, played tennis, and worked in an after-school activities program for young children. the client attends a goal-setting group for persons with depression. Which is the most helpful approach for the OT leading this group to take with members? a. encourage the members to discuss long-range planning b. say as little as possible to allow the members to do most of the talking c. remain cheerful and upbeat to alleviate the member's depression d. facilitate reality-testing of the members' negative thinking

d. facilitate reality-testing of the members' negative thinking People with depression often interpret events with exaggerated negativity. Developing the members' ability to test and correct negative thinking is an important precursor to developing goals. Long-range planning is too limited when an individual is recently diagnosed. People with depression may also have difficulty engaging in conversation. Cheerful, upbeat moods may cause them to question the therapist's empathy.

An adolescent incurred a SCI injury at the C5 level. During a family caregiver education session, the OT instructs family members in the provision of PROM to the patient's wrist and fingers. Which method of PROM should the therapist teach the family members to perform? a. extend the fingers with the wrist extended b. flex the fingers with the wrist flexed c. flex and extend the fingers with the wrist in a neutral position d. flex the fingers with wrist extension and extend the fingers with wrist flexion

d. flex the fingers with wrist extension and extend the fingers with wrist flexion a major goal of OT for a person with a SCI at C5 or C6 is to enhance the development of the tenodesis grasp. This choice will result in shortening of the flexor tendons without compromising joint ROM. Unlike other patterns, the shortening will enhance the tenodesis grip.

an OT has been counseled twice by the director of the OT department for overcharging patients for services rendered. The behaviors continue despite repeated verbal and written reprimands from the director. Which action is best for the director to take next? a. report the therapist's behavior to NBCOT b. formulate a plan of action to modify the therapist's behaviors c. terminate the employee according to facility procedures d. follow facility procedures for reporting employee misconduct

d. follow facility procedures for reporting employee misconduct the therapist has already been reprimanded more than once and has not altered the behavior. The facility's HR director will investigate the situation and determine if it is best to place the therapist on probation or terminate the therapist.

A single parent of 2 children, a 4 y/o and a 6 y/o, is referred to OT services in a rehab center. The client had a brain tumor removed 1 month ago. The referral states that the client has residual cognitive-perceptual deficits. Screening indicates that the client's sensori-motor abilities are WFL. During the eval, the OT observes that the client has difficulty dealing with increasing amounts of stimuli. This is observed in all modalities. Based on this observation, which cognitive-perceptual dysfunction does the therapist record as present when reporting the client's eval results? a. inability to abstract b. poor organizational skills c. poor semantic memory d. generalized attention deficit

d. generalized attention deficit attention requires the ability to focus on a specific stimulus without being distracted by external or internal stimuli.

a child with mild CP receives OT in a preschool setting. Which intervention approach should the therapist employ to facilitate development of typical grasp patterns? a. place soft foam tubing around objects to be grasped b. analyze the present components of the child's grasp c. analyze the missing components of the child's grasp d. grade the sizes and shapes of objects to be grasped

d. grade the sizes and shapes of objects to be grasped gradation of the size and shape of items enables the therapist to begin with items within the child's grasp capabilities and then adding different items as the client progresses. Soft tubing is a compensation approach and can be used if the child does not develop typical grasp patterns. Analyzing components of grasp is part of eval and reeval, not the intervention process.

An OT conducts an intervention session with a client recovering from a CVA to develop transfer skills. The client has a co-morbidity of epilepsy with primary generalized seizures. As the client stands to complete a transfer from the wheelchair to the bed, the client reports feeling sensations that are indicative of an aura. Which is the best immediate action for the therapist to take in response to this situation? a. provide reassurance and ask for guidance from the physician b. return the client to a seated position in the wheelchair until the sensations pass c. end the session so the client can rest d. guide the person into a sidelying position on the bed

d. guide the person into a sidelying position on the bed An aura is a brief warning stage before the tonic phase of an epileptic seizure. Some symptoms include numbness and unexplained smells. The tonic phase of a seizure includes loss of consciousness, stiffening of the body, heavy and irregular breathing, drooping, skin pallor, and occasional bladder leakage. Since a person could fall and harm his or herself during a seizure, the therapist must immediately take action to ensure the clients safety.

An interdisciplinary educational team at a local public high school recently reassessed a 17-year-old student who sustained traumatic brain injury at the age of 6 years. After several years of intervention in the schools, the reassessment results revealed significant limitations in completing written classroom assignments and homework. What is the NEXT logical step for the educational team to consider? a. develop discipline-specific goals b. introduce the students c. excuse the student from written work d. identify appropriate transition goals

d. identify appropriate transition goals At 17 years, it is necessary to begin thinking about transitioning to adulthood. Discipline specific goals would likely already be in place since the student has been receiving services. Once the student has transition goals, the team may choose to use a computer.

The director of an OT driver rehab program must determine if the program is meeting its objectives. Which of the following is most important for the director to measure during the eval of the program outcomes? a. participant's satisfaction with program services b. family member's satisfaction with program services c. the effect of the program on participant's automobile insurance premiums d. the percentage of participants who successfully return to driving

d. the percentage of participants who successfully return to driving

an OT is working with a child presenting with sensory-seeking behaviors and under reactivity to touch and movement. The child has an unusually high activity level, inability to self-calm, motor impulsivity, and frequent touching and handling of items in the environment. Using Ayers classic sensory approach, which would be most effective for the therapist to use with this child to facilitate an adaptive response? a. a pre-determined schedule of sensory activities designed by the OT b. the child's passive participation in a variety of vestibular and proprioceptive experiences c. use of a sensory void environment to promote self-regulation d. individualized therapeutic activities based on the interests of the child

d. individualized therapeutic activities based on the interests of the child Classic Ayres SI treatment is based on the principles of inner drive and active involvement of the child.

A co-worker in the therapy department complains excessively during work hours of personal problems. The co-worker does not respond to colleague's statements that this is disruptive to their work. Which action is best for the OT? a. call the employee assistance program for the co-worker b. talk with the co-worker using a client centered approach c. tactfully and firmly redirect the co-worker to work related issues d. inform the department supervisor of the situation

d. inform the department supervisor of the situation the supervisor should be informed of the situation because the employee's personal problems are entering the workplace. Tactfully redirecting the co-worker is a common technique to avoid being drawn into the problems of others, but it will not address the problem in the department.

2 weeks after beginning kindergarten, a 5 yr old with myelomeningocele develops sudden on set of headaches, vomiting, irritability and "sunken" appearance of eyes without signs of a fever. Which action is the best for the OT to take in response to the child's presenting symptoms? a. bring the child to the school nurse to determine if the observed changes in the child are indicative of the stomach flu b. inform the parents that they should immediately have a physician examine the child for tethered cord c. refer the child to the school psychologist for an assessment of school anxiety d. inform the parents that they should immediately have a physician examine the child for a shunt malfunction

d. inform the parents that they should immediately have a physician examine the child for a shunt malfunction These symptoms, along with seizures, are all symptoms of a shunt malfunction, which is a medical emergency. Stomach flu could be a reason for these symptoms but given the clustering of symptoms, the client should be checked for shunt malfunction. The signs of tethered cord include difficulties with bowel and bladder, gait disturbances, and/or foot deformities.

A patient with fibromyalgia is receiving OT to reduce pain and promote flexibility during ADL tasks. The patient expresses the intent to discontinue treatment based on information obtained during an online search which questioned the value of therapy for this diagnosis. Which of the following actions should the OT take first in response to the patient's statements? a. confront the inaccuracy of these statements and provide current evidence based research about the benefits of therapy for persons with fibromyalgia b. reassure the patient that the physician ordered therapy; therefore, it will be beneficial c. respect the patient's wishes and discontinue OT, offering to provide therapy in the future if symptoms exacerbate d. inform the patient that online information can be inaccurate and provide literature about the health benefits of therapy for fibromyalgia

d. inform the patient that online information can be inaccurate and provide literature about the health benefits of therapy for fibromyalgia The internet can be informative and resourceful, but it can also be inaccurate. It is not beneficial to directly confront the inaccuracies of the information with the latest research. This stance can be overwhelming and lead to the person feeling belittled. Reminding the person of the physician's orders does not respect the patients feelings or address the concerns. Discontinuing treatment overlooks the need to provide accurate info first to insure that the patient is making an informed decision.

an OT consults at a day program for individuals with dementia. Which group are best for the therapist to recommend the program include? a. reality orientation groups b. cognitive-behavioral groups c. parellel groups d. instrumental groups

d. instrumental groups according to Mosey's taxonomy of groups, instrumental groups help individuals function at their highest level for as long as possible. They provide supportive structured environments and appropriate activities that prevent regression, maintain function, and meet mental health needs. Groups that focus on the use of memory can be very frustrating and counter-therapeutic for people with dementia. Cognitive-behavioral groups require intact cognition and are at too high a level for these individuals. parellel groups can be indicated for individuals with dementia but a schedule should not be comprised primarily of parallel groups for they are limiting in their potential for social interaction.

An OT works with an individual with chest and UE burns. During the intervention session, the client expresses vague fears about personal safety at home and asks the therapist to advocate for an extension in the discharge date. According to the medical record, the client had incurred the burns during a cooking accident. Which is the therapist's best initial response to the client's stated concerns? a. encourage the client to speak to the social worker about discharge plans b. assure the client that pre-discharge fears are normal and expected c. document the client's concerns and recommend an extended length of stay d .invite the client to expand upon the nature of those concerns.

d. invite the client to expand upon the nature of those concerns the OT needs more info to determine the basis of the client's fears. Referring the client to a social worker may be helpful, but will not address the fears in the moment.

an OT works at a psychosocial clubhouse. The therapist is leading a closed group on stress management that has been together for several months. One of the members shares some concerns about personal safety at home. Which is the therapist's best response to the member's expressed concerns? a. tell the individual you will privately talk about the concerns after the group b. immediately send the individual to see the clubhouse's social worker c. assure the individual that the concerns reflect normal anxieties. d. invite the individual to share more details about the concerns

d. invite the individual to share more details about the concerns It can be difficult for an individual to share concerns about personal safety. This person clearly felt safe in the group to be able to voice his/her concerns. The therapist needs more information about the safety concerns. Delaying the attainment of this info until after the group is not necessary and can risk the person changing his/her mind. The person may not have rapport with the social worker and may not feel comfortable telling the social worker his/her concerns.

an OT implements an activity program at a hospital that provides long-term care to elderly persons with severe and persistent mental illness. The OT uses a sensorimotor approach to guide the program design. Which activity is best for the OT to incorporate into a group? a. ballroom dancing using pictures of feet on the floor as visual cues for dance steps b. exercising along with a video routine c. a sing-along using songbooks and recorded music d. keeping balloons afloat while music plays.

d. keeping balloons afloat while music plays according to the sensorimotor approach, activities should not require the individuals to think about the steps needed to complete the activity. Activities should be spontaneous, "non-cortical" and fun. Other activities require the individuals to think and follow some type of predetermined format.

a rehab hospital is interested in starting a driver rehab program. Which must the OT hired to develop this program do first? a. determine the cost of commercially available driving rehab programs b. develop admission criteria for program participants c. develop a marketing plan to obtain referrals d. learn the state's driving laws and requirements

d. learn the state's driving laws and requirements state laws are essential for a therapist developing a driver rehab program. Cost is important but not the greatest priority. One must know laws to avoid potential program liability.

An OT conducts a group for long-term inmates in a forensic facility. Which of the following would be best for the therapist to focus on during the group? a. vocational planning strategies b. remedial educational activities c. money management skills d. leisure management techniques

d. leisure management techniques persons in a forensic setting have a significant amount of time that is not filled by productive or meaningful activity. Vocational planning would be most effective for people who are nearing release. Remedial educational activities would be provided by an educational professional.

an individual with a traumatic above-elbow amputation has received a body-powered prosthesis. To train the person in the operation of the terminal device, which of the following should the OT do initially during intervention? a. teach the person how to control the elbow joint b. combine training of TD use with training for elbow joint movement c. lock the elbow in full extension and teach only TD control d. lock the elbow in 90* of flexion and teach only TD control

d. lock the elbow in 90* of flexion and teach only TD control locking the elbow joint into flexion places the TD in functional position for the completion of activities with the TD. The question specifically asks about training for TD operation, not control of the elbow joint. Control of the elbow joint would occur independent of TD control training because the elbow joint must be locked for TD use in an above-elbow prosthesis.

An OTR is working with a 4-year-old who has mild to moderate tactile defensiveness. Which activity is the MOST EFFECTIVE sensory-based approach for improving tolerance of touch for this child? a. writing in shaving cream on the table top b. swinging in prone position in a net swing c. playing with a feather boa during a dress up activity d. log-rolling to snuggly wrap the body blanket

d. log-rolling to snuggly wrap the body blanket This activity provides deep touch stimuli that is usually more tolerable than light touch. Shaving cream is a teature that is often an irritant and can cause anxiety for kids with tactile defensiveness. Swinging is for tactile, vestibular, and proprioceptive input which can be overwhelming. Children with this diagnosis may not like being in prone due to tactile sensation on their stomach. Feather boas offer light sensation which is bothersome for these children.

An OTR is working with a teenage youth with a spinal cord injury (SCI) at the C5 level. The client would like to transition to more independent feeding as he is currently being assisted by his family when eating. What is the BEST device that can support independent feeding for this client? a. tenodesis splint b. cock-up splint c. resting hand splint d. long opponens splint

d. long opponens splint This type of splint is used to support the wrist due to weak wrist muscles. This splint in conjunction with other AE allows the user to move food to the mouth. A tenodesis splint is ideal for those with a C6 injury. The cock-up splint is used to immobilize and protect the wrist often for carpal tunnel. The resting hand splint treats moderate flexion contractures.

Person in this case: has degenerative joint disease and incurred an injury on the right hand. Client has pain, stiffness, and extreme temp changes in the hand. The person also has edema and hypertrophic changes consistent with CRPS. which is the most appropriate activity for the OT to recommend for the person to complete at home? a. doing light craft work b. playing a tabletop game c. performing relaxation exercises d. manually washing a car

d. manually washing a car Stress loading activities are recommended for CRPS type 1

an individual with bilateral proximal weakness identifies a goal of independence in self-feeding. Which equipment is most beneficial for the OT to recommend for goal attainment? a. extended long-handled utensils b. built-up handled utensils c. an electric feeder d. mobile arm supports

d. mobile arm supports these effectively compensate for UE weakness. Extended long-handled utensils are indicated for individuals with decreased ROM. Built-up handles are indicated for individuals with decreased grasp. An electric feeder is for people with no functional use of the UEs.

an OT working in a hand clinic conducts an eval. During the eval, the therapist uses the Froment's sign. When documenting the outcome of this eval procedure, which is the most accurate for this therapist to state was assessed? a. sensation of median nerve b. motor function of the median nerve c. sensation of the ulnar nerve d. motor function of the ulnar nerve

d. motor function of the ulnar nerve the Froment's sign assesses the motor function of the adductor pollicis which is innervated by the ulnar nerve. With an ulnar nerve injury, this attempt results in flexion of the distal joint of the thumb.

When measuring the ROM of a client's elbow, the OT records a flexion measurement of 145* . Which is most accurate for the therapist to document based on this measurement? a. hypomobility b. dysfunctional elbow ROM c. Hypermobility d. normal elbow ROM

d. normal ROM Normal elbow ROM is zero to 135-150*

During a group session at an adult day care program an older adult consistently complains that everyone is mumbling. After the group, which action should the OT take in response to those statements? a. notify the client's primary care physician that the person exhibited evidence of paranoia b. collaborate with the program director to remove groups from the client's plan c. document the objective data about the complaints in the person's chart d. notify the primary care physician that the person may need an audiological eval

d. notify the primary care physician that the person may need an audiological eval it is common for older adults to experience hearing loss.

a person recovering from skin grafting due to full thickness burns is prescribed splints to immobilize the grafted areas in anti-deformity positions. Which splint wearing schedule is best to include in the splinting protocol for the first 72 hrs post-surgery? a. 1 hr on, with 10 min off b. 4 hrs on, with 20 min off c. 8 hrs on , with 30 minutes off d. on at all times except dressing changes

d. on at all times except dressing changes

A person blinded in an accident begins an OT program for the persons with vision loss. The OT collaborates with the individual to develop an intervention plan. Which should be included in the plan as an initial focus intervention? a. developing the client's computer skills b. adapting meal prep techniques c. exploring vocational interests d. organizing the client's morning routine

d. organizing the client's morning routine This is the most appropriate initial goal because it can help the person feel confident and serve as a basis for organizing the rest of the person's home and work routines. The development of meal prep would initially focus on non-cooked foods and meals because the risk of burns and fires are a serious concern for many newly blind individuals. Therefore, this would be more of a long-term goal, similar to the other options.

the parents of a school-aged child with Rett Syndrome ask the OT for activities to do at home to help their child regain lost skills. Which of the following should the therapist include in the HEP for the parents to do with their child? a. encourage the child to use pressure distribution techniques b. use 4-step sequencing cards to increase attention c. give positive feedback for active ROM performance d. perform passive ROM to prevent contractures

d. perform passive ROM to prevent contractures Rett syndrome is a genetic progressive disorder in which motor, cognitive, social, and language skills deteriorate. Regardless of the child's current functional level, children with this progressive condition cannot regain lost skills. Therefore, the HEP must focus on preventing complications and maintaining function. PROM is an activity to prevent contractures which are a complication of this progressive condition. The child will not be able to respond to encouragement to use pressure distribution techniques.

An adult who incurred a TBI 3 months ago is referred to a home care agency to receive OT services. The client's referral states that cognition is at a level VII of the Rancho Los Amigos scale and that grasp and shoulder mobility are limited. During the initial interview, the client reports being frustrated by the inability to independently engage in the previously enjoyed and personally meaningful activity of cooking. The client reports frequently losing place when reading recipes and an inability to find things in the kitchen During meal prep tasks the client ignores items on the left side of the counter. The therapist decides to use a compensatory functional approach to improve the client's performance. Which is the most appropriate method for the therapist to use during intervention to develop meal preparation skills? a. encourage bilateral activities b. place all items on the right side of the counter c. practice scanning activities d. place brightly colored placemat on the left side of the counter

d. place a brightly colored placemat on the left side of the counter This provides an external cue which the person can be taught to scan for during meal prep. This anchoring technique is a basic compensatory functional approach. Encouraging bilateral activities and placing all items on the right side of the counter would not address the client's performance deficit.

A young child with decreased trunk stability is working on toileting training with the OTR. The OTR observes the child needs a smaller toilet seat size to improve sitting support. Which is the BEST option for the OTR to try with the small child? a. special tomato seat b. bath chair c. raised toilet seat d. plastic seat reducer ring

d. plastic seat reducer ring this is used to provide a smaller seating option. A bath chair supports bathing needs for children with significant motor impairments. A raised toilet seat is for people needing additional support in the LEs. Special tomato seats are intended to provide comfortable familiar toilet seats.

An adult with right hemisphere damage from a CVA is referred to OT. Which deficits will the patient most likely demonstrate upon eval? a. negative, self-deprecating comments and depression b. slow responses and cautious behavior c. hesitancy and fearfulness d. poor judgment and difficulty with safety issues

d. poor judgement and difficulty with safety issues a patient with this diagnosis will have impulsive behaviors and poor judgement. Slow responses, hesitancy, and cautious behavior are indicative of a left CVA. The individuals behaviors can appear slow because of the hesitancy caused by perceptual or motor planning problems.

An OTR is working in the neonatal intensive care unit and just finished an initial motor evaluation of an infant born at 32 weeks' gestation. The infant's muscle tone is low and appears to have a lax head lag. At rest, the infant's lower extremities remain extended and the hips are in external rotation. What INITIAL action should the OTR take based on this assessment? a. train family in infant massage techniques b. splint to promote flexion of extremities c. tummy time activities in prone position d. position infant in flexion with blanket support

d. position infant in flexion with blanket support This action combats the extension and positions the child in flexion so that he or she can begin to bring hands to midline, grasp, and self-soothe. Infant massage is a valuable treatment in the NICU but it would not be the initial action because the positioning is the greatest concern. Tummy time in prone is a good next step after proper positioning. This intervention is a good activity for strengthening.

An OT receives a referral to evaluate an individual's executive functioning following a mild CVA. Which are the most relevant foci for this evaluation? a. the person's attention and memory b. the person's job interests and efficacy c. the person's spatial relations and praxis d. the person's initiation and planning

d. the person's initiation and planning Executive functions are higher level abilities needed to perform unstructured multi-step activities and tasks. the 4 main components of executive functioning are volition, planning, purposeful action, and effective performance. Attention and memory are considered primary cognitive capacities.

An OT provides intervention for a 4 yr old with developmental delays characterized by the persistence of primitive postural reflexes. The child demonstrates age-appropriate cognitive skills. Which is the best play activity for the therapist to incorporate into the childs intervention? a. spinning on a swing b. putting a puzzle together c. lying on the floor and playing a game of marbles d. pretending to be an explorer climbing through caves

d. pretending to be an explorer climbing through caves Pretending to crawl through caves can help facilitate the integration of primitive postural reflexes. It is also imaginative, which is appropriate for her age. Spinning on a swing is a fast vestibular activity indicated for sensory integration dysfunction. Puzzles and playing marbles require fine motor skills and would be too advanced for a child with these reflexes.

A 9-year-old girl is referred to an outpatient pediatric clinic for a screening for occupational therapy services. Her mother reports to the OTR that her daughter constantly breaks her pencils while doing her math homework, frequently falls out of her chair at home, and hugs her family and friends at inappropriate times. Based on the other's observations, in which area does the girl require the MOST intervention? a. praxis b. sensory defensiveness c. sequencing d. proprioceptive awareness

d. proprioceptive awareness Praxis is the ability to conceptualize, plan, and execute a non-habitual motor act. The girl in this scenario seems to be more sensory seeking.

The administrator of a home care agency tells the OT to submit all intervention plans to the client's third party payers, prior to the implementation of treatment. Which is the most accurate term for the therapist to use when documenting these actions? a. concurrent review b. peer review c. utilization review d. prospective review

d. prospective review concurrent is the evaluation of ongoing intervention programs. Utilization review is a plan to review the use of resources within a facility to determine medical necessity and cost efficiency.

A client with loss of protective sensation in bilateral hands is receiving occupational therapy services. Which should the OTR utilize during intervention? a. increase the amount of force required to maintain grip of an object b. visually occlude client during intervention to promote discrimination. c. persist in activities for prolonged time periods, promoting sensation d. protect from exposure to sharp items or extreme temperatures

d. protect from exposure to sharp items or extreme temperatures Protecting the clients from these objects/sensations is recommended for safety. Increasing force is contraindicated. Occluding vision over prolonged periods is contraindicated.

A client with Charcot-Marie-Tooth disease participates in a reeval session at a wheelchair clinic. The client reports difficulty keeping both feet on the wheelchair's footrests which is the most effective action for the OT to take to address the client's stated concerns? a. implement an exercise routine to strengthen LE b. elevate the footrests c. provide heel loops on the footrests d. provide ankle straps on the footrests

d. provide ankle straps on the footrests Charcot-Marie-Tooth disease is a neuropathic muscular atrophy characterized by progressive weakness of the distal muscles of the arms and feet. It does not respond to strengthening exercises. Elevating footrests are contraindicated for edema control, LE extension contractures, and long leg casts. Heel loops on footrests only prevent the feet from slipping posteriorly which would not be adequate in this situation.

A OTR is working with a child with autism spectrum disorder on improving play and ideational praxis. Which intervention is the LEAST APPROPRIATE to use with this child? a. have the child help you build an obstacle course b. take turns between imitating the child and having the child imitate you c. use movie characters and stories to begin to introduce pretend play d. provide choice boards to aid in understanding what is to be done

d. provide choice boards to aid in understanding what is to be done This intervention would decrease a child's ability to play and use ideation. All other options would help improve play and ideational praxis.

A person recovering from a CVA accident has left-sided weakness and dysphagia. Which of the following is the most effective direct intervention approach to help the person successfully swallow ingested food? a. provide pureed, thick liquids b. provide thermal stimulation to the inferior faucial arches c. tilt the person's head back and toward the left side d. provide small, warm boluses

d. provide small, warm boluses direct intervention for oral motor control involves techniques that utilize a bolus. Providing thermal stim to the inferior faucial arches can elicit a swallow response, but its considered an indirect treatment method. Tilting the head back is contraindicated because it increases choking risks.

an elementary school-aged child with Ducheene's muscular dystrophy receives OT services. The family establishes a goal of maintaining the child's leisure and social participation. Which is the best activity for the OT to recommend that the family pursue with this child? a. electronic sports b. adapted little league baseball c. wheelchair basketball d. recreational swimming

d. recreational swimming This is a good leisure and social participation activity. It can also be helpful in maintaining the child's functional level as long as possible. The hand-eye coordination of electronic games will likely be too difficult. Baseball and basketball have mobility and coordination requirements that would be too difficult.

the staff of an acute inpatient medical unit consists of an OT and an entry-level OTA. The admission rate has increased and the OT is having difficulty completing evals in a timely manner. Which is the best action for the therapist to take in response to these practice realities? a. evaluate patients screened by nursing staff as appropriate for OT b. plan intervention based on the PT eval c. train the OTA to independently complete the eval process d. redesign the program to allow more time for evals

d. redesign the program to allow more time for evals only OTs can screen for OT. The OTs can redesign the program to include eval methods that are more efficient

During evaluation, a client with an ulnar nerve laceration determines meal preparation to be a goal of rehabilitation. Which is the MOST effective strategy to promote safety during meal preparation for this client? a. avoid the use of the hand during activity b. substitute pinch to ring and small finger c. hold heavy pots and pans close to body d. visual protection of the affected hand

d. visual protection of the affected hand When a nerve is lacerated, sensory loss is extensive. Client must be able to use vision in order to protect against burns. Avoiding the use of the hands allows for occupational withdrawal and dependency. Substitution is necessary with the median nerve. Holding heavy pots addresses ergonomic principles but these are not specific to this injury.

a middle school-aged child with a sensory processing disorder participates in weekly OT sessions in a pediatric private practice. Initial eval has identified the presence of symptoms consistent with a sensory-based motor disorder. The child has shown no improvement in coordination, equilibrium, and motor planning for the past 2 months. The parent reports that the child continues to exhibit difficulties with play, learning, and social participation. Which action is the best for the OT to take in response to these observations and parental report? a. provide the parent with a home program and discharge the child from therapy. b. refer the child to a pediatric social worker to explore potential resistance to therapy c. increase the frequency of therapy to 2 sessions per week to increase the child's engagement d. reeval the child to determine deficit areas that are contributing to dysfunction

d. reeval the child to determine deficit areas that are contributing to dysfunction the child is not making gains in OT with the current approach. Therefore, the OT should reeval the child to obtain info that can help determine an intervention plan that would more effectively meet the child's needs. the therapist should not continue to threat the child in areas that show no progress. Increasing the frequency without a revised intervention plan will not facilitate functional improvements. There is no info provided to indicate that the child is resistant to therapy.

a 3 y/o child with left spastic hemiplegia due to CP is evaluated for early intervention services. During the eval, the OT observes behaviors that seem to indicate the presence of visual deficits. Based on these observations, which action should the OT take? a. complete a motor-free visual perceptual assessment b. complete a developmental vision assessment c. refer the child to an optician d. refer the child to an optometrist

d. refer the child to an optometrist prior to conducting a visual perceptual eval, an anatomical visual assessment is required to determine visual acuity. Optometrists are the professionals who are qualified to perform eye exams to determine visual acuity, level of impairments, and damage or disease in the visual system.

An OT works in a program for survivors of domestic violence. Which of the following would the therapist do when using a client-centered approach? a. offer specific concrete behavioral suggestions for dealing with confrontations b. respond to the client's self-deprecating comments with positive feedback on personal characteristics c. reinforce only the participants neural comments about themselves and personal skills d. reflectively paraphrase the participant's statements to clarify their feelings.

d. reflectively paraphrase the participant's statements to clarify their feelings the main principle of client-centered therapy is that it is directed by the person. An intervention goal when working with survivors of DV is to increase their awareness of feelings. The best approach is to accept what the client is saying unconditionally and reflect back what he/she is saying in a nonjudgmental manner.

during an accreditation self-study, the OT department personnel review all charts. An OT notices that a colleague fills out the review forms without reading the charts. Which action is best for the therapist to take in response to this observation? a. talk to the colleague directly b. report the incident to the director of medical records c. report the colleague to the accrediting agency d. report the observation to the OT supervisor

d. report the observation to the OT supervisor the therapist directly observed a colleague committing an act that directly violates the AOTA code of ethics. Any overt violation of this code should be reported to the direct supervisor.

an individual with moderate intellectual disability moves into a group home. An initial goal established for this resident is the development of socially acceptable table manners. The OT uses a behavior modification approach to achieve this goal. Which intervention technique is the best for the therapist to include in the design of the group home dining experience? a. negative reinforcement for socially inappropriate behaviors b. clear explanations of behavior expected during dinning c. clear explanations about the effects of inappropriate behaviors on others d. rewards for socially appropriate behaviors

d. rewards for socially appropriate behaviors a behavior modification program provides positive reinforcement for desired behaviors.

A family is looking for additional seating options in the home for their 6-year-old daughter with a genetic disorder that impacts muscle control and use. The child uses a wheelchair for mobility and several adaptive seating options. The family would like to explore a supportive option for the children's playroom during movie times or family game time. What would be the BEST option for the OTR to recommend? a. corner chair b. therapy bench c. rifton modular wooden chair with pommel d. rockers pediatric chill out chair

d. rockers pediatric chill out chair this chair provides alternative options for people not in their wheelchair. The chair includes a soothing rocking motion that helps calm the individual.

an individual with OCD participates in a vocational group. The client asks the OT to speak to the supervisor of the transitional employment program. The client is concerned that compulsive behaviors are interfering with job performance and may result in the loss of a new TEP placement. Which is the therapist's best response to these expressed concerns? a. instruct the client to speak directly to the TEP supervisor about the right to receive reasonable accommodations b. schedule a re-eval of the client's work and behavior skills c. assure the client that it is natural to have initial difficulties at a new job d. schedule an appointment with the client and the TEP supervisor

d. schedule an appointment with the client and the TEP supervisor meeting with the individual and the supervisor will enable the therapist to provide support for the individual's concerns. The therapist can also facilitate a dialogue about the specific difficulties that the individual is experiencing from both the employee and employer perspectives.

An adolescent with a complete myelomeningocele at the T9 level is diagnosed with diabetes. The OT meets with the adolescent and a nurse practitioner to review the impact of this new diagnosis on the teen's health and daily routines. Which is the most important for the OT to emphasize during this discussion? a. skin inspection performed by a parent b. UE strengthening c. Self-directed pressure relief d. self-initiated pressure relief

d. self-initiated pressure relief the person with a complete myelomeningocele at the T9 level has absent sensation in the lower extremities and buttocks. Therefore, frequent pressure relief is essential to prevent skin breakdown. UE strengthening can be helpful to provide pressure relief, however, a T9 lesion does not affect the UEs so the teens strength is likely WFL. The teen will be capable of self initiating pressure relief and does not have to self-direct the performance of this activity.

an older adult with persistent balance difficulty and a history of recent falls (2 in the last 3 mo) receives home care OT services. During the initial session, which client factors are most important for the OT to consider? a. spinal musculoskeletal changes secondary to degenerative joint disease b. cardiovascular endurance and level of dyspnea during IADLs c. mental functions of attention and orientation during functional mobility d. sensory functions and sensory organization of balance

d. sensory functions and sensory organization of balance a critical component of balance control is sensory input from the somatosensory, visual, and vestibular receptors. With age, these systems change and can become compromised causing safety concerns. There is no information to indicate that the person has a cognitive deficit, degenerative joint disease, or cardiopulmonary disorder.

an individual recovering from a TBI is assessed to be at a Level VI of the Rancho los amigos scale. Which should the OT use to implement treatment? a. sensory stimulation activities such as moving to music b. repetitive self-care tasks such as brushing hair c. community re-entry activities such as taking the bus d. simple meal prep tasks such as making sandwiches

d. simple meal prep tasks such as making sandwiches at level VI the individual is appropriate and goal-directed but can become confused. Cues are required. Sensory stimulation activities would be appropriate for level III. Repetitive self-care tasks would be appropriate for Level V.

During an intervention session, a client complains of dry mouth due to prescribed medications. Which is the most effective strategy for the OT to suggest the client to manage this side effect? a. suck on ice b. suck on hard candies c. drink iced tea d. sip water

d. sip water Sucking on ice/candies presents a possible chocking risk. Iced tea contains caffeine, which can increase dehydration.

an OT provides a wellness and prevention education series to a community senior center. The topic of the week is joint protection. Which joint principle should the therapist include in the presentation? a. stand diagonally to the side of containers to be opened or closed to maximize torque b. work through the pain experienced during activities by performing stretching exercise c. preserve joint ROM and muscle strength by using the minimal effort to perform an activity. d. start an activity only if it can be immediately stopped when it requires capabilities beyond existing capabilities.

d. start an activity only if it can be immediately stopped when it requires capabilities beyond existing capabilities. the other choices are opposite of joint protection principles.

An OTR is planning interventions for a client with chronic inflammatory disease in bilateral hands who demonstrates minimal inflammation and low endurance during tasks. Which interventions would be MOST appropriate for this client? a. resting hand splints b. restrictive splinting c. ROM to point of pain d. stretch to end range

d. stretch to end range This scenario indicates RA. At this stage in the disease, stretching to end range will promote increased ROM, strength, and endurance. Splints are beneficial to decrease acute inflammation, decrease pain, and protect joints. The client is not currently experiencing significant inflammation.

The supervisor of an acute inpatient unit requests that a recently hired entry-level OT write summaries of several eval sessions that were completed by another therapist. The evaluating therapist had to leave work unexpectedly due to a medical emergency and is not expected to return to work. Which is the best response for the therapist to make in response to this request? a. comply with the supervisor's request but as for the supervisor to co-sign the notes b. request time to complete an independent eval of each individual previously evaluated c. report the supervisor's request to the facility's administration d. suggest that the therapist's evaluation results be documented by the supervisor

d. suggest that the therapist's evaluation results be documented by the supervisor it is not appropriate for a peer to document results of an evaluation session in which he/she did not participate. It is acceptable for a supervisor to provide documentation based on staff's input, as long as the documentation reports that it is based upon the work of a given staff member. In an acute inpatient setting, there is insufficient time to complete another eval. There is nothing to report to administration at this time.

a client expresses an interest in playing a computer game with another group member during a leisure skills group. The OT reviews the client's cognitive evaluation and agrees that the game is a good choice for the client's cognitive level. After 15 min of engaging in the computer game, the client rubs both eyes, looks around, and reports trouble focusing. Which should the OT do in response? a. provide verbal encouragement for the client to complete the game before taking a break b. suggest the client and the other member play a different video game that is easier for the client c. discontinue the session and advise the client to select a different leisure activity to do with the other member d. suggest the client and other member talk about the games progress in between completing their turns.

d. suggest the client and other member talk about the games progress in between completing their turns. the client appears to be experiencing visual strain. The best choice is to decrease this strain by providing perioding breaks.

an OT completes the discharge plan for an individual with a right CVA who has completed a 3-week inpatient rehab program. The person is right-hand dominant. The patient exhibits residual cognitive perceptual deficits but seems unaware of these problems. Which discharge recommendation is best for the OT to discuss with the patient and the patient's family? a. an extension of LOS b. a full-time home health aide c. assistance with personal care d. supervision for cooking

d. supervision for cooking a right CVA results in left-sided deficits, decreased judgement, and diminished insight. These can pose safety risks during cooking. Since the person is right-hand dominant. the ability to perform many personal tasks will likely remain intact. 3 weeks of rehab has likely taught the person many strategies and skills, therefore, full time supervision or increased LOS is not warranted.

A client has a 3 yr history of MS. One of the client's disabling symptoms is a persistent and severe diplopia, which leaves the client frequently nauseated and unable to complete desired activities. Which adaptive strategy is most effective for the OT to recommend the client use during ADLs and IADLs? a. wearing magnifying glasses b. wear prism glasses c. wear shaded glasses d. wear an eye patch on one eye

d. wear an eye patch on one eye double vision can be managed by patching one eye. Typically they are on a patch schedule where they alternate which eye is patched. Eye patching is an entry-level skill that can be performed by an OT.

An adult with hereditary ataxia receives home care OT services. Recently, the client has become more withdrawn and the client's spouse has become more verbal about caregiver strain. During an intervention, the OT works with the client on attaining the goal of dressing independently. The therapist notices and comments on several large bruises on the middle section of the clients back. The client tearfully states that a bad fall that morning had caused these and that the progression of the ataxia is becoming too difficult to handle. Which is the best action for the OT to take in response to the observed bruises and the client's statements? a. provide reassurance and support of the client's legitimate feelings of loss b. conduct an eval of the home to remove items that can contribute to falls c. report the incident to the local domestic violence hotline d. supportively question the client about the incident

d. supportively question the client about the incident while persons with ataxia do often fall, it would require a very unusual fall to incur bruises in the middle of the back. The possibility that the injuries were the result of an incident of domestic violence must seriously be considered given the location of the injury and increasing evidence of caregiver strain. Due to the serious nature, the therapist must give the client an opportunity to disclose this information. Providing a home eval and reassurance can be beneficial but do not assess the immediate risks. Contacting the domestic violence hotline at this point is premature and will likely increase the clients fear of disclosure and escalate the situation.

A home-care OT plans intervention for an individual with agoraphobia with panic attacks. The OT plans to use a cognitive-behavioral approach. Which approach is the most effective for the therapist to use during intervention? a. a token reward system b. behavioral extinction c. sensorimotor tasks d. systematic desensitization

d. systematic desensitization This is a cognitive-behavioral technique that introduces graded levels of the anxiety-producing stimuli

A middle school student with learning disabilities exhibits no behavioral problems in the classroom. However, whenever the class is in a line waiting to switch classrooms, the student becomes agitated and often pushes classmates. The OT consultant advises the teacher that this behavior may be indicative of an underlying problem. Which of the following is the most accurate for the therapist to identify as a potential disorder warranting further eval? a. gravitational insecurity b. a conduct disorder c. antisocial tendencies d. tactile defensiveness

d. tactile defensiveness The tactile stimuli due to closeness of peers in a line can become overwhelming to an individual with tactile defensiveness.

after evaluating a client with right sided weakness and decreased motor control, an OT decides to use interventions based on the PNF approach. When applying PNF principles, which of the following actions should the therapist have the client do during an intervention session to increase the use of the right UE and hand? a. reach overhead with the right hand to retrieve a dish out of a higher cabinet and set it down on the countertop in front. b. reach to the right side to retrieve an item out of the refrigerator at hip height and place it into the left hand to set it on the countertop to the left c. use both hands together to pour juice out of a heavy other into a glass on a countertop d. take items out of a dishwasher on the right side and reach across the body to place them in the upper cabinet on the opposite side

d. take items out of a dishwasher on the right side and reach across the body to place them in the upper cabinet on the opposite side PNF is a technique that involves diagonal patterns of movement and rotational trunk movements.

Upon eval, the therapist determines that the patient has right homonymous hemianopsia. The therapist provides recommendations to modify the patient's room to enhance independence. Which are the most appropriate recommendations for the therapist to make for the placement of the patient's telephone and radio? a. telephone on the left side and radio on left side b. telephone on the right side and radio on the right side c. telephone on the right side and radio on the left d. telephone on the left side and radio on the right

d. telephone on the left side and radio on the right the telephone must be laced within the person's intake visual field (which in this case is the left) so that they can access it in case of an emergency. But the radio can be placed outside the person's visual field to encourage scanning of their environment.

An individual attends an outpatient parenting skills group. The person has a history of serious recurrent depression and is taking Nardil. The client complains of recurrent headaches and difficulty focusing during the day (e.g when helping kids with homework). Which action is the best for the OT to make in response to the client's expressed concerns. a. instruct the client in stress reduction techniques b. ask the group for suggestions on how to deal with the parenting stress of homework. c. suggest that the individual consult with a nurse practitioner for headache relief strategies. d. tell the client you will be notifying the psychiatrist of these complaints.

d. tell the client you will be notifying the psychiatrist of these complaints Nardil is an MAOI. It has serious side effects when a person eats foods that contain the amino acid Tyramine. Tyramine increases BP and can lead to stroke. Headache and heart problems are the first sign of a problem.

An individual is evaluated for a repetitive stress disorder. The individual complains of numbness and tingling of the thumb, index, middle, and radial half of the ring finger and aching pain in the proximal forearm. The client states that these symptoms are not evident at night. The OT notes a positive Tinel's sign. Which site should the therapist document as the location of this sign for this client? a. the wrist b. the guyon's canal c. the elbow d. the forearm

d. the forearm these signs indicate pronator teres syndrome. A positive Tinel's sign at the forearm is consistent with this syndrome which is a medial nerve compression between the two heads of the pronator teres. Symptoms are similar to carpal tunnel except there is aching pain in the forearm with no night symptoms.

a 2-month old infant with bilateral hip dislocation is being discharged from an acute pediatric facility. The OT has developed a home program for the parents of this firstborn child. Which is most important for the OT to determine before instructing the parents in the details of this home program? a. the child's insurance reimbursement plan b. the parent's level of formal education c. the family's home environment d. the parent's degree of anxiety

d. the parent's degree of anxiety Prior to providing the details of the home program, the OT should determine the parent's level of anxiety since excess anxiety can impact their comprehension and retention of the instructions given. While other factors may also be considered, they do not represent immediate priorities for hospital-based instruction.

a child with a diagnosis of TBI is evaluated by an OT. The child presents with extension of both UEs and flexion of both LEs following a stimulus of neck extension. When interpreting this observation, which statement is most accurate for the therapist to document. a. the presence of + ATNR, which is "abnormal" and has reappeared after the TBI b. the presence of + STNR, which is "normal" and not affected by the TBI c. the presence of a + ATNR, which is "normal" and not affected by the TBI d. the presence of a + STNR which is "abnormal" and has reappeared after the TBI

d. the presence of a + STNR which is "abnormal" and has reappeared after the TBI STNR is facilitated by flexion of the neck followed by extension of the neck. The response is that the flexion of the neck results in bilateral UE flexion and bilateral LE extension. Positive reactions for this reflex are only normal up to 4-6 months of age.

An individual who is scheduled for a right total hip arthroplasty is referred to OT. A posterolateral surgical approach will be used. The client has expressive aphasia resulting from a CVA incurred 4 years ago. Medicare is the client's only health care benefit. The client states that private payment for any health care services or equipment is not possible at this time. What should the OT focus on during preoperative OT interventions? a. the performance of UE and LE strengthening exercises b. the performance of IADLs in sitting rather than standing c. the practice of non-weight bearing crutch walking d. the use of modified techniques to perform bed transfers and ADLs

d. the use of modified techniques to perform bed transfers and ADLs These essential ADLs will be done daily during the postop rehab phase. The client will be more successful if he/she is prepared and understands the techniques. There is nothing in the scenario to indicate poor UE and LE strength. Performing tasks in sitting is more stressful on the hip than standing and are contraindicated. Restrictions for weight bearing depend on several factors that may not be determined until after the procedure.

An OT working in an early intervention meets with the parents of an infant who cries a lot and has difficulty being soothed. The parents are concerned that they are unable to comfort their child. What is the most effective strategy for the therapist to recommend to the parents? a. loosely wrap the infant in a blanket b. provide frequent and rapid changes in movement c. do nothing, as the infant's behavior is typical d. tightly wrap the infant in a blanket

d. tightly wrap the infant in a blanket this can provide controlled and consistent firm presser that is non-aversive and soothing. Loosely wrapping the infant provides inconsistent and variable input that can increase discomfort. Rapid and frequent changes are contraindicated because they can increase tone and stimulate arousal.

following an exacerbation of postpolio syndrome, a client is referred to OT for a wheelchair eval to enable independent mobility. The client wears bilateral hip-knee ankle foot orthoses to provide support during independent transfers and brief standing periods throughout the workday. The client's insurance is Medicare. Which is the best seat-width measurement for the OT to recommend for the client's wheelchair prescription? a. two inches wider than the widest point across the individual's hips or thighs b. four inches wider than the widest person across the individual's hips or thighs while wearing orthoses c. a standard adult seat width to ensure Medicare reimbursement d. two inches wider than the point across the individual's hips or thighs while wearing orthoses

d. two inches wider than the point across the individual's hops or thighs while wearing orthoses this will allow for ease of movement in and out of the chair. A standard adult chair would not meet this individual's needs. Four inches would be to large and could complicate mobility through doorways or hallways

A client with a traumatic below-elbow amputation of the dominant right arm has participated in OT for prosthetic training. The therapist prepares a discharge plan that incorporates recommendations to facilitate independence in meal preparation and feeding. Which method is best for the therapist to recommend the client use to cut meat? a. hold a regular knife in the terminal device and hold a regular fork in the left hand b. hold a knife with a built-up handle in the terminal device and hold the plate with the left hand c. use a one handed technique with a rocker knife in the left hand and hold the place either the terminal device d. use a regular fork in the terminal device and hold a regular knife in the left hand.

d. use a regular fork in the terminal device and hold a regular knife in the left hand this is the safest option for using the prosthesis to cut meat. It is also the method most people use to cut food so it will facilitate the person's resumption of typical activities.

a high school student is referred to OT for ADL training. The student has non-spastic CP resulting in right side hemiparesis and decreased muscle tone. During a transition planning meeting, the student's teacher reports the student has had several accidents during a meal prep class. As part of the transition plan, the OT will teach the student adaptive techniques used in a kitchen setting to compensate for right side weakness. Which of the following adaptations should the therapist recommend to improve the student's independence in preparing meals safely? a. prepare foods that do not require cutting b. use a microwave oven to cook food c. use a weighted knife to cut food d. use oven mitts that extend to the elbows

d. use oven mitts that extend to the elbows Non-spastic CP will exhibit decreased or fluctuating muscle tone and can induce hemiparesis which indicates the arm and leg on one side of the body is weakened. Adaptations to an activity or environment can prevent injuries in the kitchen.

a patient who is s/p left frontal lobe ischemia has difficulty bearing weight through the right LE during reaching activities. The OT implements a Motor Re-Learning Program. Which is the best intervention for the therapist to provide according to this approach? a. therapeutic handling to affect the CNS b. a stool to sit on during reaching activities c. joint compression to the right LEs during reaching activities d. verbal and visual feedback while practicing reaching

d. verbal and visual feedback while practicing reaching A MRP approach provides verbal and visual feedback to give a person the input needed to make postural and limb adjustments. Therapeutic handling to affect the CNS is consistent with a neurodevelopmental therapy approach. Joint compression is a technique used for sensory modulation disorders (tactile defensiveness, hyper/hypo sensitivities, and sensory seeking).

An OTR is working in an acute, psychiatric unit at the community hospital. The OTR receives a referral for an adolescent with a history of self-mutilation. The mother reports that she discovered her daughter's condition after washing her bloody bed sheets. She also mentions that her daughter is under extreme stress at school as she is in the running to become valedictorian of her high school. What is the MOST BENEFICIAL interest for the client to explore upon discharge? a. joining the debate team to improve college applications b. meal prep to make quick easy meals and save time c. going to movies and hanging out with friends on weekends d. yoga and mindfulness groups that meet at high school

d. yoga and mindfulness groups that meet at high school People who self mutilate have a hard time calming the mind and body. These strategies are helpful to replace self-harm behaviors. Joining the debate team is too stressful and competitive. Going to movies and hanging with friends is beneficial but it would be more effective to explore stress management and emotional regulation activities.


Related study sets

Positioning and Projections of the Wrist

View Set

BA370 All Chapters Exam 1 SDSU Gaffen

View Set

PP: RNSG 1538 Family Mastery Quiz

View Set

Learning Module 32: Compositional Stoichiometry

View Set